Part 2 : 11/11/10 07:41:20 Question 1 · PDF file22,000 pumps were produced although 25,000...

92
Question 1 - CMA 694 3-29 - Performance Measurement One approach to measuring divisional performance is return on investment. Return on investment is expressed as operating income A. Divided by total assets. B. Divided by the current year's capital expenditures plus cost of capital. C. Divided by fixed assets. D. Minus imputed interest charged for invested capital. A. By definition. The formula for return on investment is operating income divided by total assets (or average total assets). B. This answer is incorrect. See the correct answer for a complete explanation. C. This answer is incorrect. See the correct answer for a complete explanation. D. This answer is incorrect. See the correct answer for a complete explanation. Question 2 - CMA 1291 3-9 - Performance Measurement A segment of an organization is referred to as an investment center if it has A. Authority to make decisions affecting the major determinants of profit including the power to choose its markets and sources of supply and significant control over the amount of invested capital. B. Authority to make decisions over the most significant costs of operations including the power to choose the sources of supply. C. Authority to provide specialized support to other units within the organization. D. Authority to make decisions affecting the major determinants of profit including the power to choose its markets and sources of supply. A. By definition, an investment center is a part of the business that has the authority to make decisions affecting the major determinants of profit including the power to choose its markets and sources of supply and significant control over the amount of invested capital. B. This is the definition of a cost center. C. This is the definition of a service center, which is a type of cost center. D. This is the definition of a profit center. An investment center also has a responsibility for invested capital. Question 3 - CMA 693 3-20 - Overhead Tiny Tykes Corporation had the following activity relating to its fixed and variable overhead for the month of July. Actual costs Fixed overhead $120,000 Variable overhead 80,000 Flexible budget Variable overhead 90,000 Applied Part 2 : 11/11/10 07:41:20 (c) HOCK international, page 1

Transcript of Part 2 : 11/11/10 07:41:20 Question 1 · PDF file22,000 pumps were produced although 25,000...

Question 1 - CMA 694 3-29 - Performance Measurement

One approach to measuring divisional performance is return on investment. Return on investment is expressed as operating income

A. Divided by total assets.B. Divided by the current year's capital expenditures plus cost of capital.C. Divided by fixed assets.D. Minus imputed interest charged for invested capital.

A. By definition. The formula for return on investment is operating income divided by total assets (or average total assets).

B. This answer is incorrect. See the correct answer for a complete explanation.

C. This answer is incorrect. See the correct answer for a complete explanation.

D. This answer is incorrect. See the correct answer for a complete explanation.

Question 2 - CMA 1291 3-9 - Performance Measurement

A segment of an organization is referred to as an investment center if it has

A. Authority to make decisions affecting the major determinants of profit including the power to choose its markets and sources of supply and significant control over the amount of invested capital.B. Authority to make decisions over the most significant costs of operations including the power to choose the sources of supply.C. Authority to provide specialized support to other units within the organization.D. Authority to make decisions affecting the major determinants of profit including the power to choose its markets and sources of supply.

A. By definition, an investment center is a part of the business that has the authority to make decisions affecting the major determinants of profit including the power to choose its markets and sources of supply and significant control over the amount of invested capital.

B. This is the definition of a cost center.

C. This is the definition of a service center, which is a type of cost center.

D. This is the definition of a profit center. An investment center also has a responsibility for invested capital.

Question 3 - CMA 693 3-20 - Overhead

Tiny Tykes Corporation had the following activity relating to its fixed and variable overhead for the month of July.

Actual costs Fixed overhead $120,000 Variable overhead 80,000 Flexible budget Variable overhead 90,000 Applied

Part 2 : 11/11/10 07:41:20

(c) HOCK international, page 1

Fixed overhead 125,000 Variable overhead spending variance 2,000F Production volume variance 5,000U

The fixed overhead efficiency variance is

A. $5,000 favorable.B. Never a meaningful variance.C. $3,000 favorable.D. $3,000 unfavorable.

A. There is no fixed overhead efficiency variance because fixed costs are not related to levels of output and therefore are unable to be used efficiently or inefficiently.

B. There is no fixed overhead efficiency variance because fixed costs are not related to levels of output and therefore are unable to be used efficiently or inefficiently.

C. There is no fixed overhead efficiency variance because fixed costs are not related to levels of output and therefore are unable to be used efficiently or inefficiently.

D. There is no fixed overhead efficiency variance because fixed costs are not related to levels of output and therefore are unable to be used efficiently or inefficiently.

Question 4 - CMA 1295 3-4 - Overhead

Variable overhead is applied on the basis of standard direct labor hours. If, for a given period, the direct labor efficiency variance is unfavorable, the variable overhead efficiency variance will be

A. The same amount as the labor efficiency variance.B. Favorable.C. Unfavorable.D. Zero.

A. This answer is incorrect. See the correct answer for a complete explanation.

B. This answer is incorrect. See the correct answer for a complete explanation.

C. An unfavorable direct labor efficiency variance means that more direct labor hours were used for production than were budgeted. The variable overhead variance is calculated as follows: (Standard Activity Level for Actual Output - Actual Activity Level) * Standard Application Rate. In this formula actual activity level is actual direct labor hours since it is the allocation base. We know that actual labor hours exceeded the standard, which gives us a negative, or unfavorable, variable overhead variance.

D. This answer is incorrect. See the correct answer for a complete explanation.

Question 5 - CMA 1291 3-7 - Performance Measurement

Which one of the following items would most likely not be incorporated into the calculation of a division's investment base when using the residual income approach for performance measurement and evaluation?

A. Fixed assets employed in division operations.

Part 2 : 11/11/10 07:41:20

(c) HOCK international, page 2

B. Division inventories when division management exercises control over the inventory levels.C. Division accounts payable when division management exercises control over the amount of short-term credit used.D. Land being held by the division as a site for a new plant.

A. When the denominator for residual income is calculated, it would normally include assets that are in use and expected to generate a return. Fixed assets that are used in division operations are expected to provide a return in the current period and would usually be included in the residual income calculation.

B. When the denominator for residual income is calculated, it would normally include assets that are in use and expected to generate a return. Inventories that management controls are expected to provide a return in the current period and would usually be included in the residual income calculation.

C. When the denominator for residual income is calculated, it would normally include assets that are in use and expected to generate a return. Accounts payable that management controls are expected to provide a return in the current period and would usually be included in the residual income calculation.

D. When the denominator for residual income is calculated, it would normally include assets that are in use and expected to generate a return. Therefore, land that is being held for a future plant would not be included because it is not expected to provide any return in the current period.

Question 6 - CMA 1290 3-9 - Overhead

Franklin Glass Works' production budget for the year ended November 30 was based on 200,000 units. Each unit requires two standard hours of labor for completion. Total overhead was budgeted at $900,000 for the year, and the fixed overhead rate was estimated to be $3.00 per unit. Both fixed and variable overhead are assigned to the product on the basis of direct labor hours. The actual data for the year ended November 30 are presented as follows.

Actual production in units 198,000 Actual direct labor hours 440,000 Actual variable overhead $352,000 Actual fixed overhead $575,000

The fixed overhead applied to Franklin's production for the year is

A. $600,000.B. $594,000.C. $484,200.D. $575,000.

A. This is the budged fixed overhead amount based on 200,000 units of production. In the calculation of applied overhead we have to use the standard input for the actual level of output. The formula is Standard Application rate * Standard input for the Actual level of output. See the correct answer for a complete explanation.

B. The amount overhead applied is calculated as Standard Application rate * Standard input for the Actual level of output. Fixed overhead is assigned to the product on the basis of direct labor hours. The fixed overhead rate was estimated to be $3.00 per unit and two hours are required to produce one unit. Thus, application rate is $1.50 ($3.00 / 2). The standard direct hours allowed for actual output are 396,000 (2 * 198,00 actual production in units). The applied overhead equals $594,000 ($1.50 * 396,000). We also could calculate the amount of applied overhead using actual units produce and unit fixed overhead application rate ($3.00 * 198,000) which gives the same answer $594,000.

C. The amount of overhead applied is calculated as Standard Application rate * Standard input for the Actual level of output. See the correct answer for a complete explanation.

Part 2 : 11/11/10 07:41:20

(c) HOCK international, page 3

D. This is the amount of actual fixed overhead $575,000. In fact, the overhead applied and actual overhead differ in most cases, as in this particular case. See the correct answer for a complete explanation.

Question 7 - CMA 697 3-29 - Performance Measurement

Listed below is selected financial information for the Western Division of the Hinzel Company for last year.

Account Amount

(thousands) Average working capital $ 625 General and administrative expenses 75 Net sales 4,000 Average plant and equipment 1,775 Cost of goods sold 3,525

If Hinzel treats the Western Division as an investment center for performance measurement purposes, what is the before-tax return on investment (ROI) for last year?

A. 19.79%B. 34.78%C. 16.67%D. 22.54%

A. This answer fails to subtract general and administrative costs in the calculation of income. See the correct answer for a complete explanation.

B. This answer is incorrect. See the correct answer for a complete explanation.

C. ROI is calculated as Net Income divided by Average Total Assets (or Investments). Average assets are equal to the sum of average plant and equipment plus working capital or $2,400,000 ($625,000 + $1,775,000). Net Incomes equals Net Sales minus COGS and G&A expenses or $400,000 ($4,000,000 - $3,525,000 - $75,000). Now we can calculate ROI: $400,000 / $2,400,000 = 16,67%.

D. This answer does not include working capital in average total assets. See the correct answer for a complete explanation.

Question 8 - CMA 1294 3-27 - Overhead

Water Control Inc. manufactures water pumps and uses a standard cost system. The standard factory overhead costs per water pump are based on direct labor hours and are as follows:

Variable overhead (4 hours at $8/hour) - $32Fixed overhead (4 hours at $5*/hour) - $20Total overhead cost per unit - $52* Based on a capacity of 100,000 direct labor hours per month.

The following additional information is available for the month of November:

22,000 pumps were produced although 25,000 had been scheduled for production.94,000 direct labor hours were worked at a total cost of $940,000.The standard direct labor rate is $9 per hour.

Part 2 : 11/11/10 07:41:20

(c) HOCK international, page 4

The standard direct labor time per unit is 4 hours.Variable overhead costs were $740,000.Fixed overhead costs were $540,000.

The variable overhead spending variance for November was

A. $48,000 unfavorable.B. $40,000 unfavorable.C. $60,000 favorable.D. $12,000 favorable.

A. This is the variable overhead efficiency variance. See the correct answer for a complete explanation.

B. This is the fixed overhead spending variance. See the correct answer for a complete explanation.

C. This answer uses the budgeted hours (100,000), not the actual hours (94,000). The variable overhead spending variance is calculated as follows: (Standard Application Rate - Actual Application Rate) * Actual Quantity.

D. The variable overhead spending variance is calculated as follows: (Standard Application Rate - Actual Application Rate) * Actual Quantity. The standard application of variable overhead is $8 per labor hour, actual quantity in labor hours is 94,000. The actual application rate of variable overhead multiplied by actual quantity of activity level is the actual amount of variable overhead, or $740,000. Thus, the variable overhead spending variance is $12,000 favorable ($8 * 94,000 - $740,000). The budgeted amount is greater than the actual amount and that is why the variance is favorable.

Question 9 - CMA 687 4-17 - Variances - General

Selo Imports uses flexible budgeting for the control of costs. The company's annual master budget includes $324,000 for fixed production supervisory salaries at a volume of 180,000 units. Supervisory salaries are expected to be incurred uniformly throughout the year. During the month of September 15,750 units were produced, and production supervisory salaries incurred were $28,000. A performance report for September would reflect a budget variance of

A. $350 favorable.B. $350 unfavorable.C. $1,000 unfavorable.D. $1,000 favorable.

A. The variance is unfavorable because the budgeted amount of fixed overhead is less than actual amount.

B. In this solution the salaries are treated as variable costs instead of fixed costs. See the correct answer for a complete explanation.

C. The fixed (or Spending) overhead budget variance is equal to the difference between the budgeted and actual amounts of fixed overhead. The budgeted (static budget) amount of fixed overhead was $27,000 ($325,000 / 12). Thus, the fixed overhead budget variance is ($1,000) unfavorable ($27,000 - $28,000).

D. The variance is unfavorable because the budgeted amount of fixed overhead is less than actual.

Question 10 - CMA 691 3-24 - Performance Measurement

Most firms use return on investment (ROI) to evaluate the performance of investment center managers. If top management wishes division managers to use all assets without regard to financing, the denominator in the ROI

Part 2 : 11/11/10 07:41:20

(c) HOCK international, page 5

calculation will be

A. Total assets employed.B. Shareholders' equity.C. Working capital plus other assets.D. Total assets available.

A. If the company uses total assets employed, there will be no motivation for the managers to make better use of the assets that are currently not employed.

B. Using shareholders' equity takes into account only those assets financed with equity. It does not include assets that were financed by debt, or other liabilities.

C. By using working capital in this measure the company is excluding assets that are financed by short-term obligations. This is because working capital is current assets minus current liabilities.

D. If the company does not care what the financing of the assets used is, the company should use total assets available as the denominator in ROI. This will mean that managers are expected to use all of the assets to generate return for the company.

Question 11 - CMA 692 3-23 - Responsibility Accounting

The WK Company uses a performance reporting system that reflects the company's decentralization of decision making. The departmental performance report shows one line of data for each subordinate who reports to the group vice president. The data presented show the actual costs incurred during the period, the budgeted costs, and all variances from budget for that subordinate's department. The WK Company is using a system called

A. Cost-benefit accounting.B. Flexible budgeting.C. Responsibility accounting.D. Program budgeting.

A. This is not an accounting method.

B. Flexible budgeting is the creation of a budget for different levels of outputs. This is not what is described in the question.

C. Responsibility accounting is a system in which costs are allocated to managers and/or departments based on who is responsible for the incurrence of the costs. This is the method described in the question.

D. This is a budgeting system and it does not describe the situation in the question.

Question 12 - CMA 1294 3-28 - Overhead

Water Control Inc. manufactures water pumps and uses a standard cost system. The standard factory overhead costs per water pump are based on direct labor hours and are as follows:

Variable overhead (4 hours at $8/hour) - $32Fixed overhead (4 hours at $5*/hour) - $20Total overhead cost per unit - $52* Based on a capacity of 100,000 direct labor hours per month.

Part 2 : 11/11/10 07:41:20

(c) HOCK international, page 6

The following additional information is available for the month of November:

22,000 pumps were produced although 25,000 had been scheduled for production.94,000 direct labor hours were worked at a total cost of $940,000.The standard direct labor rate is $9 per hour.The standard direct labor time per unit is 4 hours.Variable overhead costs were $740,000.Fixed overhead costs were $540,000.

The variable overhead efficiency variance for November was

A. $200,000 unfavorable.B. $60,000 favorable.C. $48,000 unfavorable.D. $96,000 unfavorable.

A. This is the difference between the actual direct labor costs over actual variable overhead costs which does not mean anything. See the correct answer for a complete explanation.

B. This answer uses capacity, not actual hours. See the correct answer for a complete explanation.

C. The variable overhead efficiency variance is calculated as follows: (Standard Activity Level for Actual Output - Actual Activity Level) * Standard Application Rate. Or, putting the numbers into the equation, (88,000 - 94,000) * $8 = ($48,000). Because this is a negative number, the variance is unfavorable. The standard activity level is calculated as 4 hours of direct labor per unit and since 22,000 units were produced during the period, this gives us 88,000 hours as the standard activity level.

D. This is the difference between standard hours allowed for the actual output and capacity hours. See the correct answer for a complete explanation.

Question 13 - CIA 589 IV-14 - Other

The following data are available for July:

Budget Actual Sales 40,000 units 42,000 units Selling price $6 per unit $5.70 per unit Variable cost $3.50 per unit $3.40 per unit

What is the sales quantity variance for July?

A. $12,600 unfavorable.B. $12,000 unfavorable.C. $5,000 favorable.D. $4,600 favorable.

A. The variance is favorable as quantity actually sold (42,000) is greater than budgeted (40,000). See the correct answer for a complete explanation.

B. The variance is favorable as quantity actually sold (42,000) is greater than budgeted (40,000). See the correct answer for a complete explanation.

C. The sales quantity variance measures the impact of the difference in sales volume and is calculated as follows:(Actual Sales Volume - Budgeted Sales Volume) * Standard Contribution per Unit. The standard

Part 2 : 11/11/10 07:41:20

(c) HOCK international, page 7

contribution per unit is $2.50 ($6.00 sales price - $3.50 variable costs). The sales volume variance is $5,000 favorable [(42,000 - 40,000) * $2.50].

D. The sales quantity variance measures the impact of the difference in sales volume and is calculated as follows:(Actual Sales Volume - Budgeted Sales Volume) * Standard Contribution per Unit. We have to use the standard contribution margin ($2.50), not the actual contribution margin ($2.30) as is used in this answer. See the correct answer for a complete explanation.

Question 14 - CIA 1191 IV-19 - Transfer Pricing

A carpet manufacturer maintains a retail division consisting of stores stocking its brand and other brands, and a manufacturing division that makes carpets and pads. An outside market exists for carpet padding material in which all padding produced can be sold. The proper transfer price for padding transferred from the manufacturing division to the retail division is

A. Variable manufacturing division production cost plus variable selling and administrative cost.B. Variable manufacturing division production cost.C. The market price at which the retail division could purchase padding.D. Variable manufacturing division production cost plus allocated fixed factory overhead.

A. When there is an external market for the product, market price is almost always the best transfer price to use.

B. When there is an external market for the product, market price is almost always the best transfer price to use.

C. The basic issue of transfer prices is how much should one unit of a company charge another unit of the same company for its goods or services. The goal in setting a transfer price is that the method used will stimulate the department managers to do what will provide the greatest benefit to the company as a whole, rather than to act in their own interest. Since there is an outside market exists for padding produced can be sold there best transfer price is a market price.

D. When there is an external market for the product, market price is almost always the best transfer price to use.

Question 15 - CIA 1191 IV-18 - Responsibility Accounting

A company plans to implement a bonus plan based on segment performance. In addition, the company plans to convert to a responsibility accounting system for segment reporting. The following costs, which have been included in the segment performance reports that have been prepared under the current system, are being reviewed to determine if they should be included in the responsibility accounting segment reports:

I. Corporate administrative costs allocated on the basis of net segment sales.

II. Personnel costs assigned on the basis of the number of employees in each segment.

III. Fixed computer facility costs divided equally among each segment.

IV. Variable computer operational costs charged to each segment based on actual hours used times a predetermined standard rate; any variable cost efficiency or inefficiency remains in the computer department.

Of these four cost items, the only item that could logically be included in the segment performance reports prepared on a responsibility accounting basis would be the

A. Personnel costs.

Part 2 : 11/11/10 07:41:20

(c) HOCK international, page 8

B. Fixed computer facility costs.C. Variable computer operational costs.D. Corporate administrative costs.

A. An allocation basis should be one in which the managers are able to control the incurrence of costs by their department. In an allocation basis of personnel, an individual segment manager is able to control only their own staff numbers, not the staff numbers of other segments. Therefore, this allocation of costs would not be appropriate for responsibility accounting.

B. The equal allocation of fixed computer costs is not appropriate for responsibility accounting because the individual mangers are not able to control how much the computer department spends. This also does not relate to the usage of the computer department by each segment.

C. An allocation based on the actual usage of the item being allocated is an appropriate method of allocation for responsibility accounting. Therefore, these variable computer costs can be included in the responsibility accounting report.

D. An allocation basis should be one in which the managers are able to control the incurrence of costs by their department. In an allocation basis of sales, an individual segment manager is able to control only their own sales, not the sales of other segments. Therefore, this allocation of costs would not be appropriate for responsibility accounting.

Question 16 - CIA 1192 IV-22 - Responsibility Accounting

An organization employs a system of internal reporting that furnishes departmental managers with revenue and cost information on only those items that are subject to their control. Items not subject to the manager's control are not included in the performance reports. This method of accounting is known as

A. Responsibility accounting.B. Absorption cost accounting.C. Segment reporting.D. Contribution margin reporting.

A. Responsibility accounting is a system in which the cost of revenue data is reported based on who (manager or division) is able to control them or is responsible for them. This is the system described in the question.

B. Absorption cost accounting relates to the allocation of fixed costs. This is not descriptive of the method used by this company.

C. Segment reporting is the reporting of results by segment - either product line, geography or some other distinguishing characteristic. This is not descriptive of the method used by this company.

D. Contribution margin reporting breaks costs down into fixed and variable costs. While this may be used in conjunction with responsibility accounting, this by itself is not describing the method used by the company.

Question 17 - CMA 695 3-27 - Variances - General

Clear Plus, Inc. manufactures and sells boxes of pocket protectors. The static master budget and the actual results for May appear below.

Actuals Static

Budget

Part 2 : 11/11/10 07:41:20

(c) HOCK international, page 9

Unit sales 12,000 10,000 Sales $132,000 $100,000 Variable costs of sales 70,800 60,000 Contribution margin 61,200 40,000 Fixed costs 32,000 30,000 Operating income $29,200 $10,000

Which one of the following statements concerning Clear Plus, Inc.'s actual results for May is correct?

A. The flexible budget variance is $8,000 favorable.B. The sales price variance is $32,000 favorable.C. The sales volume variance is $8,000 favorable.D. The flexible budget variable cost variance is $10,800 unfavorable.

A. To answer such a question we have to solve for each possible suggested statement. It is better to start with the easiest ones. The flexible budget variance equals the difference between actual operating results and flexible budget operating results. The flexible budget contribution margin equals actual units times standard contribution margin or $48,000 ($4.00 standard contribution margin * 12,000 actual quantity). The flexible budget operating income equals flexible budget contribution margin less master budget (static budget) fixed costs: $18,000 ($48,000 - $30,000). Now we can determine the flexible budget variance as $11,200 favorable ($29,200 - $18,000). So this statement is not correct.

B. To answer such a question we have to solve for each possible suggested statement. It is better to start with the easiest ones. The sales price variance measures the impact of the difference in contribution margin per unit caused by variance in price and is calculated as follows: (Actual Contribution per unit - Standard Contribution per unit) * Actual Quantity. Actual contribution per unit is $5.10 ($61,200 ÷ 12,000), the standard contribution margin is $4.00 ($40,000 ÷ 10,000). The sales price variance is therefore $13,200 favorable ($5.10 - $4.00) * 12,000. So this statement is not correct.

C. To answer such a question we have to solve for each possible suggested statement. It is better to start with the easiest ones. The sales volume variance measures the impact of difference in sales volume and is calculated as follows: (Actual Sales Volume - Budgeted Sales Volume) * Standard Contribution per Unit. The standard contribution per unit is $4.00 ($40,000 ÷ 10,000). The sales volume variance is therefore $8,000 favorable (12,000 - 10,000) * $4.00. The variance is favorable as the actual quantity of units sold is greater than budgeted. So this statement is correct.

D. To answer such a question we have to solve for each possible suggested statement. It is better to start with the easiest ones. The flexible budget variable cost variance is the difference between the flexible budget variable costs and actual variable costs. The standard variable cost is calculated using static (master) budget figures: $60,000 ÷ 10,000 = $6.00. The flexible budget variable cost is calculated as the standard unit variable cost multiplied by the actual level of output ($6.00 * 12,000 = $72,000). The flexible budget variable cost variance is $1,200 favorable ($72,000 - $70,800). So this is not a correct statement.

Question 18 - CMA 1289 4-2 - Variances - Material and Labor

An unfavorable direct labor efficiency variance could be caused by a(n)

A. Unfavorable material usage variance.B. Unfavorable variable overhead spending variance.C. Favorable fixed overhead volume variance.D. Favorable variable overhead spending variance.

Part 2 : 11/11/10 07:41:20

(c) HOCK international, page 10

A.

An unfavorable direct labor efficiency variance means that more time was spent in production than budgeted. A number of reasons could cause this: poor performance of production employees, poor product design, waste, theft, poor material quality, etc.

An unfavorable material usage variance means that more material was spent to produce units of finished product. Poor material quality could cause an unfavorable material usage variance. The poor quality material could also require more time spent by production workers to perform their task. That is why an unfavorable direct labor efficiency variance could be caused by unfavorable material usage variance.

B. The variable overhead spending variance is the difference between the standard application rate and the actual application rate (the actual application rate is calculated as the actual overhead costs ÷ the actual usage of the allocation base) multiplied by the actual quantity of the application base. The difference in application rates (positive or negative) does not relate to the direct labor usage (efficiency) variance.

C. The fixed overhead volume variance is the difference between the amount of fixed overhead applied (standard rate * standard input for the actual level of output) and the budgeted amount of fixed overhead. The difference (positive or negative) between applied and budgeted fixed overhead does not relate to the direct labor usage (efficiency) variance.

D. The variable overhead spending variance is the difference between the standard application rate and the actual application rate (the actual application rate is calculated as the actual overhead costs ÷ the actual usage of the allocation base) multiplied by the actual quantity of the application base. The difference in application rates (positive or negative) does not relate to the direct labor usage (efficiency) variance.

Question 19 - CMA 1290 3-10 - Overhead

Franklin Glass Works' production budget for the year ended November 30 was based on 200,000 units. Each unit requires two standard hours of labor for completion. Total overhead was budgeted at $900,000 for the year, and the fixed overhead rate was estimated to be $3.00 per unit. Both fixed and variable overhead are assigned to the product on the basis of direct labor hours. The actual data for the year ended November 30 are presented as follows.

Actual production in units 198,000 Actual direct labor hours 440,000 Actual variable overhead $352,000 Actual fixed overhead $575,000

Franklin's fixed overhead volume variance for the year is

A. $55,000 unfavorable.B. $6,000 unfavorable.C. $19,000 favorable.D. $25,000 favorable.

A. The fixed overhead volume variance is difference between the amount of fixed overhead applied (standard rate * standard input allowed for the actual level of output) and the budgeted amount of fixed overhead. See the correct answer for a complete explanation.

B. The fixed overhead volume variance is difference between the amount of fixed overhead applied (standard rate * standard input allowed for the actual level of output) and the budgeted amount of fixed overhead. The fixed overhead rate was estimated to be $3.00 per unit and 200,000 units were scheduled for production. The total budgeted fixed overhead was $600,000 ($3.00 * 200,000). Two hours are required to produce one unit. Thus, the application rate is $1.50 ($3.00 / 2). The standard direct hours allowed for actual output are 396,000 (2 * 198,00 actual production in units). The applied overhead equals $594,000 ($1.50 * 396,000). We also could

Part 2 : 11/11/10 07:41:20

(c) HOCK international, page 11

calculate the amount of applied overhead using actual units produce and unit fixed overhead application rate ($3.00 * 198,000) which gives the same answer $594,000. Thus, fixed overhead volume variance is $6,000 unfavorable ($594,000 - $600,000). The applied amount was less than the budgeted amount which means that some amount of overhead wasn't applied to the final product i.e. it was underapplied.

C. The fixed overhead volume variance is difference between the amount of fixed overhead applied (standard rate * standard input allowed for the actual level of output) and the budgeted amount of fixed overhead. See the correct answer for a complete explanation.

D. The fixed overhead volume variance is difference between the amount of fixed overhead applied (standard rate * standard input allowed for the actual level of output) and the budgeted amount of fixed overhead. See the correct answer for a complete explanation.

Question 20 - CMA 693 3-12 - Performance Measurement

Edith Carolina, president of the Deed Corporation, requires a minimum return on investment of 8% for any project to be undertaken by her company. The company is decentralized, and leaves investment decisions up to the discretion of the division managers as long as the 8% return is expected to be realized. Michael Sanders, manager of the Cosmetics Division, has had a return on investment of 14% for his division for the past 3 years and expects the division to have the same return in the coming year. Sanders has the opportunity to invest in a new line of cosmetics which is expected to have a return on investment of 12%.

If the Deed Corporation evaluates managerial performance using residual income based on the corporate minimum required rate of return, what will be the preference for taking on the proposed cosmetics line by Edith Carolina and Michael Sanders?

A. CarolinaSanders AcceptRejectB. CarolinaSanders RejectAcceptC. CarolinaSanders RejectRejectD. CarolinaSanders AcceptAccept

A. Residual income measures the dollar return of an investment. Since the return on this investment is 12% and the required return is 8%, both of these projects will have a positive residual income and both Carolina and Sanders would accept the proposal.

B. Residual income measures the dollar return of an investment. Since the return on this investment is 12% and the required return is 8%, both of these projects will have a positive residual income and both Carolina and Sanders would accept the proposal.

C. Residual income measures the dollar return of an investment. Since the return on this investment is 12% and the required return is 8%, both of these projects will have a positive residual income and both Carolina and Sanders would accept the proposal.

D. Residual income measures the dollar return of an investment. Since the return on this investment is 12% and the required return is 8%, both of these projects will have a positive residual income and both Carolina and Sanders would accept the proposal.

Question 21 - CMA 1295 3-8 - Variances - Material and Labor

The efficiency variance for either labor or materials can be divided into

A. Spending variance and yield variance.

Part 2 : 11/11/10 07:41:20

(c) HOCK international, page 12

B. Yield variance and price variance.C. Volume variance and mix variance.D. Yield variance and mix variance.

A. The total material quantity (efficiency) and labor efficiency variances can be broken down into the two subvariances - the mix and the yield variances.

B. The total material quantity (efficiency) and labor efficiency variances can be broken down into the two subvariances - the mix and the yield variances.

C. The total material quantity (efficiency) and labor efficiency variances can be broken down into the two subvariances - the mix and the yield variances.

D. The total material quantity (efficiency) and labor efficiency variances can be broken down into the two subvariances - the mix and the yield variances. The mix variance is the part of the quantity variance that results because the mix of material actually used was different from the mix that was supposed to have been used. (For example, including more corn and less wheat in the cereal than is normal). The yield variance results from the difference between the total quantity of the inputs that were actually used to produce the actual output and the standard quantity that should have been used to produce the actual output.

Question 22 - CMA 695 3-25 - Variances - Material and Labor

Price variances and efficiency variances can be key to the performance measurement within a company. In evaluating the performance within a company, a materials efficiency variance can be caused by all of the following except the

A. Design of the product.B. Sales volume of the product.C. Actions of the purchasing department.D. Performance of the workers using the material.

A. The materials efficiency variance could be caused by a number of reasons: poor workers performance, spoilage, shrinkage, theft, design of the product, poor quality of materials, downtimes, etc. The design of the product can cause the materials efficiency variance.

B. The materials efficiency variance is simply the difference between the actual material usage and the standard usage for this level of output, multiplied by the standard price. It does not relate to the sales volume of the product.

C. The materials efficiency variance could be caused by a number of reasons: poor workers performance, spoilage, shrinkage, theft, design of the product, poor quality of materials, downtimes, etc. Actions of the purchasing department can cause the materials efficiency variance.

D. The materials efficiency variance could be caused by a number of reasons: poor workers performance, spoilage, shrinkage, theft, design of the product, poor quality of materials, downtimes, etc. The performance of the workers using the material can cause the materials efficiency variance.

Question 23 - CMA 692 3-14 - Responsibility Accounting

The most fundamental responsibility center affected by the use of market-based transfer prices is a(n)

A. Profit center.B. Investment center.

Part 2 : 11/11/10 07:41:20

(c) HOCK international, page 13

C. Cost center.D. Production center.

A. A profit center is responsible for revenues and costs (and therefore profit). The transfer price does affect a profit center.

B. While an investment center would be affected by the transfer price, it is not the most fundamental (basic) of the centers that is affected by the transfer price.

C. A cost center is not responsible for revenues so it would not be affected by the transfer price.

D. Production centers are not always responsible for revenues and costs so the transfer price that is used would not always affect a production center.

Question 24 - CMA 1296 3-2 - Responsibility Accounting

The segment margin of an investment center after deducting the imputed interest on the assets used by the investment center is known as

A. Return on assets.B. Operating income.C. Residual income.D. Return on investment.

A. The return on assets does not take into account the imputed interest rate on the assets of the investment center. See the correct answer for a complete explanation.

B. Operating income does not take into account the imputed interest rate on the assets of the investment center. See the correct answer for a complete explanation.

C. Residual income is the amount of return after a certain required return on the assets in use by the division. This describes the situation in the question.

D. Return on investment is calculated as the income divided by the invested amount. There is no deduction of an imputer interest rate in the calculation of return on investment.

Question 25 - CMA 697 3-30 - Performance Measurement

James Webb is the general manager of the Industrial Product Division, and his performance is measured using the residual income method. Webb is reviewing the following forecasted information for his division for next year:

Category Amount(thousands)

Working capital $1,800 Revenue 30,000 Plant and equipment 17,200

If the imputed interest charge is 15% and Webb wants to achieve a residual income target of $2,000,000, what will costs have to be in order to achieve the target?

A. $10,800,000

Part 2 : 11/11/10 07:41:20

(c) HOCK international, page 14

B. $25,690,000C. $9,000,000D. $25,150,000

A. See the correct answer for a complete explanation.

B. See the correct answer for a complete explanation.

C. See the correct answer for a complete explanation.

D. Residual income is equal to net income before taxes on project or investment opportunity minus target return in dollars (a % of assets or invested capital). Invested capital is equal to the sum of working capital and plant and equipment or $19,000,000 ($1,800,000 + $17,200,000). Hence, imputed interest charge is equal to $2,850,000 ($19,000,000 * 15%). Net income has to be equal to the sum of imputed interest charge and amount of residual income or $4,850,000 ($2,850,000 + $2,000,000). Thus, costs are equal to the revenue minus net income or $25,150,000 (30,000,000 - $4,850,000).

Question 26 - CIA 595 III-24 - Control Concepts and Principles

Which of the following management practices involves concentrating on areas that deserve attention and placing less attention on areas operating as expected?

A. Benchmarking.B. Management by objectives.C. Management by exception.D. Responsibility accounting.

A. Benchmarking is the process of a company using the standards set by other companies as a target or model for its own operations. (This is also called best practices.) It is the process of continuously trying to emulate (imitate) the best companies in the world.

B. The primary MBO objective is goal congruence; it is behavioral, communication-oriented, and a responsibility approach system.

C. Management by exception means that management focus on areas where there are problems, as identified by the fact that there is a variance from the standard. To manage by exception the standards and the system where variances are identified and reported to the appropriate level of the company have to be set.

D. Responsibility accounting is an accounting system that measures accounting results of each responsibility center separately; it also measures consolidated results.

Question 27 - CMA 1293 3-17 - Responsibility Accounting

The budgeting process that uses management by objectives and input from the individual manager is an example of the application of

A. Human resource management.B. Responsibility accounting.C. Capital budgeting.D. Flexible budgeting.

A. Human resource management is not what is described in the question.

Part 2 : 11/11/10 07:41:20

(c) HOCK international, page 15

B. Though this is not the best definition of responsibility accounting, this answer is a better choice than any of the other answers.

C. Capital budgeting is the long term budgeting for fixed assets. This is not what is described in the question.

D. Flexible budgeting is the creation of a budget for different levels of outputs. This is not what is described in the question.

Question 28 - CMA 1296 3-23 - Variances - General

The purpose of identifying manufacturing variances and assigning their responsibility to a person/department should be to

A. Determine the proper cost of the products produced so that selling prices can be adjusted accordingly.B. Pinpoint fault for operating problems in the organization.C. Use the knowledge about the variances to promote learning and continuous improvement in the manufacturing operations.D. Trace the variances to finished goods so that the inventory can be properly valued at year-end.

A. The selling price is mostly determined by the manufacturing costs and other aspects like market price level, service costs allocation, etc. The variances are usually not significant aspects in process of selling price determination.

B. To pinpoint fault for operating problems in the organization is the one of the reasons of responsibility accounting. However, it is not the major and ultimate purpose of variances and responsibility accounting.

C. The purpose of identifying manufacturing variances and assigning their responsibility to a person/department is to use the knowledge about the variances to promote learning and continuous improvement. One of the main purposes for responsibility centers and responsibility accounting are to enable evaluation of subunits' performance and contribute to measuring the performance of the subunits' managers. This provides motivation for managers of the subunits. By knowing what they are responsible for and controlling those items, managers should be more motivated than if they were evaluated on something outside of their control.

D. Depending on the significance of variance it is either written off to the finished goods account or allocated to the inventory accounts and finished goods on pro-rata basis. They are not valued at the end of the year.

Question 29 - CMA 1291 3-14 - Other

Folsom Fashions sells a line of women's dresses. Folsom's performance report for November follows.

Actual Budget Dresses sold 5,000 6,000 Sales $235,000 $300,000 Variable costs (145,000) (180,000) Contribution margin 90,000 120,000 Fixed costs (84,000) (80,000) Operating income $6,000 $40,000

The company uses a flexible budget to analyze its performance and to measure the effect on operating income of the various factors affecting the difference between budgeted and actual operating income.

Part 2 : 11/11/10 07:41:20

(c) HOCK international, page 16

The effect of the sales quantity variance on the contribution margin for November is

A. $15,000 unfavorable.B. $20,000 unfavorable.C. $30,000 unfavorable.D. $18,000 unfavorable.

A. This is the sales price variance. See the correct answer for a complete explanation.

B. This question is asking for the sales volume/quantity variance that is calculated as follows: (Actual Sales Volume - Budgeted Sales Volume) * Standard Contribution per Unit. The total budgeted contribution margin was $120,000, which gives us a $20 contribution margin per unit ($120,000 / 6,000). Now we can calculate the sales volume variance: (5,000 - 6,000) * $20 = ($20,000) unfavorable. The actual sales volume was lower than budgeted, and that caused the negative impact of $20,000 on the contribution margin.

C. This is the total sales variance, which includes variances caused by differences in both the sales price and in the quantity sold. The total sales variance is the difference between actual and budgeted amount of contribution margin. The sales volume/quantity variance is calculated as follows: (Actual Sales Volume - Budgeted Sales Volume) * Standard Contribution per Unit. See the correct answer for a complete explanation.

D. This variance was calculated by using the actual unit contribution margin, not the standard unit contribution margin. See the correct answer for a complete explanation.

Question 30 - CIA 595 III-96 - Responsibility Accounting

Which of the following techniques would be best for evaluating the management performance of a department that is operated as a cost center?

A. Return on assets ratio.B. Variance analysis.C. Payback method.D. Return on investment ratio.

A. A cost center is a type of center in responsibility accounting classifications that is responsible only for the incurrence of costs. A cost center does not have any revenue, and therefore does not have any profit. Thus, any type of return ratios cannot be calculated.

B. A cost center is a type of center in responsibility accounting classifications that is responsible only for the incurrence of costs. A cost center does not have any revenue, and therefore does not have any profit. Thus, performance evaluation based on variance analysis of costs is the best basis for performance evaluation of a cost center manager.

C. The payback method is usually used to analyze alternative investment opportunities, not management performance.

D. Cost center is a type of center in responsibility accounting classifications that is responsible only for the incurrence of costs. A cost center does not have any revenue, and therefore does not have any profit. Thus, any type of return ratios cannot be calculated.

Question 31 - CIA 594 III-73 - Variances - Material and Labor

A company manufactures one product and has a standard cost system. In April the company had the following

Part 2 : 11/11/10 07:41:20

(c) HOCK international, page 17

experience:

Direct Materials Direct Labor Actual $/unit of input (lbs. & hrs.) $28 $18 Standard price/unit of input $24 $20 Standard inputs allowed per unit of output 10 4 Actual units of input 190,000 78,000 Actual units of output 20,000 20,000

The direct materials efficiency variance for April is

A. $156,000 favorable.B. $240,000 favorable.C. $760,000 unfavorable.D. $240,000 unfavorable.

A. This is the direct labor rate variance. See the correct answer for a complete explanation.

B. The quantity variance (also called the efficiency or usage variance) is calculated as: (Standard Quantity for Actual Output - Actual Quantity) * Standard Price. The standard quantity allowed for the output of 20,000 units of product is 200,000 lb. (20,000 * 10lb. standard quantity of material per unit of finished product). The direct materials efficiency variance is $240,000 favorable [(200,000 - 190,000) * $24]. The variance is favorable because the standard quantity allowed for the actual output is greater than actual quantity of materials used in production.

C. This is the direct materials price variance. The direct materials efficiency variance is favorable because the standard quantity allowed for the actual output is greater than actual quantity of materials used in production. See the correct answer for a complete explanation.

D. The direct materials efficiency variance is favorable because the standard quantity allowed for the actual output is greater than actual quantity of materials used in production. See the correct answer for a complete explanation.

Question 32 - CMA 694 3-21 - Variances - Material and Labor

Under a standard cost system, the materials efficiency variances are the responsibility of

A. Sales and industrial engineering.B. Purchasing and industrial engineering.C. Production and industrial engineering.D. Purchasing and sales.

A. Industrial engineers are involved in designing product and setting the standards of material usage thus, bearing a part of responsibility of efficiency of material usage. However, sales personnel rarely can influence material usage by production workers.

B. Industrial engineers are involved in designing product and setting the standards of material usage thus, bearing a part of responsibility of efficiency of material usage. However, purchasing personnel rarely can influence material usage by production workers.

C. The quantity variance (also called the efficiency or usage variance) is calculated as: (Standard Quantity for Actual Output - Actual Quantity) * Standard Price. Industrial engineers are involved in designing product and setting the standards of material usage thus, bearing a part of responsibility of efficiency of material usage. Production workers are those who actually convert materials into the finished product and directly responsible for efficiency of material usage. Therefore, both of these parties will have some responsibility for

Part 2 : 11/11/10 07:41:20

(c) HOCK international, page 18

the materials efficiency variance.

D. Purchasing and sales personnel rarely can influence material usage by production workers.

Question 33 - CMA 1294 3-22 - Responsibility Accounting

If a manufacturing company uses responsibility accounting, which one of the following items is least likely to appear in a performance report for a manager of an assembly line?

A. Materials.B. Supervisory salaries.C. Equipment depreciation.D. Repairs and maintenance.

A. In accordance with responsibility accounting, a manager should be held responsible only for those factors which he or she can control. The assembly line manager will have control over the materials used and therefore they should be included in the performance report of the assembly line manager.

B. In accordance with responsibility accounting, a manager should be held responsible only for those factors which he or she can control. The assembly line manager will have input into determining the salaries of supervisors. Therefore, they are controllable by the manager, so they should be included in the assembly line manager's performance report.

C. In accordance with responsibility accounting, a manager should be held responsible only for those factors which he or she can control. Costs of fixed assets and depreciation metehods and policies are not under the control of an assembly line manager, because those things are decided at a higher level. Therefore, depreciaiton would not normally be included in a performance report for the assembly line manager under responsibility accounting.

D. In accordance with responsibility accounting, a manager should be held responsible only for those factors which he or she can control. The assembly line manager will have control over repairs and maintenance and therefore they should be included in the performance report of the assembly line manager.

Question 34 - CMA Sample Q3-11 - Variances - General

Garland Company uses a standard cost system. The standard for each finished unit of product allows for 3 pounds of plastic at $0.72 per pound. During December, Garland bought 4,500 pounds of plastic at $0.75 per pound, and used 4,100 pounds in the production of 1,300 finished units of product. What is the materials purchase price variance for the month of December?

A. $135 unfavorable.B. $117 unfavorable.C. $150 unfavorable.D. $123 unfavorable.

A. The price variance is calculated as follows: (Standard Price - Actual Price) * Actual Quantity. The purchase price variance is calculated using all of the units purchased, not just the units that are put into production. The purchase price variance is $135 unfavorable [($0.72 - $0.75) * 4,500]. The actual price exceeds the standard, thus, the variance is unfavorable.

B. The price variance is calculated as follows: (Standard Price - Actual Price) * Actual Quantity. The purchase price variance is calculated using all of the units purchased, not just the units that are put into production. See the correct answer for a complete explanation.

Part 2 : 11/11/10 07:41:20

(c) HOCK international, page 19

C. The price variance is calculated as follows: (Standard Price - Actual Price) * Actual Quantity. The purchase price variance is calculated using all of the units purchased, not just the units that are put into production. See the correct answer for a complete explanation.

D. The price variance is calculated as follows: (Standard Price - Actual Price) * Actual Quantity. The purchase price variance is calculated using all of the units purchased (4,500), not just the units that are put into production (4,100). See the correct answer for a complete explanation.

Question 35 - CMA 692 3-21 - Variances - Material and Labor

Jackson Industries employs a standard cost system in which direct materials inventory is carried at standard cost. Jackson has established the following standards for the prime costs of one unit of product.

StandardQuantity

StandardPrice

StandardCost

Direct materials 5 pounds $3.60/pound $18.00 Direct labor 1.25 hours 12.00/hour 15.00 $33.00

During May, Jackson purchased 125,000 pounds of direct materials at a total cost of $475,000. The total factory wages for May were $364,000, 90% of which were for direct labor. Jackson manufactured 22,000 units of product during May using 108,000 pounds of direct materials and 28,000 direct labor hours.

The direct labor usage (efficiency) variance for May is

A. $5,850 unfavorable.B. $6,000 favorable.C. $6,000 unfavorable.D. $5,850 favorable.

A. The labor efficiency variance is calculated as: (Standard Hours for Actual Output - Actual Quantity) * Standard Rate. In calculation we have to use standard rate ($12), not the actual rate ($11.70) as was used in this answer choice.

B. This answer is incorrect. See the correct answer for a complete explanation.

C. The labor efficiency variance is calculated as: (Standard Hours for Actual Output - Actual Quantity) * Standard Rate. The standard hours allowed for the actual level of output is 27,500 hours (1.25 hours per unit, 22,000 units produced). The labor efficiency variance is $6,000 unfavorable [(27,500 - 28,000) * $12]. The actual hours exceed the budgeted amount, which means the variance is unfavorable.

D. This answer is incorrect. See the correct answer for a complete explanation.

Question 36 - CIA 594 III-71 - Responsibility Accounting

Which of the following is not true of responsibility accounting?

A. The focus of cost center managers will normally be more narrow than that of profit center managers.B. Every factor that affects a firm's financial performance ultimately is controllable by someone, even if that someone is the person at the top of the firm.C. When a responsibility account system exists, operations of the business are organized into separate areas

Part 2 : 11/11/10 07:41:20

(c) HOCK international, page 20

controlled by individual managers.D. Managers should only be held accountable for factors over which they have significant influence.

A. A cost center has only costs under its control while a profit center has costs and revenues. Therefore, the manager of a cost center will be more narrow in their focus.

B. Unfortunately, not everything that impacts a firm's performance is controllable by someone in the firm. A good example of this type of exogenous variable is the economy. As the economy itself improves or weakens, the company will also be affected. But, noin the company is in a position to influence the economy.

C. In order for a responsibility system to work, the company needs to be organized into areas or divisions or some other breakdown based upon what a position is able to influence and control.

D. In responsibility accounting, managers should be held accountable only for the things that they are able to control.

Question 37 - CMA 694 3-19 - Variances - General

Which one of the following is least likely to be involved in establishing standard costs for evaluation purposes?

A. Industrial engineers.B. Top management.C. Quality control personnel.D. Budgetary accountants.

A. Industrial engineers are involved in establishing standard costs for evaluation purposes.

B. Top management is primarily involved in formulating strategy plans and budgets. Establishing standard costs for evaluation purposes is a task of management of a lower level. These standards are used to estimate what costs should be under normal conditions of operations. Industrial engineers, budgetary accountants, quality control personnel and employees who will be evaluated using these criteria are involved in the process.

C. Quality control personnel are involved in establishing standard costs for evaluation purposes.

D. Budgetary accountants are involved in establishing standard costs for evaluation purposes.

Question 38 - CMA 1291 3-2 - Variances - Material and Labor

Arrow Industries employs a standard cost system in which direct materials inventory is carried at standard cost. Arrow has established the following standards for the prime costs of one unit of product.

StandardQuantity

StandardPrice

StandardCost

Direct materials 8 pounds $1.80 per pound $14.40 Direct labor .25 hours 8.00 per hour 2.00 $16.40

During November, Arrow purchased 160,000 pounds of direct materials at a total cost of $304,000. The total factory wages for November were $42,000, 90% of which were for direct labor. Arrow manufactured 19,000 units of product during November using 142,500 pounds of direct materials and 5,000 direct labor hours.

Part 2 : 11/11/10 07:41:20

(c) HOCK international, page 21

The direct materials usage (quantity) variance for November is

A. $17,100 unfavorable.B. $14,400 unfavorable.C. $1,100 favorable.D. $17,100 favorable.

A. The material quantity variance is favorable. See the correct answer for a complete explanation.

B. The material quantity variance is favorable. See the correct answer for a complete explanation.

C. This answer is incorrect. See the correct answer for a complete explanation.

D. The material quantity variance (also called the efficiency or usage variance) is the difference between the actual material usage and the standard usage for this level of output, multiplied by the standard price; or (SQ - AQ) * SP. The standard quantity is 8 pounds per unit. The standard quantity to produce 19,000 units is 152,000 pounds. Actually 142,500 pounds of direct materials were used. The material quantity variance is $17,100 favorable (152,000 - 142,500)* $1.80.

Question 39 - CMA 1294 3-21 - Responsibility Accounting

Sherman Company uses a performance reporting system that reflects the company's decentralization of decision making. The departmental performance report shows one line of data for each subordinate who reports to the group vice president. The data presented show the actual costs incurred during the period, the budgeted costs, and all variances from budget for that subordinate's department. Sherman is using a type of system called

A. Flexible budgeting.B. Contribution accounting.C. Responsibility accounting.D. Cost-benefit accounting.

A. Flexible budgeting is the creation of a budget for different levels of outputs. This is not what is described in the question.

B. In contribution accounting variable costs are subtracted from revenues.

C. Responsibility accounting is a system in which costs are allocated to managers and/or departments based on who is responsible for the incurrence of the costs. This is the method described in the question.

D. This is not an accounting method.

Question 40 - CMA 691 3-29 - Performance Measurement

The selection of the denominator in the return on investment (ROI) formula is critical to the measure's effectiveness. Which denominator is criticized because it combines the effects of operating decisions made at one level of the organization with financing decisions made at another organizational level?

A. Total assets employed.B. Total assets available.C. Shareholders' equity.D. Working capital.

Part 2 : 11/11/10 07:41:20

(c) HOCK international, page 22

A. This does not take into account the source of financing since it all assets that are employed, no matter the source of the financing.

B. This does not take into account the source of financing since it all assets that are available, no matter the source of financing.

C. Shareholders' equity is not a good denominator to use in the ROI calculation because it takes into account decisions made about financing that are not made at the manager level. The decision to use equity for the financing of assets is one that is made at the highest levels in the organization.

D. This does not take into the source of financing of assets as it will include those assets financed by debt and equity.

Question 41 - CMA 1292 3-19 - Overhead

Nanjones Company manufactures a line of products distributed nationally through wholesalers. Presented below are planned manufacturing data for the year and actual data for November of the current year. The company applies overhead based on planned machine hours using a predetermined annual rate.

Planning Data Annual November Fixed manufacturing overhead $1,200,000 $100,000 Variable manufacturing overhead 2,400,000 220,000 Direct labor hours 48,000 4,000 Machine hours 240,000 22,000

Data for November Direct labor hours (actual) 4,200 Direct labor hours (plan based on output) 4,000 Machine hours (actual) 21,600 Machine hours (plan based on output) 21,000 Fixed manufacturing overhead $101,200 Variable manufacturing overhead $214,000

The fixed overhead volume variance for November was

A. $1,200 unfavorable.B. $5,000 favorable.C. $10,000 favorable.D. $5,000 unfavorable.

A. This answer is incorrect. See the correct answer for a complete explanation.

B. The fixed overhead volume variance is difference between the amount of fixed overhead applied (standard rate * standard input for the actual level of output) and the budgeted amount of fixed overhead. Nanjones applies overhead based on planned machine hours using a predetermined annual rate. The amount of the planned fixed manufacturing overhead was $1,200,000 and amount of planned machine hours were 240,000. Thus, standard application rate for fixed manufacturing overhead was $5 ($1,200,000 / 240,000). The number of planned machine hours for the actual level of output in November was 21,000. Now we can calculate the amount of applied fixed manufacturing overheads as $105,000 ($5 * 21,000). The budgeted amount of fixed manufacturing overhead is static budget number planned for November was $100,000. Therefore, the fixed overhead volume variance is: $105,000 - $100,000 = $5,000.

C. The fixed overhead volume variance is difference between the amount of fixed overhead applied (standard rate * standard input for the actual level of output) and the budgeted amount of fixed overhead. If in the calculation of fixed manufacturing overhead applied the static budget of machine hours (22,000) were used we would come up with

Part 2 : 11/11/10 07:41:20

(c) HOCK international, page 23

amount of $110,000 FMOH applied. And the fixed overhead volume variance would be $10,000 ($110,000 - $100,000) favorable; which is incorrect. In the calculation of fixed overhead applied, we should use number of standard machine hours for the actual level of output (21,000 hours). See the correct answer for a complete explanation.

D. This answer is incorrect. See the correct answer for a complete explanation.

Question 42 - CMA 695 3-24 - Variances - Material and Labor

Blaster Inc., a manufacturer of portable radios, purchases the components from subcontractors to use to assemble into a complete radio. Each radio requires three units each of Part XBEZ52, which has a standard cost of $1.45 per unit. During May, Blaster experienced the following with respect to Part XBEZ52.

Units Purchases ($18,000) 12,000 Consumed in manufacturing 10,000 Radios manufactured 3,000

During May, Blaster Inc. incurred a materials efficiency variance of

A. $1,450 unfavorable.B. $4,350 favorable.C. $1,450 favorable.D. $4,350 unfavorable.

A. The efficiency variance is calculated as (SQ - AQ) * SP. The standard quantity is 9,000 (3 units of part XBEZ52 to produce one radio and 3,000 radios were manufactured). The actual quantity is 10,000 units and the standard price is $1.45 per part XBEZ52. The efficiency variance is $1,450 unfavorable [(9,000 - 10,000)* $1.45].

B. The efficiency variance is $1,450 unfavorable because budget quantity of part XBEZ52 was less then consumed in manufacturing. Also the quantity consumed in manufacturing (10,000), not the number of purchased units (12,000) should be used in calculation.

C. The efficiency variance is $1,450 unfavorable because the budgeted quantity of part XBEZ52 was less then was actually consumed in manufacturing.

D. The efficiency variance is calculated as (SQ - AQ) * SP. In this formula as the actual quantity we have to use the quantity consumed in manufacturing (10,000), not the number of purchased units (12,000).

Question 43 - CMA 1290 3-7 - Overhead

Franklin Glass Works' production budget for the year ended November 30 was based on 200,000 units. Each unit requires two standard hours of labor for completion. Total overhead was budgeted at $900,000 for the year, and the fixed overhead rate was estimated to be $3.00 per unit. Both fixed and variable overhead are assigned to the product on the basis of direct labor hours. The actual data for the year ended November 30 are presented as follows.

Actual production in units 198,000 Actual direct labor hours 440,000 Actual variable overhead $352,000 Actual fixed overhead $575,000

Part 2 : 11/11/10 07:41:20

(c) HOCK international, page 24

Franklin's variable overhead spending variance for the year is

A. $22,000 unfavorable.B. $19,800 favorable.C. $20,000 favorable.D. $20,000 unfavorable.

A.

The variable overhead spending variance is the difference between the standard application rate and the actual application rate (the actual application rate is calculated as the actual overhead costs ÷ the actual usage of the allocation base) multiplied by the actual quantity of the application base. (Standard Application Rate - Actual Application Rate) * Actual Quantity.

The fixed overhead rate was estimated to be $3.00 per unit, thus the budgeted amount of fixed overhead for budgeted level of production of 200,000 units was $600,000 ($3 * 200,000). Thus, the budgeted amount of variable overhead was $300,000 ($900,000 - $600,000). Each unit requires two standard hours of labor, hence the labor rate per unit was $0.75 ($300,000 ÷ (200,000 * 2)). The actual variable overhead was $352,000. The actual production hours were 440,000. The standard application rate multiplied by the actual quantity is $330,000 ($0.75 * 440,000). The variable overhead spending variance is $22,000 unfavorable ($330,000 - $352,000).

B. This answer is incorrect. See the correct answer for a complete explanation.

C. This answer is incorrect. See the correct answer for a complete explanation.

D. This answer is incorrect. See the correct answer for a complete explanation.

Question 44 - CMA 1291 3-16 - Variances - General

Folsom Fashions sells a line of women's dresses. Folsom's performance report for November follows.

Actual Budget Dresses sold 5,000 6,000 Sales $235,000 $300,000 Variable costs (145,000) (180,000) Contribution margin 90,000 120,000 Fixed costs (84,000) (80,000) Operating income $6,000 $40,000

The company uses a flexible budget to analyze its performance and to measure the effect on operating income of the various factors affecting the difference between budgeted and actual operating income.

The variable cost flexible budget variance for November is

A. $4,000 unfavorable.B. $5,000 unfavorable.C. $5,000 favorable.D. $4,000 favorable.

A. This is the fixed cost variance. However, the question asks for the variable cost variance.

B.

Part 2 : 11/11/10 07:41:20

(c) HOCK international, page 25

The actual cost incurred was lower than the budgeted amount for the actual activity, so the variance is favorable.

C.

The flexible budget variance for variable costs is the difference between the actual variable cost incurred and the budgeted amount for the actual activity. The budgeted variable cost per unit sold is $180,000 in budgeted total variable costs ÷ 6,000 budgeted units sold, or $30 per unit. Since 5,000 dresses were sold, the budgeted amount for the actual activity is $30 * 5,000, or $150,000.

The actual total variable cost incurred was $145,000. Therefore, the flexible budget variance for variable costs is the difference between $150,000 and $145,000, or $5,000. The variance is favorable because the actual cost incurred ($145,000) was lower than the budgeted amount for the actual activity ($150,000).

D. This is the difference between the actual and budgeted fixed cost. However, the question asks for the variable cost flexible budget variance.

Question 45 - CIA 592 IV-18 - Variances - General

The following is a standard cost variance analysis report on direct labor cost for a division of a manufacturing company.

Job Actual Hours atActual Wages

Actual HoursStandard Wages

Standard HoursStandard Wages

213 $3,243 $3,700 $3,100 215 15,345 15,675 15,000 217 6,754 7,000 6,600 219 19,788 18,755 19,250 221 3,370 3,470 2,650 Totals $48,500 $48,600 $46,600

What is the total flexible budget direct labor variance for the division?

A. $100 favorable.B. $2,000 unfavorable.C. $1,900 favorable.D. $1,900 unfavorable.

A. This answer is incorrect. See the correct answer for a complete explanation.

B. This answer is incorrect. See the correct answer for a complete explanation.

C. This answer is incorrect. See the correct answer for a complete explanation.

D. The total labor variance (also called the flexible budget variance) is the difference between the standard costs for the actual level of output (the flexible budget) and the actual costs incurred by the company. In the third column there are standard costs for the actual level of output presented and in first column there are actual costs incurred. Thus, the total labor variance is $46,600 - $48,500 = ($1,900) unfavorable.

Question 46 - CMA 693 3-16 - Variances - General

In analyzing company operations, the controller of the Jason Corporation found a $250,000 favorable flexible-budget

Part 2 : 11/11/10 07:41:20

(c) HOCK international, page 26

revenue variance. The variance was calculated by comparing the actual results with the flexible budget. This variance can be wholly explained by

A. The total static budget variance.B. Changes in unit selling prices.C. The total flexible budget variance.D. The total sales volume variance.

A. This answer is incorrect. See the correct answer for a complete explanation.

B. The simple formula to calculate revenue is Revenue = Quantity * Price. Changes in revenue come from either change in quantity of product sold or change in selling price. Since we use the flexible budget in this analysis there will be no difference between the numbers of actual sales volume and budgeted sales volume. The difference in the flexible budget revenue and actual revenue can be caused only by a difference between the actual and budgeted selling price.

C. This answer is incorrect. See the correct answer for a complete explanation.

D. The sales volume variance measures the impact of difference in sales volume and is calculated as follows: (Actual Sales Volume - Budgeted Sales Volume) * Standard Contribution per Unit. Since we use flexible budget in the analysis there will be no difference between numbers of actual sales volume and budgeted sales volume. The total sales volume variance is zero.

Question 47 - CMA 694 3-18 - Performance Measurement

The imputed interest rate used in the residual income approach to performance evaluation can best be described as the

A. Historical weighted-average cost of capital for the company.B. Target return on investment set by the company's management.C. Average return on investments for the company over the last several years.D. Average lending rate for the year being evaluated.

A. This answer is incorrect. See the correct answer for a complete explanation.

B. In residual income the imputer rate of interest that is used can best be described as the target return on investment that has been set by management.

C. This answer is incorrect. See the correct answer for a complete explanation.

D. This answer is incorrect. See the correct answer for a complete explanation.

Question 48 - CMA 687 4-16 - Other

The following information is available for the Mitchelville Products Company for the month of July.

MasterBudget Actual

Units 4,000 3,800 Sales revenue $60,000 $53,200 Variable manufacturing costs 16,000 19,000

Part 2 : 11/11/10 07:41:20

(c) HOCK international, page 27

Fixed manufacturing costs 15,000 16,000 Variable selling and administrative expense 8,000 7,600 Fixed selling and administrative expense 9,000 10,000

The contribution margin volume variance for the month of July would be

A. $200 favorable.B. $400 unfavorable.C. $1,800 unfavorable.D. $6,800 unfavorable.

A. This is the difference between budgeted and actual units. See the correct answer for a complete explanation.

B. This is the difference between the actual and budgeted variable S&A costs. This number does not mean anything.

C. The contribution margin volume variance is calculated as follows: (Standard Quantity - Actual Quantity) * Contribution Margin. The budgeted contribution margin per unit is $9 [($60,000 sales revenue - $16,000 variable manufacturing costs - $8,000 variable S&A costs) / 4,000 units].

D. This is the difference between actual and budgeted revenue. See the correct answer for a complete explanation.

Question 49 - CMA 696 3-27 - Transfer Pricing

Parkside Inc. has several divisions that operate as decentralized profit centers. Parkside's Entertainment Division manufactures video arcade equipment using the products of two of Parkside's other divisions. The Plastics Division manufactures plastic components, one type that is made exclusively for the Entertainment Division, while other less complex components are sold to outside markets. The products of the Video Cards Division are sold in a competitive market; however, one video card model is also used by the Entertainment Division. The actual costs per unit used by the Entertainment Division are presented below.

PlasticComponents

VideoCards

Direct material $1.25 $2.40 Direct labor 2.35 3.00 Variable overhead 1.00 1.50 Fixed overhead .40 2.25 Total cost $5.00 $9.15

The Plastics Division sells its commercial products at full cost plus a 25% markup and believes the proprietary plastic component made for the Entertainment Division would sell for $6.25 per unit on the open market. The market price of the video card used by the Entertainment Division is $10.98 per unit.

Assume that the Entertainment Division is able to purchase a large quantity of video cards from an outside source at $8.70 per unit. The Video Cards Division, having excess capacity, agrees to lower its transfer price to $8.70 per unit. This action would

A. Optimize the profit goals of the Entertainment Division while subverting the profit goals of Parkside Inc.B. Subvert the profit goals of the Video Cards Division while optimizing the profit goals of the Entertainment Division.C. Optimize the overall profit goals of Parkside Inc.D. Allow evaluation of both divisions on the same basis.

A. The Entertainment Division is indifferent as to whom to by video cards from as the external price is the same as the internal price. The profit goals of Parkside Inc. would not be subverted because this action would be congruent with overall profit goals of the company. Variable cost of production of Video Cards division are less than the suggested

Part 2 : 11/11/10 07:41:20

(c) HOCK international, page 28

discounted price, thus, the use of idle capacity enhances profits.

B. The Entertainment Division is indifferent as to whom to by the video cards from as the external price is the same as the internal price. The profit goals of the Video Cards Division are, however, subverted.

C. This action is goal congruent. The Entertainment Division is indifferent to whom to by video cards from as the external price is the same as the internal price. The profit goals of Parkside Inc. would not be subverted because this action would be congruent with overall profit goals of the company. Variable cost of production of Video Cards division are less than the suggested discounted price, thus, use of idle capacity enhances profits. Thus, it would optimize the overall profit goals of the company.

D. The selling price of Video Cards Division is less then the cost of production, and the price the Entertainment Division buys video cards is a market price. It wouldn't allow the evaluation of both divisions on the same basis.

Question 50 - CMA 696 3-26 - Transfer Pricing

Parkside Inc. has several divisions that operate as decentralized profit centers. Parkside's Entertainment Division manufactures video arcade equipment using the products of two of Parkside's other divisions. The Plastics Division manufactures plastic components, one type that is made exclusively for the Entertainment Division, while other less complex components are sold to outside markets. The products of the Video Cards Division are sold in a competitive market; however, one video card model is also used by the Entertainment Division. The actual costs per unit used by the Entertainment Division are presented below.

PlasticComponents

VideoCards

Direct material $1.25 $2.40 Direct labor 2.35 3.00 Variable overhead 1.00 1.50 Fixed overhead .40 2.25 Total cost $5.00 $9.15

The Plastics Division sells its commercial products at full cost plus a 25% markup and believes the proprietary plastic component made for the Entertainment Division would sell for $6.25 per unit on the open market. The market price of the video card used by the Entertainment Division is $10.98 per unit.

A per-unit transfer price from the Video Cards Division to the Entertainment Division at full cost, $9.15, would

A. Satisfy the Video Cards Division's profit desire by allowing recovery of opportunity costs.B. Allow evaluation of both divisions on a competitive basis.C. Provide no profit incentive for the Video Cards Division to control or reduce costs.D. Encourage the Entertainment Division to purchase video cards from an outside source.

A. This transfer price does not satisfy the Transistor Division's profit desire by allowing the recovery of opportunity costs because the transfer price is a full cost price.

B. This transfer price does not allow for the evaluation of both divisions on a competitive basis. The Entertainment Division 'buys' video cards from Video Cards Division at 'full cost' transfer price. However, it sells its product at market price.

C. This transfer price does not provide profit incentive to the Video Cards Division as it only covers the costs of production and does not allow a profit. It also does not encourage the Video Cards Division to reduce costs as full cost transfer price is accepted by Entertainment Division.

D. A transfer price of full cost is beneficial to the buying department as it is less than market price.

Part 2 : 11/11/10 07:41:20

(c) HOCK international, page 29

Question 51 - CIA 1191 IV-15 - Variances - Material and Labor

The total budgeted direct labor cost of a company for the month was set at $75,000 when 5,000 units were planned to be produced. The following standard cost, stated in terms of direct labor hours (DLH), was used to develop the budget for direct labor cost:

1.25 DLH x $12.00/DLH = $15.00/unit produced

The actual operating results for the month were as follows:

Actual units produced 5,200 Actual direct labor hours worked 6,600 Actual direct labor cost $77,220

The direct labor efficiency variance for the month would be

A. $4,200 unfavorable.B. $2,220 unfavorable.C. $3,000 unfavorable.D. $1,200 unfavorable.

A. The direct labor efficiency variance is calculated as follows: (Standard Hours for Actual Output - Actual Hours) * Standard Rate. This incorrect answer of $4,200 unfavorable occurs if we use hours scheduled for initially planned output of 5,000 units or master budget figure (1.25 DLH * 5,000 = 6,250 DLH) instead of standard hours allowed for actual output. In this case we have to use the flexible budget figure for standard direct labor hours allowed for the actual output of 5,200 units (1.25 DLH * 5,200 = 6,500 DLH). See the correct answer for a complete explanation.

B. This is the difference between actual costs and budgeted costs (master budget). This is not the direct labor efficiency variance. See the correct answer for a complete explanation.

C. The direct labor efficiency variance is calculated as follows: (Standard Hours for Actual Output - Actual Hours) * Standard Rate. This incorrect answer of $3,000 unfavorable occurs if we use standard hours for the initially budgeted level of output of 5,000 units or master budget figure (1.25 DLH * 5,200 = 6,500 DLH) instead of actual hours. In this case we need to use actual hours (6,600). See the correct answer for a complete explanation.

D. The direct labor efficiency variance is calculated as follows: (Standard Hours for Actual Output - Actual Hours) * Standard Rate. The standard hours for actual output were 6,500 hours (1.25 DLH * 5,200 actual units produced). Thus, the direct labor efficiency variance is $1,200 unfavorable ((6,500 - 6,600)* $12.00/DLH(. That the planned for the level of output hours were less than the actual hours gave us the unfavorable variance.

Question 52 - CMA 693 3-14 - Responsibility Accounting

The least complex segment or area of responsibility for which costs are allocated is a(n)

A. Investment center.B. Contribution center.C. Cost center.D. Profit center.

A. This answer is incorrect. See the correct answer for a complete explanation.

Part 2 : 11/11/10 07:41:20

(c) HOCK international, page 30

B. This answer is incorrect. See the correct answer for a complete explanation.

C. The cost center is the least complex of the different types of centers. It is responsible only for the incurrence of costs.

D. This answer is incorrect. See the correct answer for a complete explanation.

Question 53 - CIA 582 IV-22 - Variances - Material and Labor

Which of the following is least likely to cause an unfavorable materials quantity (usage) variance?

A. Scheduling of substantial overtime.B. Machinery that has not been maintained properly.C. Materials that do not meet specifications.D. Labor that possesses skills equal to those required by the standards.

A. An unfavorable materials quantity (usage) variance means that more materials were consumed by production than was scheduled by the standard. This can happen due to the number of reasons: poor workers performance, spoilage, shrinkage, theft, design of the product, poor quality of materials, machine downtimes, etc. Substantial overtimes may affect the performance of workers and be the cause of material efficiency variance.

B. An unfavorable materials quantity (usage) variance means that more materials were consumed by production than was scheduled by the standard. This can happen due to the number of reasons: poor workers performance, spoilage, shrinkage, theft, design of the product, poor quality of materials, machine downtimes, etc. Thus, machinery that has not been maintained properly may be the cause of material efficiency variance.

C. An unfavorable materials quantity (usage) variance means that more materials were consumed by production than was scheduled by the standard. This can happen due to the number of reasons: poor workers performance, spoilage, shrinkage, theft, design of the product, poor quality of materials, machine downtimes, etc. Thus, materials that do not meet specifications may be the cause of material efficiency variance.

D. An unfavorable materials quantity (usage) variance means that more materials were consumed by production than was scheduled by the standard. This can happen due to the number of reasons: poor workers performance, spoilage, shrinkage, theft, design of the product, poor quality of materials, machine downtimes, etc. Since the labor has skills equal to those required by the standards it is most likely not a cause of material efficiency variance.

Question 54 - CIA 596 III-90 - Variances - General

A manufacturing cell's partial productivity can be measured using data on

A. Scrap.B. Inventory shrinkage.C. Inventory turnover.D. Direct materials usage.

A. Scrap can't be used as a measure of partial productivity as it is not a finished product and not an input.

B. Shrinkage occurs when a product simply evaporates or loses some of its quantity through time. It does not reflect on manufacturing productivity.

C. Inventory turnover how long inventory is on hand before it is used. This is a measure not only of production but also

Part 2 : 11/11/10 07:41:20

(c) HOCK international, page 31

purchasing.

D. A partial productivity measure is usually a ratio of output to the quantity of a single factor (material, labor, capital). Examples of partial productivity measures are number of units produced per pound of direct material, or per direct labor hour. Direct material usage can be the measure of manufacturing cell's partial productivity because direct material is the input.

Question 55 - CIA 1190 IV-20 - Transfer Pricing

A limitation of transfer prices based on actual cost is that they

A. Must be adjusted by some markup.B. Charge inefficiencies to the department that is transferring the goods.C. Lack clarity and administrative convenience.D. Can lead to suboptimal decisions for the company as a whole.

A. By definition, in cost-based transfer pricing, costs do not need to be adjusted by any markup.

B. Inefficiencies are charged to the department that is receiving, not transferring, the goods.

C. Cost-based transfer price provide clarity and administrative convenience.

D. The basic issue of transfer prices is how much should one unit of a company charge another unit of the same company for its goods or services. The goal in setting a transfer price is that the method used will stimulate the department managers to do what will provide the greatest benefit to the company as a whole, rather than to act in their own interest. Transfer price based on actual cost does not motivate managers to use resources more efficiently which can lead to suboptimal decisions for the company as a whole.

Question 56 - CMA 694 3-28 - Responsibility Accounting

DigitalTech uses an accounting system that charges costs to the manager who has the authority to make decisions incurring the costs. For example, if a sales manager authorizes a rush order that results in additional manufacturing costs, these additional costs are charged to the sales manager. This type of accounting system is known as

A. Contribution accounting.B. Responsibility accounting.C. Transfer-pricing accounting.D. Functional accounting.

A. Contribution accounting focuses on the variable costs and their behavior as production levels change. The selling price minus variable costs is the contribution margin. Fixed costs are then subtracted from the contribution to determine the profit.

B. Responsibility accounting is a system in which an individual is held accountable (responsible) for things that they are able to control. It is this person's responsibility to address any issues that arise related to these costs that they control.

C. Transfer prices are the prices used in the 'sale' of goods within the company. See the correct answer for a complete explanation.

D. Functional accounting assigns costs based on the function that is performed. See the correct answer for a complete explanation.

Part 2 : 11/11/10 07:41:20

(c) HOCK international, page 32

Question 57 - CIA 1192 IV-20 - Variances - General

A manufacturer has the following direct materials standard for one of its products.

Direct materials: 3 pounds @ $1.60/pound = $4.80

The company records all inventory at standard cost. Data for the current period regarding the manufacturer's budgeted and actual production for the product as well as direct materials purchases and issues to production for manufacture of the product are presented as follows.

Budgeted production for the period: 8,000 unitsActual production for the period: 7,500 units

Direct materials purchases:Pounds purchased: 25,000 poundsTotal cost: $38,750

Direct materials issued in production: 23,000 pounds

The direct materials purchase price variance for the current period is

A. $1,250 favorable.B. $1,150 favorable.C. $1,125 favorable.D. $1,200 favorable.

A. The purchase price variance is calculated as follows: (SP - AP) * AQ, where AQ is the actual quantity purchased. The actual price for the pound of material is $1.55 ($38,750 / 25,000). The direct materials purchase price variance is $ $1,250 ($1.60 - $1.55) * 25,000. The actual price the materials were purchased for is less than standard price that's why the variance is favorable.

B. This is the material price variance, not purchase price variance. However, to calculate purchase price variance we need to use purchased quantity of material (25,000), not consumed by production (23,000).

C. This result is calculated using the quantity of materials allowed for the production 22,500 pounds (3 lb. Per unit of finished product and 7,500 units produced). However, to calculate purchase price variance we need to use purchased quantity of material (25,000). See the correct answer for a complete explanation.

D. This result is calculated using the budgeted quantity of materials 24,000 pounds (3 lb. per unit of finished product and 8,000 units budgeted for the production). However, to calculate purchase price variance we need to use purchased quantity of material (25,000). See the correct answer for a complete explanation.

Question 58 - CMA 694 3-23 - Variances - Material and Labor

A favorable materials price variance coupled with an unfavorable materials usage variance would most likely result from

A. Product mix production changes.B. The purchase and use of higher than standard quality materials.C. Machine efficiency problems.D. The purchase of lower than standard quality materials.

Part 2 : 11/11/10 07:41:20

(c) HOCK international, page 33

A. Product mix production changes would not cause the materials price variance.

B. The purchase and use of higher than standard quality materials would most likely result in an unfavorable materials price variance and either not significantly affect the material usage variance or would lead to favorable material efficiency variance.

C. Machine efficiency problems would not cause the materials price variance.

D. The favorable materials price variance means that the material was purchased for a lower price than planned. This could happen due to the discount of quantity, quality or some other reason. The unfavorable materials usage variance means that the production process used more material than had been planned. This could happen due to the lower level skilled workers, poor material quality or some other reason. However, these two variances together are most likely caused by the same reason. This reason could be the purchase of lower than standard quality materials.

Question 59 - CIA 1185 IV-12 - Other

Actual and budgeted information about the sales of a product are presented for June as follows.

Actual Budget Units 8,000 10,000 Sales Revenue $92,000 $105,000

The sales price variance for June was

A. $10,000 favorable.B. $8,000 favorable.C. $10,000 unfavorable.D. $10,500 unfavorable.

A. This answer is the result of using budgeted quantity of units sold (10,000), but we have to use actual units sold (8,000) in calculation. See the correct answer for a complete explanation.

B. The sales price variance measures the impact of the difference in contribution margin per unit and is calculated as follows: (Actual Contribution per unit - Standard Contribution per unit) * Actual Quantity. In this question we cannot determine the sales contribution per unit and will use the sales unit price in the calculation. The actual unit sales price is $11.50 ($92,000 ? 8,000). The standard unit sales price is $10.50 ($105,000 ? 10,000). The sales price variance is $8,000 favorable ($11.50 - $10.50) * 8,000. The actual sales price was higher than budget, which means that the variance is favorable.

C. The actual sales price ($11.50) was higher than budget ($10.50), which means that variance is favorable. See the correct answer for a complete explanation.

D. The actual sales price ($11.50) was higher than budget ($10.50), which means that variance is favorable. See the correct answer for a complete explanation.

Question 60 - CMA 694 3-24 - Responsibility Accounting

Decentralized firms can delegate authority and yet retain control and monitor managers' performance by structuring the organization into responsibility centers. Which one of the following organizational segments is most like an independent business?

Part 2 : 11/11/10 07:41:20

(c) HOCK international, page 34

A. Profit center.B. Revenue center.C. Cost center.D. Investment center.

A. A profit center comes close to an independent business, but a profit center is not responsible for a return on the investment made into it.

B. A revenue center is not responsible for costs, and in that respect does not resemble an independent business.

C. A cost center is responsible only for costs and not revenues. Therefore, it is not like an independent business, which also has a revenue responsibility.

D. An investment center is the most like an independent business because it is measured on its return on investment as well as on the level of profit.

Question 61 - CIA 1192 IV-21 - Variances - Material and Labor

A manufacturer has the following direct materials standard for one of its products.

Direct materials: 3 pounds @ $1.60/pound = $4.80

The company records all inventory at standard cost. Data for the current period regarding the manufacturer's budgeted and actual production for the product as well as direct materials purchases and issues to production for manufacture of the product are presented as follows.

Budgeted production for the period: 8,000 unitsActual production for the period: 7,500 units

Direct materials purchases:Pounds purchased: 25,000 poundsTotal cost: $38,750

Direct materials issued in production: 23,000 pounds

The materials efficiency variance for the current period is

A. $1,600 favorable.B. $775 unfavorable.C. $3,200 favorable.D. $800 unfavorable.

A. The quantity variance (also called the efficiency or usage variance) is calculated as: (SQ - AQ) * SP or (Standard Quantity for Actual Output - Actual Quantity) * Standard Price. As the formula shows, we need to use flexible budget standard quantity allowed for the actual level of output (22,500), not simply standard quantity of master budget figure (24,000). See the correct answer for a complete explanation.

B. The quantity variance (also called the efficiency or usage variance) is calculated as: (SQ - AQ) * SP. We have to use the standard price ($1.60), not the actual price ($1.55).

C. This is the difference between the actual quantity purchased (25,000) and actual quantity of materials used in production times the standard price ($1.60), which does not mean anything. See the correct answer for a complete explanation.

Part 2 : 11/11/10 07:41:20

(c) HOCK international, page 35

D. The quantity variance (also called the efficiency or usage variance) is calculated as: (SQ - AQ) * SP or (Standard Quantity for Actual Output - Actual Quantity) * Standard Price. The standard quantity of material scheduled for actual output was 22,500 lb. (7,500 units of finished product and 3 lb. of material needed to complete one unit of product). The actual quantity was 23,00 lb. The quantity variance is $800 unfavorable (22,500 - 23,000) * $1.60. The quantity of material allowed for production was less than actual amount, which gives us unfavorable variance.

Question 62 - CMA 694 3-20 - Variances - Material and Labor

Under a standard cost system, the materials price variances are usually the responsibility of the

A. Sales manager.B. Purchasing manager.C. Production manager.D. Cost accounting manager.

A. The material price variance is the difference between standard price and actual price of material multiplied by the quantity actually consumed by production. The sales manager is not involved in the purchasing activity where the price is negotiated.

B. The material price variance is the difference between the standard price and actual price of material times quantity actually consumed by production. The purchasing manager is responsible for purchasing activity where price is negotiated and is therefore usually held responsible for the material price variance.

C. The material price variance is the difference between standard price and actual price of material multiplied by the quantity actually consumed by production. The production manager is not involved in the purchasing activity where the price is negotiated.

D. The material price variance is the difference between standard price and actual price of material multiplied by the quantity actually consumed by production. The cost accounting manager is not involved in the purchasing activity where the price is negotiated.

Question 63 - CMA 696 3-22 - Variances - Material and Labor

Ardmore Enterprises uses a standard cost system in its small appliance division. The standard cost of manufacturing one unit of Zeb is as follows:

Materials - 60 pounds at $1.50 per pound $90 Labor - 3 hours at $12 per hour 36 Factory overhead - 3 hours at $8 per hour 24 Total standard cost per unit $150

The budgeted variable factory overhead rate is $3 per labor hour, and the budgeted fixed factory overhead is $27,000 per month. During May, Ardmore produced 1,650 units of Zeb compared with a normal capacity of 1,800 units. The actual cost per unit was as follows:

Materials (purchased and used) - 58 pounds at $1.65 per pound $95.70 Labor - 3.1 hours at $12 per hour 37.20 Factory overhead - $39,930 per 1,650 units 24.20 Total actual cost per unit $157.10

Part 2 : 11/11/10 07:41:20

(c) HOCK international, page 36

The total materials quantity variance for May is

A. $14,355 favorable.B. $4,950 favorable.C. $14,355 unfavorable.D. $4,950 unfavorable.

A. The quantity of material used (95,700) was less than budgeted quantity (99,000). This means that variance is favorable. See the correct answer for a complete explanation.

B. The quantity variance (also called the efficiency or usage variance) is calculated as: (Standard Quantity for Actual Output - Actual Quantity) * Standard Price. The standard quantity to produce 1,650 units equals 99,000 units (1,650 * 60 lb.). The actual quantity of material used was 58 lb. per unit of finished product or 95,700 units in total (1,650 * 58 lb.). The quantity variance is $4,950 favorable (99,000 - 95700) * $1.50. The quantity used was less than budgeted which means that variance is favorable.

C. This is the material price variance. See the correct answer for a complete explanation.

D. The quantity variance is $4,950 favorable (99,000 - 95700) * $1.50. The quantity of material used (95,700) was less than budgeted quantity (99,000) and this means that variance is favorable. See the correct answer for a complete explanation.

Question 64 - CMA 1295 3-25 - Variances - General

Which one of the following variances is most controllable by the production control supervisor?

A. Material price variance.B. Fixed overhead budget variance.C. Variable overhead spending variance.D. Material usage variance.

A. The purchasing department is the most responsible for the material price variance.

B. The fixed overhead budget variance cannot be controlled by production control supervisor.

C. The variable overhead spending variance is related to the difference between the standard application rate and the actual application rate (this is calculated as the actual overhead costs / the actual usage of the allocation base). The variable overhead spending variance is calculated as follows: (Standard Application Rate - Actual Application Rate) * Actual Quantity. Both the price (application rate) and the quantity (level of activity) is involved in the calculation. However, the production control supervisor does not control the prices (rates).

D. The material usage variance is the difference between the actual material usage and the standard usage for this level of output, multiplied by the standard material price. This variance occurs during the production process and is therefore most controllable by the production control supervisor. There are a number of reasons that could cause this variance: poor production employees' performance, product design, waste, theft, and poor material quality, etc.

Question 65 - CMA 694 3-30 - Transfer Pricing

An appropriate transfer price between two divisions of The Stark Company can be determined from the following data:

Fabricating Division

Part 2 : 11/11/10 07:41:20

(c) HOCK international, page 37

Market price of subassembly $50 Variable cost of subassembly $20 Excess capacity (in units) 1,000 Assembling Division Number of units needed 900

What is the natural bargaining range for the two divisions?

A. Between $20 and $50.B. $50 is the only acceptable price.C. Any amount less than $50.D. Between $50 and $70.

A. The basic issue of transfer prices is simply how much should one unit of a company charge another unit of the same company for its goods or services. The goal in setting a transfer price is that the method used will stimulate the department managers to do what will provide the greatest benefit to the company as a whole, rather than to act in their own interest. When there is an external market for the product, market price is almost always the best transfer price to use. Thus, market price is in the natural range. In this situation the company has idle capacity and variable cost approach to determine transfer price works well. Since Fabricating Division has enough capacity to fulfill demand of Assembling Division without any over-times variable cost approach is also acceptable.

B. The acceptable range is between $20 and $50.

C. The transfer price should not be lower than variable costs of production, even if there is excess capacity. As long as selling price covers the variable costs and even a very small amount of fixed costs in case of excess capacity, it is beneficial to the company.

D. It would subvert overall profit goals of the company to choose a transfer price above the free market level.

Question 66 - CMA 692 3-17 - Variances - Material and Labor

An organization that specializes in reviewing and editing technical magazine articles sets the following standards for evaluating the performance of the professional staff:

Annual budgeted fixed costs for normal capacity level of 10,000 articles reviewed and edited: $600,000Standard professional hours per 10 articles: 200 hoursFlexible budget of standard labor costs to process 10,000 articles: $10,000,000

The following data apply to the 9,500 articles that were actually reviewed and edited during the current year:Total hours used by professional staff: 192,000 hoursFlexible costs: $9,120,000Total cost: 9,738,000

The labor efficiency variance for the year is

A. $100,000 unfavorable.B. $500,000 favorable.C. $238,000 unfavorable.D. $380,000 favorable.

A. The labor efficiency variance is calculated as follows: (Standard Hours for Actual Output - Actual Quantity) * Standard Rate. The standard hours for the actual output are 190,000 (20 hours per one article, 9,500 articles

Part 2 : 11/11/10 07:41:20

(c) HOCK international, page 38

that were actually reviewed and edited). The standard rate is $1,000 per article ($10,000,000 / 10,000) and $50 per labor hour ($1,000 / 20). Putting all of this into the formula, the labor efficiency variance is $100,000 unfavorable [(190,000 - 192,000) * $50].

B. The labor efficiency variance is calculated as follows: (Standard Hours for Actual Output - Actual Quantity) * Standard Rate. We have to use the standard hours allowed for the actual output (190,000), instead of the hours in the fixed budget 200,000. From that number we have to subtract actual hours (192,000), not standard hours allowed for actual output (190,000).

C. This is the difference between the standard labor costs and total actual costs, which does not mean anything.

D. This answer is incorrect. See the correct answer for a complete explanation.

Question 67 - CIA 587 IV-15 - Responsibility Accounting

Overtime conditions and pay were recently set by the personnel department. The production department has just received a request for a rush order from the sales department. The production department protests that additional overtime costs will be incurred as a result of the order. The sales department argues that the order is from an important customer. The production department processes the order. To control costs, which department should never be charged with the overtime costs generated as a result of the rush order?

A. Shared by production department and sales department.B. Sales department.C. Personnel department.D. Production department.

A. The overtime premium should be charged to the department that is responsible for the overhead being worked. If the reason that overtime is worked is simply that the volume of work requires it, the overtime premium is charged to overhead and allocated to all of the units produced. Given the situation in this question, it is reasonable that at the overtime premium be charged to the production and sales departments.

B. The overtime premium should be charged to the department that is responsible for the overhead being worked. If the reason that overtime is worked is simply that the volume of work requires it, the overtime premium is charged to overhead and allocated to all of the units produced. Given the situation in this question, it is reasonable that at least some of the overtime premium be charged to the sales department.

C. The overtime premium should be charged to the department that is responsible for the overhead being worked. If the reason that overtime is worked is simply that the volume of work requires it, the overtime premium is charged to overhead and allocated to all of the units produced. Because the personnel department would not have been able to cause or prevent the overtime work, the costs should not be charged to the personnel department.

D. The overtime premium should be charged to the department that is responsible for the overhead being worked. If the reason that overtime is worked is simply that the volume of work requires it, the overtime premium is charged to overhead and allocated to all of the units produced. Given the situation in this question, it is reasonable that at least some of the overtime premium be charged to the production department.

Question 68 - CMA 1290 3-6 - Overhead

Franklin Glass Works' production budget for the year ended November 30 was based on 200,000 units. Each unit requires two standard hours of labor for completion. Total overhead was budgeted at $900,000 for the year, and the fixed overhead rate was estimated to be $3.00 per unit. Both fixed and variable overhead are assigned to the product

Part 2 : 11/11/10 07:41:20

(c) HOCK international, page 39

on the basis of direct labor hours. The actual data for the year ended November 30 are presented as follows.

Actual production in units 198,000 Actual direct labor hours 440,000 Actual variable overhead $352,000 Actual fixed overhead $575,000

Franklin's variable overhead efficiency variance for the year is

A. $33,000 favorable.B. $33,000 unfavorable.C. $35,520 favorable.D. $66,000 unfavorable.

A. This answer is incorrect. See the correct answer for a complete explanation.

B. The variable overhead efficiency variance determines the amount of the total variance that was caused by a different usage of the allocation base than was expected (i.e. the standard hours for the actual output). The variable overhead efficiency variance is calculated as: (Standard Activity Level for Actual Output - Actual Activity Level) * Standard Application Rate. The fixed overhead rate was estimated to be $3.00 per unit, thus the budgeted amount of fixed overhead for budgeted level of production of 200,000 units was $600,000 ($3 * 200,000). Thus, the budgeted amount of variable overhead was $300,000. Each unit requires two standard hours of labor, hence the labor rate per unit was $0.75 ($300,000 / (200,000*2)). The standard hours were 396,000 (198,000 * 2) and the actual hours were 440,000. Now we can calculate the variable overhead efficiency variance as follows: (396,000 - 440,000) * $0.75 = (33,000) unfavorable variance.

C. This answer is incorrect. See the correct answer for a complete explanation.

D. This answer was calculated using incorrect labor rate of $1.50. It was caused by using the wrong number of hours budgeted (200,000) for production. In fact, 400,000 hours was scheduled for production, as each unit requires two standard hours of labor and 200,000 was planned to produce. See the correct answer for a complete explanation.

Question 69 - CMA 695 3-23 - Other

Blaster Inc., a manufacturer of portable radios, purchases the components from subcontractors to use to assemble into a complete radio. Each radio requires three units each of Part XBEZ52, which has a standard cost of $1.45 per unit. During May, Blaster experienced the following with respect to Part XBEZ52.

Units Purchases ($18,000) 12,000 Consumed in manufacturing 10,000 Radios manufactured 3,000

During May, Blaster Inc. incurred a purchase price variance of

A. $450 unfavorable.B. $450 favorable.C. $500 favorable.D. $600 unfavorable.

A. The price variance is calculated as follows: (SP - AP) * AQ. This answer is based on standard per output quantity usage (9,000 units) instead of the actual quantity purchased (12,000). See the correct answer for a complete explanation.

Part 2 : 11/11/10 07:41:20

(c) HOCK international, page 40

B. This answer is incorrect. See the correct answer for a complete explanation.

C. This answer is incorrect. See the correct answer for a complete explanation.

D. The price variance is calculated as follows: (SP - AP) * AQ. The standard price is $1.45 per unit, the actual price is $1.50 per unit ($18,000 / 12,000), and actual quantity purchased was 12,000 units. The price variance is $600 unfavorable [($1.45 - $1.50) * 12,000].

Question 70 - CIA 594 III-72 - Variances - Material and Labor

A company manufactures one product and has a standard cost system. In April the company had the following experience:

Direct Materials Direct Labor Actual $/unit of input (lbs. & hrs.) $28 $18 Standard price/unit of input $24 $20 Standard inputs allowed per unit of output 10 4 Actual units of input 190,000 78,000 Actual units of output 20,000 20,000

The direct materials price variance for April is

A. $240,000 unfavorable.B. $760,000 unfavorable.C. $156,000 favorable.D. $760,000 favorable.

A. This is the materials efficiency variance, which is the answer to a different question. See the correct answer for a complete explanation.

B. The direct materials price variance is calculated as follows: (Standard Price - Actual Price) * Actual Quantity. All the components of the formula are in the data given. The direct materials price variance is $760,000 unfavorable ($24 - $28) * 190,000.

C. The variance is unfavorable because the actual price ($28) is higher than standard ($24). See the correct answer for a complete explanation.

D. The variance is unfavorable because the actual price ($28) is higher than standard ($24). See the correct answer for a complete explanation.

Question 71 - CMA 693 3-26 H1 - Variances - General

Which one of the following variances is of least significance from a behavioral control perspective?

A. Unfavorable labor efficiency variance amounting to 10% more than the budgeted hours for the output attained.B. Fixed overhead volume variance resulting from management's decision midway through the fiscal year to reduce its budgeted output by 20%.C. Favorable labor rate variance resulting from an inability to hire experienced workers to replace retiring workers.D. Unfavorable material quantity variance amounting to 20% of the quantity allowed for the output attained.

A. An unfavorable labor efficiency variance means that workers spent more time to perform work than was budgeted.

Part 2 : 11/11/10 07:41:20

(c) HOCK international, page 41

There could be many reasons to this: work was performed by new unskilled workers, poor usage of materials or technology, etc. So this variance is quite significant from a behavioral control perspective.

B. Fixed overhead volume variance is difference between the amount of fixed overhead applied (standard rate * standard input for the actual level of output) and the budgeted amount of fixed overhead. It is not very much involved with performance of employees, as in most cases top management is responsible for this variance. This variance could be caused by different reasons: economic situation, weather or change in planned output as it is in this question. So this variance is least significance from a behavioral control perspective.

C. The flip side of the favorable labor rate variance resulting from an inability to hire experienced workers to replace retiring workers could be an unfavorable labor efficiency variance or even unfavorable material quantity variance (if waste occurs). So this variance is quite significance from a behavioral control perspective.

D. An unfavorable material quantity variance means that workers used more material than was budgeted. Variance analysis is supposed to give an answer to the question 'who is responsible for the variance'. In this situation the excessive use of material could come from either unskilled workers who could spoil some of material or from purchase of poor quality material or both. So this variance is quite significant from a behavioral control perspective.

Question 72 - CIA 1188 IV-23 - Transfer Pricing

The price that one division of a company charges another division for goods or services provided is called the

A. Market price.B. Distress price.C. Transfer price.D. Outlay price.

A. The market price is determined purely by the forces of supply and demand without interference from an outside source. This concept assumes that markets are efficient, which is not always true in practice. Market price is also an approach to determine a transfer price.

B. The distress price is a reduced price (sometimes significantly) occurs in urgent sales. Distress price is also an approach to determine a transfer price.

C. By definition. The transfer price is the price charged by one unit of the company to another unit of the same company for the services or goods produced by the first unit and "sold" to the second unit.

D. The outlay price is an approach to determine a transfer price.

Question 73 - CMA 1296 3-21 - Variances - Material and Labor

David Rogers, purchasing manager at Fairway Manufacturing Corporation, was able to acquire a large quantity of raw material from a new supplier at a discounted price. Marion Conner, inventory supervisor, is concerned because the warehouse has become crowded and some things had to be rearranged. Brian Jones, vice president of production, is concerned about the quality of the discounted material. However, the Engineering Department tested the new raw material and indicated that it is of acceptable quality. At the end of the month, Fairway experienced a favorable materials usage variance, a favorable labor usage variance, and a favorable materials price variance. The usage variances were solely the result of a higher yield from the new raw material. The favorable materials price variance would be considered the responsibility of the

A. Engineering manager.B. Inventory supervisor.

Part 2 : 11/11/10 07:41:20

(c) HOCK international, page 42

C. Vice president of production.D. Purchasing manager.

A. The engineering manager does not participate in purchasing activity and usually can't influence the price of materials.

B. The inventory supervisor is not responsible for the price variance, because he or she does not participate in purchasing activity and usually can't influence the price of materials.

C. The vice president of production was concerned only about the quality of new material in this situation. The vice president does not participate in purchasing activity and usually can't influence the price of materials.

D. The material price variance is the difference between the standard price and the actual price times the actual quantity of materials consumed by production. The purchasing manager is usually responsible for the material price variance as the price is determined during the purchasing activity.

Question 74 - CMA 694 3-27 - Responsibility Accounting

The Stonebrook Company uses a performance reporting system that reflects the company's decentralization of decision making. The departmental performance reports show actual costs incurred during the period against budgeted costs. Any variances from the budget are assigned to the individual department manager who controls the costs. Stonebrook is using a type of system called

A. Flexible budgeting.B. Transfer-pricing accounting.C. Activity-based budgeting.D. Responsibility accounting.

A. Flexible budgeting is the preparation of a budget for different levels of activity. See the correct answer for a complete explanation.

B. Transfer prices are the prices used in the 'sale' of goods within the company. See the correct answer for a complete explanation.

C. Activity based budgeting is a budgeting concept in which the budget is built using activities that cause costs to be incurred. See the correct answer for a complete explanation.

D. Responsibility accounting is a system in which an individual is held accountable (responsible) for things that they are able to control. It is this person's responsibility to address any issues that arise related to these costs that they control.

Question 75 - CMA 1295 3-3 - Variances - General

The variance that arises solely because the quantity actually sold differs from the quantity budgeted to be sold is

A. Sales mix variance.B. Static budget variance.C. Master budget increment.D. Sales volume variance.

A. The sales mix variance arises when the actual mix of products sold differs from the budgeted mix, though, the total quantity of products sold can be the same as budgeted.

Part 2 : 11/11/10 07:41:20

(c) HOCK international, page 43

B. Besides costs, the static budget also includes revenues. Thus, the static budget variance is the difference between budgeted costs and revenues and the actual results.

C. The master budget increment is an increase of budgeted amount of company's master budget.

D. The sales volume variance measures the impact of difference in sales volume and is calculated as follows: (Actual Sales Volume - Budgeted Sales Volume) * Standard Contribution per Unit.

Question 76 - CMA 693 3-15 - Variances - General

The flexible budget for the month of May was for 9,000 units with direct material at $15 per unit. Direct labor was budgeted at 45 minutes per unit for a total of $81,000. Actual output for the month was 8,500 units with $127,500 in direct material and $77,775 in direct labor expense. The direct labor standard of 45 minutes was maintained throughout the month. Variance analysis of the performance for the month of May would show a(n)

A. Favorable material usage variance of $7,500.B. Favorable direct labor efficiency variance of $1,275.C. Unfavorable direct labor price variance of $1,275.D. Unfavorable direct labor efficiency variance of $1,275.

A.

This problem does not give enough information to enable calculation of a materials usage variance. The materials quantity, or usage, variance is the difference between the standard quantity of materials for the actual output and the actual quantity of materials that were used for the actual output, multiplied by the standard price per unit of direct materials. The problem does not give us either the standard quantity or the actual quantity of materials per unit of output. Nor does it give us the standard price per unit of direct materials. We are given only the standard price for the total direct materials per unit of finished product.

Therefore, we have no way of telling whether the actual quantity used was different from the quantity planned for the output; and we have no way of telling whether the actual price per unit was different from the planned price per unit. By dividing the actual material cost by the actual number of units produced and the planned materials costs by the planned number of units, we can determine that the actual direct materials cost was $15 per finished unit of product and the planned direct materials cost was also $15 per finished unit of product, so we know that the total direct materials variance (price and quantity) was zero. However, that could consist of any variances at all for price and quantity that would net to zero.

Since there is just not enough information given in this problem to permit a calculation of the materials usage variance (or the materials price variance, either), it cannot be said that the variance was $7,500 favorable.

B.

Because the budgeted and actual hours were the same, there was no direct labor efficiency variance. The formula is: (SQ - AQ) * SP or (Standard Hours for Actual Output - Actual Hours) * Standard Rate. We know that the company maintained the direct labor standard of 45 minutes, or .75 of an hour per unit, through the month. The standard labor hour rate is $12 per hour, calculated as follows: 9,000 units planned * .75 of an hour per unit = total hours planned of 6,750. A total cost of $81,000 was planned, so therefore, the standard hourly rate must be $81,000 / 6,750, or $12 per hour.

The Standard Hours for the Actual Output is .75 * 8,500, or 6,375 hours. Since we are told that the direct labor standard was maintained, the Actual Hours must be the same as the Standard Hours, or 6,375.

Putting the numbers into the formula, we get (6,375 - 6,375) * $12 = 0. However, it is not really necessary to go through all those calculations. Because the problem states that the direct labor standard was maintained throughout

Part 2 : 11/11/10 07:41:20

(c) HOCK international, page 44

the period, we can know that the direct labor efficiency variance must be zero.

C.

The direct labor price variance is calculated (SP - AP) * AQ or (Standard Rate - Actual Rate) * Actual Hours. The standard labor rate per hour is $12, calculated as follows: the labor standard is 45 minutes per unit, or .75 of an hour. 9,000 units were planned, so total planned hours was .75 * 9,000 or 6,750 hours. Total planned cost was $81,000, so the standard cost per hour was $81,000 / 6,750 hours, or $12 per hour.

The actual labor rate was $12.20, calculated as follows: The labor standard of 45 minutes per unit or .75 of an hour per unit was maintained. 8,500 units were actually produced, so the total actual hours was .75 * 8,500 or 6,375 hours. Total actual cost was $77,775, so the actual cost per hour (the actual rate) was $77,775 / 6,375 hours, or $12.20 per hour.

Putting the numbers into the formula, we get ($12 - $12.20) * 6,375 = $1,275 Unfavorable.

D.

Because the budgeted and actual hours were the same, there was no direct labor efficiency variance. The formula is: (SQ - AQ) * SP or (Standard Hours for Actual Output - Actual Hours) * Standard Rate. We know that the company maintained the direct labor standard of 45 minutes, or .75 of an hour per unit, through the month. The standard labor hour rate is $12 per hour, calculated as follows: 9,000 units planned * .75 of an hour per unit = total hours planned of 6,750. A total cost of $81,000 was planned, so therefore, the standard hourly rate must be $81,000 / 6,750, or $12 per hour.

The Standard Hours for the Actual Output is .75 * 8,500, or 6,375 hours. Since we are told that the direct labor standard was maintained, the Actual Hours must be the same as the Standard Hours, or 6,375.

Putting the numbers into the formula, we get (6,375 - 6,375) * $12 = 0. However, it is not really necessary to go through all those calculations. Because the problem states that the direct labor standard was maintained throughout the period, we can know that the direct labor efficiency variance must be zero.

Question 77 - CIA 588 IV-19 - Transfer Pricing

The alpha division of a company, which is operating at capacity, produces and sells 1,000 units of a certain electronic component in a perfectly competitive market. Revenue and cost data are as follows:

Sales $50,000 Variable costs 34,000 Fixed costs 12,000

The minimum transfer price that should be charged to the beta division of the same company for each component is

A. $34B. $12C. $50D. $46

A. Because the alpha division is operating at capacity, the minimum price it will charge an internal division is the market price. See the correct answer for a complete explanation.

B. Because the alpha division is operating at capacity, the minimum price it will charge an internal division is the market price. See the correct answer for a complete explanation.

Part 2 : 11/11/10 07:41:20

(c) HOCK international, page 45

C. Because the alpha division is operating at capacity, the minimum price it will charge an internal division is the market price. The market price per unit is $50. If they were to charge less than the market price they would be losing money since they could sell that same unit to a different customer at a higher price.

D. Because the alpha division is operating at capacity, the minimum price it will charge an internal division is the market price. See the correct answer for a complete explanation.

Question 78 - CMA 1293 3-22 - Variances - Material and Labor

ChemKing uses a standard costing system in the manufacture of its single product. The 35,000 units of raw material in inventory were purchased for $105,000, and two units of raw material are required to produce one unit of final product. In November, the company produced 12,000 units of product. The standard allowed for material was $60,000, and there was an unfavorable quantity variance of $2,500.

ChemKing's standard price for one unit of material is

A. $3.00.B. $2.00.C. $5.00.D. $2.50.

A. This is the actual price of raw materials ($105,000 ÷ 35,000 = $3), not the standard price. See the correct answer for a complete explanation.

B. This answer is incorrect. See the correct answer for a complete explanation.

C. This is the standard direct materials cost per unit of product. However, the question asks for the standard cost per unit of direct material. See the correct answer for a complete explanation.

D. The standard price is calculated using budgeted amounts. ChemKing expected to make 12,000 units, which would require 24,000 units of raw material. Since the budgeted cost of these raw material units was $60,000, the cost per unit is $2.50 ($60,000 ÷ 24,000).

Question 79 - CIA 593 IV-14 - Other

For a single-product company, the sales volume variance is

A. The difference between flexible budget and actual sales volume, times master budget unit contribution margin.B. The difference between actual and master budget sales volume, times actual unit contribution margin.C. The difference between flexible budget and master budget sales volume, times actual budget unit contribution margin.D. The difference between flexible budget and master budget sales volume, times master budget unit contribution margin.

A. There is no difference between the flexible budget and the actual sales volume as the actual sales volume is a flexible budget figure because the flexible budget uses the actual level of output.

B. The sales volume variance formula is: (Actual Sales Volume - Budgeted Sales Volume) * Standard Contribution per Unit. As we can see from the formula, the standard unit contribution margin is used in calculation, not the actual contribution margin.

C. The sales volume variance basic formula is: (Actual Sales Volume - Budgeted Sales Volume) * Standard

Part 2 : 11/11/10 07:41:20

(c) HOCK international, page 46

Contribution per Unit. As we can see from the formula the standard unit contribution margin is used in calculation, not the actual number.

D. The sales volume variance formula is: (Actual Sales Volume - Budgeted Sales Volume) * Standard Contribution per Unit. Standard contribution per unit is defined at the beginning of the year using the master budget. The budgeted sales volume is defined at the beginning of the year and it is a master budget figure. Actual sales volume is a flexible budget figure as flexible budget uses the actual level of output. Thus, the sales volume variance is the difference between flexible budget and master budget sales volume, times master budget unit contribution margin.

Question 80 - CIA 1191 IV-16 - Variances - General

A company producing a single product employs the following direct material cost standard for each unit of output:

3 pounds of material x $4/pound = $12/output unitData regarding the operations for the current month are as follows:

Planned production 26,000 units Actual production 23,000 units Actual purchases of direct materials (75,000 pounds) $297,000 Direct materials used in Production 70,000 pounds

What would be the amount of the direct materials purchase price variance and direct materials quantity variance that the company would recognize for the month?

Purchase Price Variance / Quantity Variance

A. $3,000 favorable / $4,000 unfavorableB. $3,000 favorable / $24,000 unfavorableC. $3,120 favorable / $32,000 favorableD. $2,800 favorable / $4,000 unfavorable

A. The purchase price variance is calculated as follows: (Standard Price - Actual Price) * Actual Quantity. It is calculated using all of the units purchased, not just the units that are put into production. Actual price*actual quantity equals $297,000. The standard price * actual quantity equals $300,000 ($4/pound * 75,000 pounds). The purchase price variance is $3,000 favorable ($300,000 - $297,000). The quantity variance is calculated as follows: (Standard Quantity for Actual Output - Actual Quantity) * Standard Price. The standard quantity for actual level of output is 69,000 pounds (3 lb.* 23,000 units). Hence, the quantity variance is $4,000 unfavorable [(69,000 - 70,000) * $4].

B. The quantity variance is $4,000 unfavorable. See the correct answer for a complete explanation.

C. This answer is incorrect. See the correct answer for a complete explanation.

D. The purchase price variance is $3,000 favorable. It is calculated using all of the units purchased (75,000), not just the units that are put into production (70,000). See the correct answer for a complete explanation.

Question 81 - CMA 1295 3-5 - Responsibility Accounting

Responsibility accounting defines an operating center that is responsible for revenue and costs as a(n)

Part 2 : 11/11/10 07:41:20

(c) HOCK international, page 47

A. Profit center.B. Operating unit.C. Division.D. Revenue center.

A. By definition, a profit center is responsible for costs and revenues.

B. An operating unit could be any of the different types of centers.

C. A division could be any of the different types of centers.

D. A revenue center is responsible only for revenues, and not costs.

Question 82 - CMA 696 3-23 - Variances - Material and Labor

Ardmore Enterprises uses a standard cost system in its small appliance division. The standard cost of manufacturing one unit of Zeb is as follows:

Materials - 60 pounds at $1.50 per pound $90 Labor - 3 hours at $12 per hour 36 Factory overhead - 3 hours at $8 per hour 24 Total standard cost per unit $150

The budgeted variable factory overhead rate is $3 per labor hour, and the budgeted fixed factory overhead is $27,000 per month. During May, Ardmore produced 1,650 units of Zeb compared with a normal capacity of 1,800 units. The actual cost per unit was as follows:

Materials (purchased and used) - 58 pounds at $1.65 per pound $95.70 Labor - 3.1 hours at $12 per hour 37.20 Factory overhead - $39,930 per 1,650 units 24.20 Total actual cost per unit $157.10

The materials price variance for May is

A. $14,355 favorable.B. $14,355 unfavorable.C. $14,850 favorable.D. $14,850 unfavorable.

A. The variance is unfavorable as the actual price of pound of material ($1.65) was greater than budgeted ($1.50). See the correct answer for a complete explanation.

B. The price variance is calculated as follows: (Standard Price - Actual Price) * Actual Quantity. The actual price was $1.65 per pound and the standard price was $1.50 per pound. The actual quantity used was 95,700 pounds (1,650 units produced and 58 pounds per unit). Putting these numbers into the formula, we get (1.50 - $1.65) * 95,700. The unfavorable variance is $14,355.

C. The variance is unfavorable as the actual price of pound of material ($1.65) was greater than budgeted ($1.50). See the correct answer for a complete explanation.

D. The price variance is calculated as follows: (Standard Price - Actual Price) * Actual Quantity. This incorrect answer is based on using the standard quantity, not the actual quantity. See the correct answer for a complete explanation.

Part 2 : 11/11/10 07:41:20

(c) HOCK international, page 48

Question 83 - CMA 695 3-10 - Control Concepts and Principles

A standard costing system is most often used by a firm in conjunction with

A. Management by objectives.B. Flexible budgets.C. Target (hurdle) rates of return.D. Participative management programs.

A. MBO can be used in conjunction with a standard costing system, but they are not necessarily related. The primary MBO objective is goal congruence; it is a behavioral, communication-oriented, responsibility approach system.

B. A standard cost is an estimate of the cost the company expects to incur in the production process. Without a standard cost, the analysis of actual activities and results is very difficult because there is no standard to measure the performance against. Standard costs are best used with a flexible budgeting system in order to provide the most useful variance analysis. Flexible budget makes estimation of what costs should be for every level of output.

C. The rate of return considers revenues and costs. A standard costing system controls costs instead of than revenue.

D. Participative management lies in the area of communication and does not relate to standard costs.

Question 84 - CMA 695 3-20 - Performance Measurement

REB Service Co. is a computer service center. For the month of May 1995, REB had the following operating statistics:

Sales $450,000 Operating income 25,000 Net profit after taxes 8,000 Total assets 500,000 Shareholders' equity 200,000 Cost of capital 6%

Based on the above information, which one of the following statements is correct? REB has a

A. Return on investment of 4%.B. Residual income of $(22,000).C. Residual income of $(5,000).D. Return on investment of 1.6%.

A. Return on investment is calculated as operating income divided by total assets. The total assets were $500,000 and operating income was $25,000. This gives a return on investment of 5%.

B. This answer is incorrect. See the correct answer for a complete explanation.

C. The target income for this company is $30,000, as this is 6% of the assets of the company. As the income was only $25,000, the residual income is $(5,000).

D. Return on investment is calculated as operating income divided by total assets. The total assets were $500,000 and operating income was $25,000. This gives a return on investment of 5%.

Part 2 : 11/11/10 07:41:20

(c) HOCK international, page 49

Question 85 - CMA 1292 3-30 - Performance Measurement

Richmond Enterprises is reviewing its policies and procedures in an effort to enhance goal congruence throughout the organization. The processes that are most likely to encourage this behavior are

A. Participatory budgeting, reciprocal cost allocation, and management-by-objective performance evaluation.B. Cost-based transfer pricing, imposed budgeting, and activity-based costing.C. Cost-based transfer pricing, management-by-objective performance evaluation, and participatory budgeting.D. Reciprocal cost allocation, zero-base budgeting, and standard costing.

A. Of the choices given, these are the elements that are most likely to promote goal congruence throughout the organization.

B. A cost based transfer pricing policy does not encourage the control of costs by the producing department. Imposed budgeting also does not promote effective coordination between different departments in the organization.

C. A cost based transfer pricing policy does not encourage the control of costs by the producing department.

D. While there is nothing in this choice that would be detrimental to goal congruence in the organization, standard costing and zero-based budgeting are not as supportive of this as another choice is.

Question 86 - CMA 694 3-22 - Variances - Material and Labor

Under a standard cost system, labor price variances are usually not attributable to

A. The use of a single average standard rate.B. The assignment of different skill levels of workers than planned.C. Union contracts approved before the budgeting cycle.D. Labor rate predictions.

A. If the standard labor rate was set using a single average standard rate it may cause a labor efficiency variance. If the rate doesn't reflect the proportion of hours worked of each salary rate group of workers, a variance will result.

B. The assignment of different skill levels of workers than was planned in most cases will cause a labor price variances. The labor rate variance is calculated as: (Standard Rate - Actual Rate) * Actual Hours. Since there is a difference between the rates of assigned (with lower or higher working skills) and planned workers the variance will most likely occur.

C. The labor price or rate variance is calculated as: (Standard Rate - Actual Rate) * Actual Hours. The only figure Union contracts can influence is the labor rate. The standard rate is set by the budget at the beginning of the year. As Union contracts are approved before the budgeting cycle the information of potential changes in wages and salaries is already included in the budget and standards. According to such Union contract the actual labor rate has to be equal to the standard rate.

D. As all future predictions, labor rate predictions has a portion of inaccuracy. If the standard labor rate was set using these predicted numbers it may cause labor efficiency variance.

Question 87 - CIA 1189 IV-18 - Variances - Material and Labor

One of the items produced by a manufacturer of lawn and garden tools is a chain saw. The direct labor standard for assembling and testing a chain saw is 2.5 hours at $8 per hour. Budgeted production for October was 1,200 units.

Part 2 : 11/11/10 07:41:20

(c) HOCK international, page 50

Actual production during the month was 1,000 units, and direct labor cost was $27,840 for 3,200 hours. Using a two-variance system, what is the direct labor efficiency variance?

A. $2,240 unfavorable.B. $5,600 unfavorable.C. $6,090 favorable.D. $5,600 favorable.

A. This is the labor rate variance. See the correct answer for a complete explanation.

B. The labor efficiency variance is calculated as: (Standard Hours for Actual Output - Actual Quantity) * Standard Rate. The standard hours allowed for actual output are 2,500 hr. (2.5 hours per unit, 1000 units were actually produced). The direct labor efficiency variance is $5,600 unfavorable (2,500 - 3,200) * $8. The hours allowed for actual level of production were less than actually spent which gave us an unfavorable labor efficiency variance.

C. The labor efficiency variance is calculated as: (Standard Hours for Actual Output - Actual Quantity) * Standard Rate. In calculation we have to use standard labor rate ($8.00), not actual labor rate of $8.70 ($27,840 / 3,200 hr.).

D. The direct labor efficiency variance is $5,600 unfavorable [(2,500 - 3,200) * $8]. The hours allowed for actual level of production were less than actually spent, which gave us an unfavorable labor efficiency variance.

Question 88 - CMA 695 3-29 - Other

For a company that produces more than one product, the sales volume variance can be divided into which two of the following additional variances?

A. Sales mix variance and production volume variance.B. Sales price variance and flexible budget variance.C. Sales mix variance and sales price variance.D. Sales quantity variance and sales mix variance.

A. This answer is incorrect. See the correct answer for a complete explanation.

B. This answer is incorrect. See the correct answer for a complete explanation.

C. This answer is incorrect. See the correct answer for a complete explanation.

D. The sales volume variance measures the impact of the difference in sales between actual and budgeted. When there is more than one product, this total volume variance will then be broken down into two sub-variances - the sales quantity variance and the sales mix variance. The sales mix variance determines what impact on total sales volume variance the actual mix of products sold made. The sales quantity variance determines what impact on the total sales volume variance the actual quantity of products sold made.

Question 89 - CMA 697 3-22 - Variances - General

The controller for Durham Skates is reviewing the production cost report for July. An analysis of direct materials costs reflects an unfavorable flexible budget variance of $25. The plant manager believes this is excellent performance on a flexible budget for 5,000 units of direct materials. However, the production supervisor is not pleased with this result because he claims to have saved $1,200 in materials cost on actual production using 4,900 units of direct materials. The standard materials cost is $12 per unit. Actual materials used for the month amounted to $60,025.The actual average cost per unit for materials was

Part 2 : 11/11/10 07:41:20

(c) HOCK international, page 51

A. $12.01B. $12.25C. $12.24D. $12.00

A. The actual average cost per unit of material was $12.25. See the correct answer for a complete explanation.

B. Actual materials used for the month amounted to $60,025, and 4,900 units of direct materials were used. The actual average cost per unit of material was $12.25 ($60,025 / 4,900).

C. The actual average cost per unit of material was $12.25. See the correct answer for a complete explanation.

D. This is standard cost per unit of material. The actual average cost per unit of material was $12.25. See the correct answer for a complete explanation.

Question 90 - CIA 1189 IV-17 - Variances - General

One of the items produced by a manufacturer of lawn and garden tools is a chain saw. The direct labor standard for assembling and testing a chain saw is 2.5 hours at $8 per hour. Budgeted production for October was 1,200 units. Actual production during the month was 1,000 units, and direct labor cost was $27,840 for 3,200 hours. Using a two-variance system, what was the direct labor price (rate) variance for October?

A. $2,240 favorable.B. $5,600 unfavorable.C. $2,240 unfavorable.D. $3,840 favorable.

A. The budgeted labor rate ($8.00) was less than actual rate ($8.70) which gave us an unfavorable rate variance.

B. This is the direct labor efficiency variance. See the correct answer for a complete explanation.

C. The labor rate variance is calculated as follows: (Standard Rate - Actual Rate) * Actual Hours. The actual labor rate was $8.70 ($27,840 / 3,200). The labor rate variance is $2,240 unfavorable ($8.00 - $8.70)*3,200. The budgeted labor rate was less than actual which gave us the unfavorable rate variance.

D. This is the difference of actual labor cost of $27,840 and the standard labor cost of scheduled level of production $24,000 ($8 * 2.5 hours * 1,200 units). See the correct answer for a complete explanation.

Question 91 - CMA Sample Q3-2 - Responsibility Accounting

Consider the following information for Richardson Company for the prior year.The company produced 1,000 units and sold 900 units, both as budgeted.There were no beginning or ending work-in-process inventories and no beginning finished goods inventory.Budgeted and actual fixed costs were equal, all variable manufacturing costs were affected by production volume only, and all variable selling costs were affected by sales volume only.

Budgeted per unit revenues and costs were as follows:

Per unit Sales price $100

Part 2 : 11/11/10 07:41:20

(c) HOCK international, page 52

Direct materials 30 Direct labor 20 Other variable manufacturing costs 10 Fixed selling costs 5 Variable selling costs 12 Fixed selling costs ($33,600 total) 4 Fixed administrative costs ($1,800 total) 2

The contribution margin earned by Richardson for the prior year was

A. $25,200B. $35,000C. $28,000D. $31,500

A. Contribution margin calculated as revenues minus variable costs. Unit contribution margin is sales price minus variable costs, or $28 per unit ($100 - $30 - $20 - $10 - $12). There were 900 units sold, so the total contribution margin is $25,200 ($28 * 900).

B. See the correct answer for a complete explanation.

C. This answer incorrectly uses the number of units produced instead of the number of units sold. See the correct answer for a complete explanation.

D. See the correct answer for a complete explanation.

Question 92 - CIA 1190 IV-18 - Other

The following exhibit reflects a summary of performance for a single item of a retail store's inventory for April.

Actual

Results

FlexibleBudget

Variances FlexibleBudget

Static(Master)Budget

Sales(units) 11,000 11,000 12,000 Revenue (sales) $208,000 $12,000 U $220,000 $240,000 Variable costs 121,000 11,000 U 110,000 120,000 Contribution Margin $87,000 $23,000 U $110,000 $120,000 Fixed costs 72,000 72,000 72,000 Operating Income $15,000 $23,000 U $38,000 $48,000

The sales volume variance is

A. $11,000 F.B. $10,000 U.C. $12,000 U.D. $1,000 F.

A. The sales volume variance is unfavorable as the master budget contribution margin is higher than flexible budget contribution margin. See the correct answer for a complete explanation.

B. The sales volume variance measures the impact of difference in sales volume and is calculated as follows: (Actual Sales Volume - Budgeted Sales Volume) * Standard Contribution per Unit. Actual sales volume times standard contribution per unit is the flexible budget figure, i.e. $110,000. Budgeted sales volume times standard contribution per unit is the static (master) budget figure, i.e. $120,000. The difference between these

Part 2 : 11/11/10 07:41:20

(c) HOCK international, page 53

two contribution margins is the sales volume variance: $10,000 unfavorable ($110,000 - $120,000).

C. The sales volume variance measures the impact of difference in sales volume and is calculated as follows: (Actual Sales Volume - Budgeted Sales Volume) * Standard Contribution per Unit. This answer is a difference between the actual sales revenue and flexible budgeted revenue, which is not an answer to the question. See the correct answer for a complete explanation.

D. The sales volume variance is unfavorable as the master budget contribution margin is higher than flexible budget contribution margin. See the correct answer for a complete explanation.

Question 93 - CMA 684 4-26 - Variances - Material and Labor

Each unit of Product XK-46 requires three direct labor hours. Employee benefit costs are treated as direct labor costs. Data on direct labor are

Number of direct employees: 25Weekly productive hours per employee: 35Estimated weekly wages per employee: $245Employee benefits (related to weekly wages): 25%

The standard direct labor cost per unit of Product XK-46 is

A. $36.75.B. $26.25.C. $29.40.D. $21.00.

A. There are 35 weekly productive hours per employee, not 25 as was used in this calculation.

B. The estimated weekly costs (wages plus employee benefits) per employee are $306.25 ($245 + $245 * 25%). The hour labor cost is $8.75 ($306.25 / 35 hours per week per employee). Three direct hours are required to produce one unit of product XK-46. Thus, the unit labor cost is $26.25 ($8.75 * 3).

C. This answer is incorrect. See the correct answer for a complete explanation.

D. Employee benefit costs should be treated as direct labor costs. We therefore have to include this amount in calculation.

Question 94 - CIA 589 IV-15 - Responsibility Accounting

In a responsibility accounting system, managers are accountable for

A. Product costs but not for period costs.B. Costs over which they have significant influence.C. Incremental costs.D. Variable costs but not for fixed costs.

A. In a responsibility accounting system, managers are responsible for the costs that they control, whether they are product or period.

B. By definition, in a responsibility accounting system, managers are responsible for the costs that they

Part 2 : 11/11/10 07:41:20

(c) HOCK international, page 54

control, whether they are fixed or variable.

C. In a responsibility accounting system, managers are responsible for the costs that they control, whether they are incremental or not.

D. In a responsibility accounting system, managers are responsible for the costs that they control, whether they are fixed or variable.

Question 95 - CMA 692 3-19 - Variances - Material and Labor

Jackson Industries employs a standard cost system in which direct materials inventory is carried at standard cost. Jackson has established the following standards for the prime costs of one unit of product.

StandardQuantity

StandardPrice

StandardCost

Direct materials 5 pounds $3.60/pound $18.00 Direct labor 1.25 hours 12.00/hour 15.00 $33.00

During May, Jackson purchased 125,000 pounds of direct materials at a total cost of $475,000. The total factory wages for May were $364,000, 90% of which were for direct labor. Jackson manufactured 22,000 units of product during May using 108,000 pounds of direct materials and 28,000 direct labor hours.

The direct materials usage (quantity) variance for May is

A. $7,200 favorable.B. $5,850 unfavorable.C. $7,600 favorable.D. $7,200 unfavorable.

A. The material usage variance is calculated as: (Standard Quantity for Actual Output - Actual Quantity) * Standard Price. The standard quantity allowed for the actual output is 110,000 pounds (5 pounds of direct materials per unit of a product and 22,000 units produced). The material usage variance is $7,200 favorable [(110,000 - 108,000) * $3,60]. The quantity used is less than budgeted for the current level of production so the variance is favorable.

B. This answer is incorrect. See the correct answer for a complete explanation.

C. The material usage variance is calculated as: (Standard Quantity for Actual Output - Actual Quantity) * Standard Price. As we can see from the formula the standard price ($3.60) has to be used, not actual price ($3.80) as was used in this answer.

D. This answer is incorrect. See the correct answer for a complete explanation.

Question 96 - CMA 1290 3-5 - Variances - General

Franklin Glass Works' production budget for the year ended November 30 was based on 200,000 units. Each unit requires two standard hours of labor for completion. Total overhead was budgeted at $900,000 for the year, and the fixed overhead rate was estimated to be $3.00 per unit. Both fixed and variable overhead are assigned to the product on the basis of direct labor hours. The actual data for the year ended November 30 are presented as follows.

Part 2 : 11/11/10 07:41:20

(c) HOCK international, page 55

Actual production in units 198,000 Actual direct labor hours 440,000 Actual variable overhead $352,000 Actual fixed overhead $575,000

The standard hours allowed for actual production for the year ended November 30 total

A. 396,000.B. 400,000.C. 247,500.D. 495,000.

A. Each unit requires two standard hours of labor for completion. The actual production in units is 198,000. Thus, the standard hours allowed for actual production for the year ended November 30 totals 396,000 (198,000 units * 2 hours per unit).

B. Each unit requires two standard hours of labor for completion. The actual production in units is 198,000. In the calculation we have to use the actual number of units (198,000), not the budgeted (200,000) number of units.

C. This answer is incorrect. See the correct answer for a complete explanation.

D. This answer is incorrect. See the correct answer for a complete explanation.

Question 97 - CMA 1292 3-21 - Performance Measurement

A firm earning a profit can increase its return on investment by

A. Increasing sales revenues and operating expenses by the same percentage.B. Increasing investment and operating expenses by the same dollar amount.C. Decreasing sales revenues and operating expenses by the same percentage.D. Increasing sales revenue and operating expenses by the same dollar amount.

A. One way to solve this problem is to simply set up a basic ROI of, for example, $100,000 of profit / $400,000 total assets giving a ROI of .25 and then go through the choices changing these figures outlined in each option. In this choice we should increase sales (which will increase income) and expenses (which will decrease income) by the same percentage. This will cause income to increase by that same percentage. Since the investment is unchanged, the ROI will increase since the income will be more than $100.

B. One way to solve this problem is to simply set up a basic ROI of, for example, $100,000 of profit / $400,000 total assets giving a ROI of .25 and then go through the choices changing these figures outlined in each option. In this choice we should increase the investment (the denominator) and expenses (which will decrease income) by the same amount. Let us assume that this amount is $50. That would make the ROI calculation $50 / $450, which is .111, or a decrease in the ROI.

C. One way to solve this problem is to simply set up a basic ROI of, for example, $100,000 of profit / $400,000 total assets giving a ROI of .25 and then go through the choices changing these figures outlined in each option. In this choice we should decrease sales (which will decrease income) and expenses (which will increase income) by the same percentage. This will cause income to decline by that same percentage and so will decrease the ROI of the company.

D. One way to solve this problem is to simply set up a basic ROI of, for example, $100,000 of profit / $400,000 total assets giving a ROI of .25 and then go through the choices changing these figures outlined in each option. In this choice we should increase sales (which will increase income) and expenses (which will decrease income) by the same amount. This will cause income to remain unchanged and so will not change the ROI of the company.

Part 2 : 11/11/10 07:41:20

(c) HOCK international, page 56

Question 98 - CMA 1294 3-24 - Variances - Material and Labor

Tower Company planned to produce 3,000 units of its single product, Titactium, during November. The standard specifications for one unit of Titactium include 6 pounds of materials at $.30 per pound. Actual production in November was 3,100 units of Titactium. The accountant computed a favorable materials purchase price variance of $380 and an unfavorable materials quantity variance of $120. Based on these variances, one could conclude that

A. The actual usage of materials was less than the standard allowed.B. More materials were used than were purchased.C. The actual cost of materials was less than the standard cost.D. More materials were purchased than were used.

A. An unfavorable materials quantity variance of $120 indicates that more materials were used than the standard allowed, not vice versa.

B. A favorable materials purchase price variance of $380 and an unfavorable materials quantity variance of $120 do not relate to the difference of quantity materials purchased and used. Material efficiency variance is the difference between standard quantity of usage and actual quantity of material used times standard price. The material purchase price variance uses the only actual quantity of material purchase in the calculation.

C. A favorable materials purchase price variance of $380 indicates that the actual price was less than the standard price. We also can prove it calculating the price variance for the actual price. The price variance is calculated as follows: (Standard Price - Actual Price) * Actual Quantity. The standard specifications for one unit of Titactium include 6 pounds of materials at $.30 per pound. Actual production was 3,100 units of Titactium. ($.30 * 6 - AP * 6) * 3,100 = $380.$5,580 - AP * 18,600 = $380. AP = $5,200 / 18,600 = $.028

D. A favorable materials purchase price variance of $380 and an unfavorable materials quantity variance of $120 do not relate to the difference in the quantity of materials purchased and used. The material efficiency variance is the difference between the standard quantity of usage and the actual quantity of material used multiplied by the standard price. The material purchase price variance uses only the actual quantity of material purchase in the calculation.

Question 99 - CIA 1193 IV-18 - Transfer Pricing

One department of an organization, Final Assembly, is purchasing subcomponents from another department, Materials Fabrication. The price that will be charged to Final Assembly by Materials Fabrication is to be determined. Outside market prices for the subcomponents are available. Which of the following is the most correct statement regarding a market-based transfer price?

A. Overall long term competitiveness is enhanced with a market-based transfer price.B. Corporate politics is more of a factor in a market-based transfer price than with other methods.C. Market transfer prices provide an incentive to use otherwise idle capacity.D. Marginal production cost transfer prices provide incentives to use otherwise idle capacity.

A. This is a very general, but true, statement. A market based transfer price will lead to the greatest good for the company in the long-term. Other, artificial, transfer prices will lead to behavior that is not in the best interest of the company as a whole.

B. In all methods of transfer pricing corporate politics will play a role. In market based transfer pricing, corporate politics are probably a little bit less as the market is setting the price and the market is outside the influence of the company, or individuals within it

C. A transfer price equal to the market price will provide incentives for the producing department to increase

Part 2 : 11/11/10 07:41:20

(c) HOCK international, page 57

production, but will not provide any incentive for the purchasing department to purchase internally.

D. If the marginal cost of production is used as the transfer price there is no incentive to use otherwise idle facilities because there will not be any contribution received from the units that are produced. Also, marginal costs of production would not be the transfer price if a market based transfer price is used.

Question 100 - CMA 1289 4-4 - Variances - General

A favorable material price variance coupled with an unfavorable material usage variance would most likely result from

A. Labor efficiency problems.B. Labor mix problems.C. The purchase and use of lower than standard quality material.D. The purchase and use of higher than standard quality material.

A. An unfavorable material usage variance could be caused by labor efficiency problems. However, a favorable material price variance does not relate to such problems.

B. An unfavorable material usage variance could be caused by labor mix problems. However, a favorable material price variance does not relate to such problems.

C. A favorable material price variance means that the price of materials consumed by production was lower than budgeted. An unfavorable material usage variance means that quantity of materials used by production is more than standard quantity. Both variances could be caused by the purchase and use of lower than standard quality material.

D. A favorable material price variance means that the price of materials consumed by production was lower than budgeted. However, an unfavorable material usage variance means that quantity of materials used by production is more than standard quantity.

Question 101 - CIA 594 III-74 - Variances - Material and Labor

A company manufactures one product and has a standard cost system. In April the company had the following experience:

Direct Materials Direct Labor Actual $/unit of input (lbs. & hrs.) $28 $18 Standard price/unit of input $24 $20 Standard inputs allowed per unit of output 10 4 Actual units of input 190,000 78,000 Actual units of output 20,000 20,000

The direct labor rate variance for April is

A. $40,000 unfavorable.B. $240,000 favorable.C. $156,000 favorable.D. $156,000 unfavorable.

A. The direct labor rate variance for April is $156,000 favorable. The standard labor rate was greater than actual, which gives the favorable variance. The formula for labor rate variance is (Standard Rate - Actual Rate) * Actual

Part 2 : 11/11/10 07:41:20

(c) HOCK international, page 58

Hours. As we can see from the formula we have to multiply the difference in labor rates by actual hours (78,000), not the actual units of output (20,000).

B. This is the direct materials efficiency variance. See the correct answer for a complete explanation.

C. The labor rate variance is calculated as follows: (Standard Rate - Actual Rate) * Actual Hours. All the components of the formula are in the data given. ($20 - $18) * 78,000 = $156,000. The direct labor rate variance for April is $156,000 favorable. The standard labor rate was greater than actual, which gives the favorable variance.

D. The direct labor rate variance for April is $156,000 favorable. The standard labor rate was greater than actual, which gives the favorable variance. See the correct answer for a complete explanation.

Question 102 - CMA 1291 3-18 - Other

Folsom Fashions sells a line of women's dresses. Folsom's performance report for November follows.

Actual Budget Dresses sold 5,000 6,000 Sales $235,000 $300,000 Variable costs (145,000) (180,000) Contribution margin 90,000 120,000 Fixed costs (84,000) (80,000) Operating income $6,000 $40,000

The company uses a flexible budget to analyze its performance and to measure the effect on operating income of the various factors affecting the difference between budgeted and actual operating income.

What additional information is needed for Folsom to calculate the dollar impact of a change in market share on operating income for November?

A. Folsom's actual market share and the actual total market size.B. Folsom's budgeted market share and the actual total market size.C. Folsom's budgeted market share, the budgeted total market size, and average market selling price.D. Folsom's budgeted market share and the budgeted total market size.

A. The budgeted market share and actual total market size in units have to be known. See the correct answer for a complete explanation.

B. The market share variance is calculated as follows: (Actual Market Share - Expected Market Share) * Actual Market Size in Units *Standard Weighted Average Contribution Margin per Unit. To determine what was the cause of decrease in sales (internal problems such as loss of competitiveness or external problems such as shrinkage of the market), we need to know the actual market size in units, actual market share, budgeted market shares and standard weighted average contribution margin per unit. In the question, we are given the actual units sold, thus, we can calculate the actual market share if we obtain the additional information of actual market size in units. We also can calculate the standard weighted average contribution margin per unit from the given data. To complete the formula we also need to obtain information of budgeted (expected) market share.

C. The budgeted market share and actual total market size in units have to be known. See the correct answer for a complete explanation.

D. The budgeted market share and actual total market size in units have to be known in order to calculate the impact of a change in market share. See the correct answer for a complete explanation.

Part 2 : 11/11/10 07:41:20

(c) HOCK international, page 59

Question 103 - CMA 692 3-16 - Overhead

An organization that specializes in reviewing and editing technical magazine articles sets the following standards for evaluating the performance of the professional staff:

Annual budgeted fixed costs for normal capacity level of 10,000 articles reviewed and edited: $600,000Standard professional hours per 10 articles: 200 hoursFlexible budget of standard labor costs to process 10,000 articles: $10,000,000

The following data apply to the 9,500 articles that were actually reviewed and edited during the current year:Total hours used by professional staff: 192,000 hoursFlexible costs: $9,120,000Total cost: 9,738,000

The fixed cost spending variance for the year is

A. $30,000 favorable.B. $18,000 favorable.C. $18,000 unfavorable.D. $48,000 unfavorable.

A. This answer is incorrect. See the correct answer for a complete explanation.

B. This answer is incorrect. See the correct answer for a complete explanation.

C. The fixed overhead spending variance is the difference between the budgeted fixed overhead costs and the actual fixed overhead costs. Actual fixed overhead costs are $618,000 ($9,738,000 total cost incurred - $9,120,000 flexible (variable) costs). The fixed overhead spending variance is $18,000 unfavorable ($600,000 budgeted fixed costs - $618,000 actual fixed costs). The actual costs exceed budgeted and that is why the variance is unfavorable.

D. The fixed overhead spending variance is simply the difference between the budgeted fixed overhead costs and the actual fixed overhead costs. This incorrect result is the consequence of the wrong assumption that fixed costs vary with the level of production. In the calculation, we have to use $600,000 of fixed costs, not $570,000 adjusted for the level of production of 9,500 articles. Again, fixed costs do not vary with the level of output.

Question 104 - CMA 686 4-14 - Responsibility Accounting

The segment margin of the Wire Division of Lerner Corporation should not include

A. Net sales of the Wire Division.B. Variable selling expenses of the Wire Division.C. Fixed selling expenses of the Wire Division.D. The Wire Division's fair share of the salary of Lerner Corporation's president.

A. Segment margin includes sales, variable costs, controllable fixed costs and traceable fixed costs. Sales should be included in the segment margin of this division.

B. Segment margin includes sales, variable costs, controllable fixed costs and traceable fixed costs. Variable selling expenses should be included in the segment margin of this division.

Part 2 : 11/11/10 07:41:20

(c) HOCK international, page 60

C. Segment margin includes sales, variable costs, controllable fixed costs and traceable fixed costs. Fixed selling expenses are controllable and should be included in the segment margin of this division.

D. Segment margin includes sales, variable costs, controllable fixed costs and traceable fixed costs. The allocation of the president's salary is an untraceable fixed cost and should not be included in segment margin.

Question 105 - CMA 1294 3-25 - Variances - Material and Labor

An unfavorable direct labor efficiency variance could be caused by a(n)

A. Unfavorable fixed overhead volume variance.B. Unfavorable variable overhead spending variance.C. Favorable variable overhead spending variance.D. Unfavorable materials usage variance.

A. The fixed overhead volume variance is difference between the amount of fixed overhead applied (standard rate * standard input for the actual level of output) and the budgeted amount of fixed overhead. As we can see from the formula it does not relate to direct labor.

B. The variable overhead spending variance is calculated as follows: (Standard Application Rate - Actual Application Rate) * Actual Quantity. As we can see from the formula it does not relate to direct labor.

C. The variable overhead spending variance is calculated as follows: (Standard Application Rate - Actual Application Rate) * Actual Quantity. As we can see from the formula it does not relate to direct labor.

D. The labor efficiency variance is calculated as follows: (Standard Hours for Actual Output - Actual Quantity) * Standard Rate. An unfavorable direct labor efficiency variance means that more hours were actually spent than the standard allowed. This could happen due to an inefficiency on the part of employees, downtime, poor quality of raw materials that required excessive rework. The efficiency or material usage variance is calculated as follows: (Standard Quantity for Actual Output - Actual Quantity) * Standard Price. An unfavorable materials usage variance indicates that more materials were used than allowed for the actual level of output. These two variances may be interrelated as working on more materials than they were supposed to caused the worker to spend more time than they should according to standard for actual output.

Question 106 - CMA 692 3-18 - Other

Jackson Industries employs a standard cost system in which direct materials inventory is carried at standard cost. Jackson has established the following standards for the prime costs of one unit of product.

StandardQuantity

StandardPrice

StandardCost

Direct materials 5 pounds $3.60/pound $18.00 Direct labor 1.25 hours 12.00/hour 15.00 $33.00

During May, Jackson purchased 125,000 pounds of direct materials at a total cost of $475,000. The total factory wages for May were $364,000, 90% of which were for direct labor. Jackson manufactured 22,000 units of product during May using 108,000 pounds of direct materials and 28,000 direct labor hours.

The purchase price variance for the direct materials acquired by Jackson Industries during May is

Part 2 : 11/11/10 07:41:20

(c) HOCK international, page 61

A. $21,600 unfavorable.B. $25,000 unfavorable.C. $28,000 favorable.D. $21,600 favorable.

A. The price variance is (SP - AP) * AQ. The actual price is $3.80 per pound. Note that we use actual quantity purchased in calculations (125,000 lb.), not quantity consumed by production (108,000 lb.) because we need to determine purchase price variance.

B. The price variance is (SP - AP) * AQ. The actual price is $3.80 per pound ($475,000 / 125,000). Note that we use the actual quantity purchased in calculations because we need to determine the purchase price variance. The actual quantity purchased is 125,000 pounds. The purchase price variance is $25,000 unfavorable [($3.60 - $3.80) * 125,000]. Since the purchasing price was higher than budgeted we deal with unfavorable variance.

C. This answer is incorrect. See the correct answer for a complete explanation.

D. This answer is incorrect. See the correct answer for a complete explanation.

Question 107 - CMA 1284 4-2 - Variances - General

Dori Castings, a job-order shop, uses a full-absorption, standard-cost system to account for its production costs. The O/H costs are applied on a direct-labor-hour basis. A production volume variance will exist for Dori in a month when

A. Production volume differs from sales volume.B. Actual direct labor hours differ from standard allowed direct labor hours.C. The fixed factory O/H applied on the basis of standard allowed direct labor hours differs from actual fixed factory O/H.D. The fixed factory O/H applied on the basis of standard allowed direct labor hours differs from the budgeted fixed factory O/H.

A. The fixed overhead volume variance is difference between the amount of fixed overhead applied (standard rate * standard input for the actual level of output) and the budgeted amount of fixed overhead. It does not relate to the difference between the sales and production volume.

B. The fixed overhead volume variance is difference between the amount of fixed overhead applied (standard rate * standard input for the actual level of output) and the budgeted amount of fixed overhead. The difference between actual direct labor hours and standard allowed direct labor hours do not relate to the fixed overhead volume variance.

C. The fixed overhead volume variance is difference between the amount of fixed overhead applied (standard rate * standard input for the actual level of output) and the budgeted amount of fixed overhead. In fact, the difference between the fixed factory overhead applied on the basis of standard allowed direct labor hours and actual fixed factory overhead determines the total fixed overhead variance.

D. By definition. The fixed overhead volume variance is the difference between the amount of fixed overhead applied (standard rate * standard input for the actual level of output) and the budgeted amount of fixed overhead.

Question 108 - CMA 692 3-20 - Variances - Material and Labor

Jackson Industries employs a standard cost system in which direct materials inventory is carried at standard cost. Jackson has established the following standards for the prime costs of one unit of product.

Part 2 : 11/11/10 07:41:20

(c) HOCK international, page 62

StandardQuantity

StandardPrice

StandardCost

Direct materials 5 pounds $3.60/pound $18.00 Direct labor 1.25 hours 12.00/hour 15.00 $33.00

During May, Jackson purchased 125,000 pounds of direct materials at a total cost of $475,000. The total factory wages for May were $364,000, 90% of which were for direct labor. Jackson manufactured 22,000 units of product during May using 108,000 pounds of direct materials and 28,000 direct labor hours.The direct labor price (rate) variance for May is

A. $8,400 unfavorable.B. $7,200 unfavorable.C. $8,400 favorable.D. $6,000 unfavorable.

A. This answer is incorrect. See the correct answer for a complete explanation.

B. This answer is incorrect. See the correct answer for a complete explanation.

C. The labor price/rate variance is calculated as: (Standard Rate - Actual Rate) * Actual Hours. The actual total direct labor cost is $327,600 ($364,000 * 90%). The standard labor rate is $12, thus, standard labor rate at actual hours is $336,000 ($12 * 28,000). The labor price/rate variance is $8,400 favorable ($336,000 - $327,600). The budgeted amount is higher than actual and this gives a favorable variance.

D. This result is the labor efficiency variance, but it is the answer to a different question. See the correct answer for a complete explanation.

Question 109 - CMA 1291 3-17 - Variances - General

Folsom Fashions sells a line of women's dresses. Folsom's performance report for November follows.

Actual Budget Dresses sold 5,000 6,000 Sales $235,000 $300,000 Variable costs (145,000) (180,000) Contribution margin 90,000 120,000 Fixed costs (84,000) (80,000) Operating income $6,000 $40,000

The company uses a flexible budget to analyze its performance and to measure the effect on operating income of the various factors affecting the difference between budgeted and actual operating income.

>The fixed cost variance for November is

A. $5,000 unfavorable.B. $4,000 unfavorable.C. $4,000 favorable.D. $5,000 favorable.

A. This answer is incorrect. See the correct answer for a complete explanation.

Part 2 : 11/11/10 07:41:20

(c) HOCK international, page 63

B. In this question we are asked to calculate the fixed cost variance. It is simply the difference between the budgeted fixed costs and the actual fixed costs. For Folsom, the fixed cost variance is $4,000 unfavorable ($80,000 - $84,000). The actual fixed cost was greater than the budgeted fixed cost, which means that the variance is unfavorable.

C. The budgeted amount of fixed cost was less than actual amount of fixed cost incurred. This means that the variance is unfavorable. See the correct answer for a complete explanation.

D. This is the variable cost variance, not the fixed cost variance. See the correct answer for a complete explanation.

Question 110 - CMA 1293 3-21 - Responsibility Accounting

A successful responsibility accounting reporting system is dependent upon

A. Identification of the management level at which all costs are controllable.B. The correct allocation of controllable variable costs.C. A reasonable separation of costs into their fixed and variable components since fixed costs are not controllable and must be eliminated from the responsibility report.D. The proper delegation of responsibility and authority.

A. Not only is the control of a cost relevant in responsibility accounting, but in some cases the responsibility for the cost being incurred may also be relevant to the allocation of costs for the responsibility report. For example, if a manager makes a decision that causes additional costs to be incurred in a different department, those additional costs should be 'allocated' to the manager who caused them to be incurred, even if that manager is unable to control the costs themselves.

B. There are some fixed costs that will also be controllable and therefore should also be included on the responsibility report.

C. Some fixed costs are controllable by the different managers so they should be included in the responsibility report. See the correct answer for a complete explanation.

D. The identification of who is responsible for the costs being incurred and the control over the costs themselves is critical to the effectiveness of a responsibility accounting system. This can be done through the proper delegation of responsibility and authority.

Question 111 - CMA 693 3-11 - Performance Measurement

Edith Carolina, president of the Deed Corporation, requires a minimum return on investment of 8% for any project to be undertaken by her company. The company is decentralized, and leaves investment decisions up to the discretion of the division managers as long as the 8% return is expected to be realized. Michael Sanders, manager of the Cosmetics Division, has had a return on investment of 14% for his division for the past 3 years and expects the division to have the same return in the coming year. Sanders has the opportunity to invest in a new line of cosmetics which is expected to have a return on investment of 12%.

If the Deed Corporation evaluates managerial performance using return on investment, what will be the preference for taking on the proposed cosmetics line by Edith Carolina and Michael Sanders?

A. CarolinaSanders RejectAcceptB. CarolinaSanders RejectRejectC. CarolinaSanders AcceptAcceptD. CarolinaSanders AcceptReject

Part 2 : 11/11/10 07:41:20

(c) HOCK international, page 64

A. This answer is incorrect. See the correct answer for a complete explanation.

B. This answer is incorrect. See the correct answer for a complete explanation.

C. This answer is incorrect. See the correct answer for a complete explanation.

D. Since return on investment uses a percentage of return as the measurement, Sanders would not accept this proposal. That is because the return of this investment is less than the return that Sanders' division has earned in the past. Therefore, accepting this investment will decrease Sanders' return. On the other hand, Carolina would accept the project because the return of this project is greater than the 8% required by the company.

Question 112 - CMA 1295 3-11 - Variances - Material and Labor

In a standard cost system, the investigation of an unfavorable material usage variance should begin with the

A. Purchasing manager only.B. Plant controller only.C. Production manager only.D. Production manager or the purchasing manager.

A. The production department can also be involved in occurrence of this variance.

B. The plant controller is a high managerial level and this manager is not the person most responsible for an unfavorable materials usage variance.

C. The purchasing department can also be involved in occurrence of this variance because the purchase of a lower quality material may lead to more wasted materials.

D. An unfavorable material usage variance means that more materials were consumed by production then had been budgeted. It can result from a number of reasons: poor production employees' performance, product design, waste, theft, and poor material quality, etc. Thus, the investigation of this variance should begin either with the production manager or the purchasing manager.

Question 113 - CIA 1193 IV-19 - Transfer Pricing

The Eastern division sells goods internally to the Western division of the same company. The quoted external price in industry publications from a supplier near Eastern is $200 per ton plus transportation. It costs $20 per ton to transport the goods to Western. Eastern's actual market cost per ton to buy the direct materials to make the transferred product is $100. Actual per ton direct labor is $50. Other actual costs of storage and handling are $40. The company president selects a $220 transfer price. This is an example of

A. Cost plus 20% transfer pricing.B. Negotiated transfer pricing.C. Cost-based transfer pricing.D. Market-based transfer pricing.

A. Since the transfer price that has been set is the market price, this is market based transfer pricing. A cost plus 20% transfer price would be based off of the cost of $190.

B. Since the transfer price that has been set is the market price, this is market based transfer pricing. A negotiated

Part 2 : 11/11/10 07:41:20

(c) HOCK international, page 65

transfer price would be one somewhere between $190 and $220 that is agreed on by the parties involved.

C. Since the transfer price that has been set is the market price, this is market based transfer pricing. A cost-based price would be $190.

D. Since the transfer price that has been set is the market price, this is market based transfer pricing.

Question 114 - CMA 1291 3-10 - Responsibility Accounting

A segment of an organization is referred to as a service center if it has

A. Authority to make decisions affecting the major determinants of profit including the power to choose its markets and sources of supply.B. Responsibility for developing markets and selling the output of the organization.C. Authority to provide specialized support to other units within the organization.D. Responsibility for combining the raw materials, direct labor, and other factors of production into a final output.

A. This is the definition of a profit center. See the correct answer for a complete explanation.

B. This is the definition of a revenue center. See the correct answer for a complete explanation.

C. By definition a service center provides support services to other departments of the organization.

D. This is the definition of a production center. See the correct answer for a complete explanation.

Question 115 - CIA 593 IV-16 - Transfer Pricing

Which of the following is the most significant disadvantage of a cost-based transfer price?

A. Requires internally developed information.B. Requires externally developed information.C. May not promote long-term efficiencies.D. Imposes market effects on company operations.

A. Other transfer price methods require internally generated information and the information that is needed for transfer pricing decisions is information that is needed for other purposes as well and is probably already produced by the company.

B. Externally generated information is required for market-based transfer pricing.

C. When the transfer price is based on the costs of production, the producing department has no incentive to control costs. Therefore, because there is no need to control costs, inefficiencies may creep into the process over time.

D. Market conditions being imposed within the company are a result of a market-based transfer price.

Question 116 - CMA 1287 4-30 - Variances - General

Todco planned to produce 3,000 units of its single product, Teragram, during November. The standard specifications

Part 2 : 11/11/10 07:41:20

(c) HOCK international, page 66

for one unit of Teragram include six pounds of materials at $.30 per pound. Actual production in November was 3,100 units of Teragram. The accountant computed a favorable materials purchase price variance of $380 and an unfavorable materials quantity variance of $120. Based on these variances, one could conclude that

A. More materials were used than were purchased.B. The actual cost of materials was less than the standard cost.C. More materials were purchased than were used.D. The actual usage of materials was less than the standard allowed.

A. A favorable materials purchase price variance and an unfavorable materials quantity variance do not relate to the quantity of materials purchased and used in production.

B. A favorable materials purchase price variance means the actual purchasing price was less than was budgeted (the standard).

C. A favorable materials purchase price variance and an unfavorable materials quantity variance do not relate to the quantity of materials purchased and used in production.

D. An unfavorable materials quantity variance means that more materials used in production than budgeted (standard). So the statement is incorrect.

Question 117 - CMA 1294 3-29 - Variances - Material and Labor

Water Control Inc. manufactures water pumps and uses a standard cost system. The standard factory overhead costs per water pump are based on direct labor hours and are as follows:

Variable overhead (4 hours at $8/hour) - $32Fixed overhead (4 hours at $5*/hour) - $20Total overhead cost per unit - $52* Based on a capacity of 100,000 direct labor hours per month.

The following additional information is available for the month of November:

22,000 pumps were produced although 25,000 had been scheduled for production.94,000 direct labor hours were worked at a total cost of $940,000.The standard direct labor rate is $9 per hour.The standard direct labor time per unit is 4 hours.Variable overhead costs were $740,000.Fixed overhead costs were $540,000.

The direct labor price variance for November was

A. $148,000 unfavorable.B. $60,000 favorable.C. $54,000 unfavorable.D. $94,000 unfavorable.

A. This is the total direct labor variance. See the correct answer for a complete explanation.

B. This answer is incorrect. See the correct answer for a complete explanation.

C. This is the direct labor efficiency variance. See the correct answer for a complete explanation.

D. The labor rate (price) variance is calculated as: (Standard Rate - Actual Rate) * Actual Hours. The standard rate is $9, the actual rate is $10 ($940,000 / 94,000), and 94,000 hours were actually worked during the period.

Part 2 : 11/11/10 07:41:20

(c) HOCK international, page 67

So, the labor price variance was $94,000 unfavorable [($9 - $10) * 94,000].

Question 118 - CMA 1296 3-16 - Responsibility Accounting

Rockford Manufacturing Corporation uses a responsibility accounting system in its operations. Which one of the following items is least likely to appear in a performance report for a manager of one of Rockford's assembly lines?

A. Repairs and maintenance.B. Materials.C. Direct labor.D. Depreciation on the manufacturing facility.

A. Repairs and maintenance would be controllable by the manager of an assembly line and would therefore be included on their report.

B. Materials would be controllable by the manager of an assembly line and would therefore be included on their report.

C. Direct labor would be controllable by the manager of an assembly line and would therefore be included on their report.

D. Depreciation is a cost that is dependent upon past decisions and the accounting policies adopted by management. Therefore, it is outside of the control of an assembly line manager and would not be included in their performance report.

Question 119 - CIA 594 III-44 - Performance Measurement

A firm prepared a segmented income statement that included the following data for its suburban marketing segment:

Fixed costs controllable by the suburban marketing segment manager $150,000 Fixed suburban marketing costs controllable by corporate management $250,000 Fixed manufacturing costs allocated to the suburban marketing segment $110,000 Variable manufacturing costs $200,000 Variable selling costs $100,000 Variable administrative costs $130,000 Net sales $950,000

The best measure of the economic performance of the suburban marketing segment is:

A. $520,000B. $370,000C. $120,000D. $10,000

A. This answer is the contribution margin of sales minus variable costs and fails to take into account the fixed costs that are controllable by the segment.

B. This answer does not include the fixed costs controllable by the company's management. This needs to be included in segment performance, but it would not be included in segment manager performance.

C.

Part 2 : 11/11/10 07:41:20

(c) HOCK international, page 68

The suburban marketing segment should be measured by what it controls and by fixed costs traceable to it, even if not controlled by it. This is called the segment margin, and the expenses included in it are the segment's variable costs, fixed costs it controls and fixed costs that are controllable by corporate management.

Segment manager performance includes only costs controllable by the segment manager. Segment manager performance would not include fixed costs controllable by the corporate management.

However, in this question we are measuring the segment, not the segment manager, because the question asks for the best measure of the economic performance of the segment. The segment margin includes the following items: Fixed costs controllable by the suburban marketing manager ($150,000), fixed costs controllable by corporate management ($250,000), the three types of variable costs ($430,000 in total) and the sales ($950,000). In total, this gives a margin for the segment of $120,000.

D. This answer incorrectly includes the allocated fixed costs. See the correct answer for a complete explanation.

Question 120 - CMA 696 3-25 - Overhead

Ardmore Enterprises uses a standard cost system in its small appliance division. The standard cost of manufacturing one unit of Zeb is as follows:

Materials - 60 pounds at $1.50 per pound $90 Labor - 3 hours at $12 per hour 36 Factory overhead - 3 hours at $8 per hour 24 Total standard cost per unit $150

The budgeted variable factory overhead rate is $3 per labor hour, and the budgeted fixed factory overhead is $27,000 per month. During May, Ardmore produced 1,650 units of Zeb compared with a normal capacity of 1,800 units. The actual cost per unit was as follows:

Materials (purchased and used) - 58 pounds at $1.65 per pound $95.70 Labor - 3.1 hours at $12 per hour 37.20 Factory overhead - $39,930 per 1,650 units 24.20 Total actual cost per unit $157.10

The flexible budget overhead variance for May is

A. $1,920 unfavorable.B. $3,270 unfavorable.C. $1,920 favorable.D. $3,270 favorable.

A. The flexible budget overhead variance is $1,920 favorable. The budgeted overhead is greater than actual which means the variance is favorable. See the correct answer for a complete explanation.

B. The flexible budget overhead variance is $1,920 favorable. The budgeted overhead is greater than actual which means the variance is favorable. See the correct answer for a complete explanation.

C. The flexible budget overhead variance equals the difference between the total actual overhead and the flexible budget total overhead. The flexible budget fixed overhead equals the master budget amount of $27,000. The budgeted variable factory overhead rate is $3 per labor hour, the standard hours to produce one unit of product is 3 hours, and 1,650 units were produced. Thus, the budgeted variable factory overhead was $14,850. The total flexible budget variable factory overhead is $41,850 ($14,850 + $27,000). The actual overhead costs were $39,930. The flexible budget overhead variance is $1,920 favorable ($41,850 - $39,930).

Part 2 : 11/11/10 07:41:20

(c) HOCK international, page 69

The budgeted overhead is greater than actual which means the variance is favorable.

D. This is the flexible budget variance based on the master budget level of output of 1,800 units. (($3 *3 * 1,800 + $27,000) - $39,930 = $3,2370 favorable(. However, we need to use the actual level of production of 1,650 units to calculate flexible budget figures. See the correct answer for a complete explanation.

Question 121 - CMA 1284 4-4 - Overhead

Dori Castings, a job-order shop, uses a full-absorption, standard-cost system to account for its production costs. The O/H costs are applied on a direct-labor-hour basis. The amount of fixed factory O/H that Dori will apply to finished production is the

A. Standard units of output for the actual direct labor hours worked times the standard fixed factory O/H rate per unit of output.B. Actual fixed factory O/H cost per direct labor hour times the standard allowed direct labor hours.C. Actual direct labor hours times the standard fixed factory O/H rate per direct labor hour.D. Standard allowed direct labor hours for the actual units of finished output times the standard fixed factory O/H rate per direct labor hour.

A. The amount of fixed factory applied calculated as Standard rate * Standard input allowed for the Actual level of output.

B. The amount of fixed factory applied calculated as Standard rate * Standard input allowed for the Actual level of output.

C. The amount of fixed factory applied calculated as Standard rate * Standard input allowed for the Actual level of output.

D. By definition. The amount of fixed factory applied calculated as Standard rate * Standard input allowed for the Actual level of output.

Question 122 - CMA 1291 3-1 - Variances - Material and Labor

Arrow Industries employs a standard cost system in which direct materials inventory is carried at standard cost. Arrow has established the following standards for the prime costs of one unit of product.

StandardQuantity

StandardPrice

StandardCost

Direct materials 8 pounds $1.80 per pound $14.40 Direct labor .25 hours 8.00 per hour 2.00 $16.40

During November, Arrow purchased 160,000 pounds of direct materials at a total cost of $304,000. The total factory wages for November were $42,000, 90% of which were for direct labor. Arrow manufactured 19,000 units of product during November using 142,500 pounds of direct materials and 5,000 direct labor hours.

The direct materials purchase price variance for November is

A. $14,250 unfavorable.B. $14,250 favorable.C. $16,000 unfavorable.

Part 2 : 11/11/10 07:41:20

(c) HOCK international, page 70

D. $16,000 favorable.

A. In the calculation of the materials purchase price variance we have to use the amount of materials purchased (160,000), not the amount consumed by production (142,500). See the correct answer for a complete explanation.

B. The variance is unfavorable because the actual price $1.90 ($304,000 ÷ 160,000) is higher than the standard price of $1.80. See the correct answer for a complete explanation.

C. The price variance is calculated as follows: (SP - AP) * AQ. The standard price is $1.80 per pound, and the actual price is $1.90 per pound ($304,000 ÷ 160,000). In the calculation of the materials purchase price variance we have to use the amount of materials purchased (160,000), not consumed by production (142,500). The price variance is $16,000 unfavorable [($1.80 - $1.90) * 160,000].

D. The variance is unfavorable because the actual price $1.90 ($304,000 ÷ 160,000) is higher than the standard price of $1.80. See the correct answer for a complete explanation.

Question 123 - CMA 693 3-19 - Variances - General

Tiny Tykes Corporation had the following activity relating to its fixed and variable overhead for the month of July.

Actual costs Fixed overhead $120,000 Variable overhead 80,000 Flexible budget Variable overhead 90,000 Applied Fixed overhead 125,000 Variable overhead spending variance 2,000F Production volume variance 5,000U

If the budgeted rate for applying variable manufacturing overhead was $20 per direct labor hour, how efficient or inefficient was Tiny Tykes Corporation in terms of using direct labor hours as an activity base?

A. 100 direct labor hours efficient.B. 400 direct labor hours inefficient.C. 100 direct labor hours inefficient.D. 400 direct labor hours efficient.

A. This answer is incorrect. See the correct answer for a complete explanation.

B. This answer is incorrect. See the correct answer for a complete explanation.

C. This answer is incorrect. See the correct answer for a complete explanation.

D.

The total variable overhead variance is equal to the difference between the actual variable overhead incurred and the budgeted variable overhead. This is also called the flexible budget variance. The formula is: Flexible Budget Amount - Actual Variable Overhead Incurred. Hence, total variable overhead variance is $10,000 favorable ($90,000 - $80,000). This total variance may be broken down into the spending and efficiency variances. The variable overhead spending variance is $2,000 favorable. The variable overhead efficiency variance equals total variable overhead variance minus variable overhead spending variance or $8,000 favorable ($10,000 - $2,000).

Part 2 : 11/11/10 07:41:20

(c) HOCK international, page 71

If the budgeted rate for applying variable manufacturing overhead was $20 per direct labor hour using direct labor hours as an activity base, this variance is equivalent to 400 direct labor hours ($8,000 ÷ $20). The number is positive which means the Tiny Tykes Corporation was 400 direct labor hours efficient.

Question 124 - CIA 594 III-40 - Transfer Pricing

Which of the following is not true about international transfer prices for a multinational firm?

A. Allows the firm to evaluate each division.B. Allows firms to attempt to minimize worldwide taxes.C. Provides each division with a profit-making orientation.D. Allows firms to correctly price products in each country in which it operates.

A. A well-devised transfer price enables the company to evaluate each of its divisions.

B. Though firms are greatly restricted by tax codes in the setting of transfer prices for tax purposes, transfer pricing does allow a firm a chance to move its profit to countries with lower tax rates.

C. If the transfer price is properly set, each division will be able to make decisions that are profit motivated, not governed from above.

D. The transfer price is the price used for internal transactions. This by itself does not enable the firm to determine the appropriate external price in each country in which it operates.

Question 125 - CMA 695 3-28 - Responsibility Accounting

In responsibility accounting, a center's performance is measured by controllable costs. Controllable costs are best described as including

A. Direct material and direct labor, only.B. Only those costs that the manager can influence in the current time period.C. Those costs about which the manager is knowledgeable and informed.D. Only discretionary costs.

A. Other types of costs (for example, marketing) are costs that may be controllable by a manager.

B. A definition of a controllable cost is a cost that the manager is able to influence in the time period under consideration.

C. Just because a manager has knowledge and information about a cost does not mean that the cost is controllable by a manager.

D. Other costs besides discretionary costs may also be controllable.

Question 126 - CIA 1190 IV-21 - Performance Measurement

When comparing the residual income of several investment centers, the validity of comparisons may be destroyed by

Part 2 : 11/11/10 07:41:20

(c) HOCK international, page 72

A. Consistent use of an imputed interest rate.B. Common amounts of invested capital for each investment center.C. Peculiarities of each investment center.D. None of the answers is correct.

A. The use of a consistent imputed interest rate is useful in the comparison of investment centers.

B. Common amounts of invested capital make the comparison of investment centers easier because there is then no size difference between them.

C. In any investment measure, the unique features of or situations faced by an individual investment center may make comparison between investment centers difficult.

D. One of the answers is correct.

Question 127 - CMA 1294 3-30 - Variances - Material and Labor

Water Control Inc. manufactures water pumps and uses a standard cost system. The standard factory overhead costs per water pump are based on direct labor hours and are as follows:

Variable overhead (4 hours at $8/hour) - $32Fixed overhead (4 hours at $5*/hour) - $20Total overhead cost per unit - $52* Based on a capacity of 100,000 direct labor hours per month.

The following additional information is available for the month of November:

22,000 pumps were produced although 25,000 had been scheduled for production.94,000 direct labor hours were worked at a total cost of $940,000.The standard direct labor rate is $9 per hour.The standard direct labor time per unit is 4 hours.Variable overhead costs were $740,000.Fixed overhead costs were $540,000.

The direct labor efficiency variance for November was

A. $120,000 favorable.B. $54,000 unfavorable.C. $108,000 favorable.D. $60,000 favorable.

A. The labor efficiency variance is calculated follows: (Standard Hours for Actual Output - Actual Quantity) * Standard Rate. There are three mistakes made to get this answer. First the actual labor rate ($10) is used instead of the standard rate ($9). Second, the capacity hours (100,000) is used instead of standard hours (88,000). Third, the standard hours (88,000) is used instead of actual hours (94,000). See the correct answer for a complete explanation.

B. The labor efficiency variance is calculated follows: (Standard Hours for Actual Output - Actual Quantity) * Standard Rate. The standard hours for actual output are 88,000 (4 hours per unit multiplied by 22,000 units produced). The actual hours worked are 94,000 and the standard labor hour rate is $9. The labor efficiency variance is $54,000 unfavorable [(88,000 - 94,000) * $9].

C. This number is the difference between standard hours (88,000) and capacity hours (100,000) multiplied by the standard rate. However, in this calculation we have to use actual hours (94,000) instead of capacity hours (100,000). See the correct answer for a complete explanation.

Part 2 : 11/11/10 07:41:20

(c) HOCK international, page 73

D. The labor efficiency variance is calculated follows: (Standard Hours for Actual Output - Actual Quantity) * Standard Rate. There are three mistakes in the formula above made to get this result. First the actual labor rate ($10) is used instead of ($9). Second, the capacity hours (100,000) is used instead of actual hours (94,000), third, the actual hours (94,000) is used instead of standard hours (88,000). See the correct answer for a complete explanation.

Question 128 - CMA 1294 3-20 - Responsibility Accounting

Fairmount Inc. uses an accounting system that charges costs to the manager who has been delegated the authority to make the decisions incurring the costs. For example, if the sales manager accepts a rush order that will result in higher than normal manufacturing costs, these additional costs are charged to the sales manager because the authority to accept or decline the rush order was given to the sales manager. This type of accounting system is known as

A. Transfer price accounting.B. Responsibility accounting.C. Reciprocal allocation.D. Functional accounting.

A. Transfer price accounting is related to the determination of the price to charge for intercompany sales.

B. Responsibility accounting is a system in which costs are allocated to managers and/or departments based on who is responsible for the incurrence of the costs. This is the method described in the question.

C. Reciprocal accounting is a method of allocating the costs of service departments to other service departments and the production departments.

D. Functional accounting does not use the responsibility for the incurrence of cost as the way of allocating the costs.

Question 129 - CMA 1295 3-7 - Overhead

The variance in an absorption costing system that measures the departure from the denominator level of activity that was used to set the fixed overhead rate is the

A. Production volume variance.B. Efficiency variance.C. Flexible budget variance.D. Spending variance.

A. In the budgeting process, the company must determine the level of activity to use. This level of activity is also called denominator level. As the budgeting takes place in the beginning of the year when the actual results are not known, the difference between the denominator and the actual results will most likely occur. The level of activity is simply the level of production. Thus, the difference (departure) from the denominator level of activity is the production volume variance.

B. This answer is incorrect. See the correct answer for a complete explanation.

C. The flexible budget variance is the difference between the actual results and the flexible budget figures.

D. The fixed overhead spending variance is the difference between actual fixed costs and budgeted fixed costs.

Part 2 : 11/11/10 07:41:20

(c) HOCK international, page 74

Question 130 - CIA 579 IV-2 - Variances - General

Which of the following factors should not be considered when deciding whether to investigate a variance?

A. Magnitude of the variance.B. Likelihood that an investigation will eliminate future occurrences of the variance.C. Trend of the variances over time.D. Whether the variance is favorable or unfavorable.

A. The magnitude (amount) of the variance should be considered in determining whether or not to investigate a variance. Management should give more attention to significant variances, measuring how material variance is. It is critical that benefits from variance investigation exceed its costs.

B. The investigation of variances especially negative variances usually takes place to eliminate future occurrence of the variance. In this process the responsible parties are investigated, corrective actions are taken or planned. Thus, it is logical to conduct investigation.

C. The trend of the variances over time should be considered in determining whether or not to investigate a variance. It is important to investigate variances when there is a strong tendency of negative variances to increase.

D. The behavior of variance (negative or positive) should not influence the decision of whether to investigate or not to investigate the variance. The variance is the measure of how much the actual results vary from the budgeted (expected) results. All significant variances should be investigated. It is also critical that benefits from variance investigation exceed its costs.

Question 131 - CIA 589 IV-16 - Transfer Pricing

Division A of a company is currently operating at 50% capacity. It produces a single product and sells all its production to outside customers for $13 per unit. Variable costs are $7 per unit, and fixed costs are $6 per unit at the current production level. Division B, which currently purchases this product from an outside supplier for $12 per unit, would like to purchase the product from Division A. Division A will operate at 80% capacity to meet outside customers' and Division B's demand. What is the minimum price that Division A should charge Division B for this product?

A. $13.00 per unit.B. $7.00 per unit.C. $9.60 per unit.D. $12.00 per unit.

A. When operating below capacity, the minimum price that the producing department will sell for is the variable cost of production. See the correct answer for a complete explanation.

B. When operating below capacity, the minimum price that the producing department will sell for is the variable cost of production. This is $7 in this question.

C. When operating below capacity, the minimum price that the producing department will sell for is the variable cost of production. See the correct answer for a complete explanation.

D. When operating below capacity, the minimum price that the producing department will sell for is the variable cost of production. See the correct answer for a complete explanation.

Question 132 - CMA 693 3-27 - Performance Measurement

Part 2 : 11/11/10 07:41:20

(c) HOCK international, page 75

Which one of the following statements pertaining to the return on investment (ROI) as a performance measurement is incorrect?

A. The use of ROI can make it undesirable for a skillful manager to take on trouble-shooting assignments such as those involving turning around unprofitable divisions.B. When the average age of assets differs substantially across segments of a business, the use of ROI may not be appropriate.C. The use of ROI may lead managers to reject capital investment projects that can be justified by using discounted cash flow models.D. ROI relies on financial measures that are capable of being independently verified while other forms of performance measures are subject to manipulation.

A. If ROI is used as the evaluation criteria, managers may not want to be responsible for working in divisions that are performing poorly. This is because the poorly performing division will have a difficult time generating the required return.

B. When there are substantially different ages of assets in different segments, the use of ROI may not be appropriate. This is because the assets will have different values based on their age and so each segment would have a different basis for what may be essentially the same assets.

C. Under ROI a manager may accept a project that has a positive net present value only because the return is not high enough in a percentage measurement.

D. ROI uses many of the same measures that other methods use and is therefore susceptible to the same manipulation as other methods are.

Question 133 - CMA 697 3-26 - Performance Measurement

Fabro Inc. produced 1,500 units of Product RX-6 last week. The inputs to the production process for Product RX-6 were as follows:

450 pounds of Material A at a cost of $1.50 per pound300 pounds of Material Z at a cost of $2.75 per pound300 labor hours at a cost of $15.00 per hour

What is the best productivity measure for the first-line supervisor in Fabro Inc.'s production plant?

A. $2.00 per pound.B. 0.33 units per dollar input.C. $15.00 per labor hour.D. 5.00 units per labor hour.

A. This is the average price for the pound of material used in production Product RX-6. However, the first-line supervisor does not control the price and his or her performance cannot be measured on this basis.

B. This is the units per labor dollar. However, the first-line supervisor does not control the labor rates and his or her performance cannot be measured on this basis.

C. This is the labor rate. However, the first-line supervisor does not control the labor rates and his or her performance cannot be measured on this basis.

D. The first-line supervisor is usually responsible for the productivity in terms of units produced during the period. He or she can not control the price of inputs (materials, labor). Thus, the adequate productivity measure for the first-line supervisor is the units produced by the labor hour, or 5.00 units per labor hour

Part 2 : 11/11/10 07:41:20

(c) HOCK international, page 76

(1,500 / 300).

Question 134 - CMA 1293 3-23 - Variances - Material and Labor

ChemKing uses a standard costing system in the manufacture of its single product. The 35,000 units of raw material in inventory were purchased for $105,000, and two units of raw material are required to produce one unit of final product. In November, the company produced 12,000 units of product. The standard allowed for material was $60,000, and there was an unfavorable quantity variance of $2,500.

The units of material used to produce November output totaled

A. 12,000 units.B. 12,500 units.C. 25,000 units.D. 23,000 units.

A. 12,000 is a number of units of finished product produced.

B. This answer is based on the assumption that only one unit of raw material is required to produce one unit of finished product. In fact, two units of raw material are required.

C. In this question, we need to solve for AQ and since we are given the quantity variance in the question, we can use this formula: (SQ - AQ) * SP = Quantity Variance. Or, (24,000 - AQ) * $2.50 = ($2,500). Solving for AQ we get 25,000. We also could have solved this in the following manner: because each unit cost $2.50 and ChemKing was $2,500 over budget, it must have used 1,000 units too many.

D. This answer is based on the assumption that the quantity material variance was favorable. It was, in fact, unfavorable.

Question 135 - CIA 590 IV-15 - Variances - General

A manager prepared the following table by which to analyze labor costs for the month:

Actual Hours atActual Rate

Actual Hours atStandard Rate

Standard Hours atStandard Rate

$10,000 $9,800 $8,820

What variance was $980?

A. Volume variance.B. Labor efficiency variance.C. Labor spending variance.D. Labor rate variance.

A. The volume variance is related to the fixed overhead and is the difference between the amount of fixed overhead applied (standard rate * standard input for the actual level of output) and the budgeted amount of fixed overhead. The volume variance cannot be determined using the given set of data.

B.

To solve that question we have to calculate suggested variances possible from this set of data. It's better to

Part 2 : 11/11/10 07:41:20

(c) HOCK international, page 77

start with most simple ones. The labor efficiency variance is: (Standard Hours for Actual Output - Actual Hours) * Standard Rate, or (Standard Hours for Actual Output * Standard Rate) - (Actual Hours * Standard Rate).

The standard hours times the standard rate is $8,820. The actual hours times the standard rate is $9,800. The difference is $(980) unfavorable.

C.

There is no such thing as a labor spending variance. Labor variances are either labor efficiency variances or labor rate variances.

The variable overhead spending variance is related to variable overhead and is the difference between the standard application rate and the actual application rate multiplied by the actual quantity of the application base (level of activity). This can also be calculated as the actual variable overhead incurred − (the actual usage of the application base * the standard variable overhead rate). This cannot be determined using the given set of data.

D.

To solve that question we have to calculate suggested variances possible from this set of data. It's better to start with most simple ones. Labor rate variance is: (Standard Rate - Actual Rate) * Actual Hours or (Standard Rate * Actual Hours) - (Actual Rate * Actual Hours).

The standard rate times the actual hours is $9,800. The actual rate times the actual hours is $10,000. The difference is $(200) unfavorable. This is not the result we are looking for.

Question 136 - CIA 1183 IV-5 - Transfer Pricing

A company has two divisions, A and B, each operated as a profit center. A charges B $35 per unit for each unit transferred to B. Other data follow:

A's variable cost per unit $30 A's fixed costs $10,000 A's annual sales to B 5,000 units A's sales to outsiders 50,000 units

A is planning to raise its transfer price to $50 per unit. Division B can purchase units at $40 each from outsiders, but doing so would idle A's facilities now committed to producing units for B. Division A cannot increase its sales to outsiders. From the perspective of the company as a whole, from whom should Division B acquire the units, assuming B's market is unaffected?

A. Division A, but only until fixed costs are covered, then from outside vendors.B. Division A, but only at the variable cost per unit.C. Division A, despite the increased transfer price.D. Outside vendors.

A.

The fixed costs are not relevant, since they will be the same regardless of where Division B gets the parts.

B. There are many acceptable methods of determining a transfer price. The best method for a particular situation depends upon the circumstances. This question asks what is the best course of action from the perspective of the company as a whole. Whenever the variable costs plus the opportunity cost of lost outside sales for the internal supplier are lower than the price of the external supplier, the company as a whole is better off if the buying department

Part 2 : 11/11/10 07:41:20

(c) HOCK international, page 78

buys internally, even if the transfer price is higher than an outside supplier's price. In this case, there is no opportunity cost of lost sales, because the question tells us that purchasing from outside would idle A's facilities now committed to producing units for B, and Division A cannot increase its sales to outsiders. Therefore, the total cost of producing the units internally is the variable cost of $30, which is lower than the outside price of $40. Thus, in this situation, the transfer cost used is not relevant to the decision.

C.

When the variable costs plus the opportunity cost of lost outside sales for the internal supplier are lower than the price of the external supplier, the company as a whole is better off if the buying department buys internally, even if the transfer price is higher than an outside supplier's price. In this case, there is no opportunity cost of lost sales, because the question tells us that purchasing from outside would idle A's facilities now committed to producing units for B, and Division A cannot increase its sales to outsiders. Therefore, the total cost of producing the units internally is the variable cost of $30, which is lower than the outside price of $40. Thus, from the perspective of the company as a whole, Division B should continue purchasing from Division A, despite the increased transfer price.

In reality, Department B should not simply accept the new transfer price of $50. The managers of the two divisions should agree on a transfer price that is acceptable to both of them. It may be that senior management will need to get involved in order to make that happen, if the two division heads cannot agree. However, that is not given as an answer choice, and of the answer choices given, this is the best choice.

D. Purchasing from outside in this situation will cause the company to pay more for each unit and to lose some contribution to its fixed costs from the internal sales. When the variable costs plus the opportunity cost of lost outside sales for the internal supplier are lower than the price of the external supplier, the company as a whole is better off if the buying department buys internally, even if the transfer price is higher than an outside supplier's price. In this case, there is no opportunity cost of lost sales, because the question tells us that purchasing from outside would idle A's facilities now committed to producing units for B, and Division A cannot increase its sales to outsiders. Therefore, the total cost of producing the units internally is the variable cost of $30, which is lower than the outside price of $40.

Question 137 - CMA 1289 4-3 - Overhead

A fixed overhead volume variance based on standard direct labor hours measures

A. Fixed overhead efficiency.B. Deviation from the normal, or denominator, level of direct labor hours.C. Deviation from standard direct labor hour capacity.D. Fixed overhead use.

A. There is no fixed overhead efficiency variance because fixed costs are not related to levels of output and therefore are unable to be used efficiently or inefficiently.

B. The fixed overhead volume variance is the difference between the amount of fixed overhead applied (standard rate * standard input for the actual level of output) and the budgeted amount of fixed overhead. The budgeted amount of fixed overhead is what we assumed as the expected level of usage at the beginning of the year when we set budget. In this situation overhead applied according to the labor hours allowed for actual level of output. If the output is more than budgeted the variance will be favorable as more overhead will be applied than was budgeted and vice versa.

C. The fixed overhead volume variance doesn't relate to the capacity of standard direct labor hours.

D. The fixed overhead volume variance doesn't relate to the fixed overhead usage.

Part 2 : 11/11/10 07:41:20

(c) HOCK international, page 79

Question 138 - CIA 1192 IV-17 - Performance Measurement

Data regarding four different products manufactured by an organization are presented as follows. Direct material and direct labor are readily available from the respective resource markets. However, the manufacturer is limited to a maximum of 3,000 machine hours per month.Products

A B C D Unit price $15 $18 $20 $25 Variable cost $7 $11 $10 $16

Units Produced per Machine Hour

A B C D 3 4 2 3

The product that is the most profitable for the manufacturer in this situation is

A. Product D.B. Product C.C. Product A.D. Product B.

A. Product D is the second most profitable product. See the correct answer for a complete explanation.

B. Product C is the least profitable product. See the correct answer for a complete explanation.

C. Product A is the third most profitable product. See the correct answer for a complete explanation.

D. In this question we deal with an operation with a constraint. To solve this question we have to identify which product has the highest contribution per machine hour (the limited recourse). First, we need calculate the contribution (sales minus variable cost) per each product: for product A $8 ($15 - $7), for product B $7 ($18 - $11), for product C $10 ($20 - $10), for product D $9 ($25 - $16). Knowing how many units of each product can be produced per machine hour, we can now calculate contribution per machine hour. For product A, the contribution per hour is $24 ($8 per unit * 3 units per machine hour), for product B it is $28 ($7 * 4), for product C it is $20 ($10 * 2), for product D it is $27 ($9 * 3). Comparing the calculated contribution per machine hour we select the highest, which is for product B.

Question 139 - CMA 1273 4-13 - Variances - General

Which of these variances is least significant for cost control?

A. Variable O/H spending variance.B. Materials quantity variance.C. Labor price variance.D. Fixed O/H volume variance.

A. The variable overhead spending variance is the difference between the standard application rate and the actual application rate (the actual application rate is calculated as the actual overhead costs ÷ the actual usage of the allocation base) multiplied by the actual quantity of the application base. Because actual variable costs are used in this variance, it is an important area of cost control.

B. Materials quantity variance is the difference in standard quantity of materials allowed for the actual level of output

Part 2 : 11/11/10 07:41:20

(c) HOCK international, page 80

and the quantity of materials actually used in production. It is an important area of cost control.

C. Labor price variance is the difference in labor rates (standard and actually used) in production. It is an important area of cost control.

D. The fixed overhead volume variance is the difference between the amount of fixed overhead applied (standard rate * standard input for the actual level of output) and the budgeted amount of fixed overhead. It does not reflect any differences between actual costs and budgeted costs. This variance concerns only the level of production. If the level of production is lower than in accordance with the master budget, the fixed overhead volume variance is unfavorable. If the production level is higher than that scheduled in the master budget, the fixed overhead volume variance will be favorable. Because this variance arises from a variance in the level of production, it is not very significant for cost control purposes.

Question 140 - CMA 1292 3-22 - Responsibility Accounting

When using a contribution margin format for internal reporting purposes, the major distinction between segment manager performance and segment performance is

A. Direct fixed cost controllable by the segment manager.B. Direct fixed cost controllable by others.C. Unallocated fixed cost.D. Direct variable costs of producing the product.

A. The difference between segment manager performance and segment performance is noncontrollable fixed costs that are traceable. Direct fixed costs controlled by the segment manager are included in both measures.

B. The difference between segment manager performance and segment performance is noncontrollable fixed costs that are traceable. These are fixed costs that are controlled by others.

C. The difference between segment manager performance and segment performance is noncontrollable fixed costs that are traceable. Unallocated fixed costs are not used in either measure.

D. The difference between segment manager performance and segment performance is noncontrollable fixed costs that are traceable. Direct variable costs of production are included in both measures.

Question 141 - CMA 696 3-28 - Transfer Pricing

Parkside Inc. has several divisions that operate as decentralized profit centers. Parkside's Entertainment Division manufactures video arcade equipment using the products of two of Parkside's other divisions. The Plastics Division manufactures plastic components, one type that is made exclusively for the Entertainment Division, while other less complex components are sold to outside markets. The products of the Video Cards Division are sold in a competitive market; however, one video card model is also used by the Entertainment Division. The actual costs per unit used by the Entertainment Division are presented below.

PlasticComponents

VideoCards

Direct material $1.25 $2.40 Direct labor 2.35 3.00 Variable overhead 1.00 1.50 Fixed overhead .40 2.25 Total cost $5.00 $9.15

Part 2 : 11/11/10 07:41:20

(c) HOCK international, page 81

The Plastics Division sells its commercial products at full cost plus a 25% markup and believes the proprietary plastic component made for the Entertainment Division would sell for $6.25 per unit on the open market. The market price of the video card used by the Entertainment Division is $10.98 per unit.

Assume that the Plastics Division has excess capacity and it has negotiated a transfer price of $5.60 per plastic component with the Entertainment Division. This price will

A. Encourage the Entertainment Division to seek an outside source for plastic components.B. Demotivate the Plastics Division causing mediocre performance.C. Cause the Plastics Division to reduce the number of commercial plastic components it manufactures.D. Motivate both divisions as estimated profits are shared.

A. Because the price is lower than the market price, the Entertainment Division will not seek an outside supplier.

B. Given that the transfer price is higher than the variable costs of production, there is no need for the producing division to be demotivated.

C. Because this price is above the variable costs of production, the Plastic Division would have no reason to reduce production because even what it sells internally generates a contribution.

D. Because the Plastics Division has excess capacity, this price is a mutually motivating price. The price for the purchasing division is less than it would pay in the market and the price for the selling division is higher than its variable costs of production. So, both divisions end up in a better position because of this price.

Question 142 - CMA 1293 3-25 - Overhead

A manufacturing firm planned to manufacture and sell 100,000 units of product during the year at a variable cost per unit of $4.00 and a fixed cost per unit of $2.00. The firm fell short of its goal and only manufactured 80,000 units at a total incurred cost of $515,000. The firm's manufacturing cost variance was

A. $35,000 unfavorable.B. $5,000 unfavorable.C. $85,000 favorable.D. $5,000 favorable.

A. This answer is incorrect. See the correct answer for a complete explanation.

B. This answer is incorrect. See the correct answer for a complete explanation.

C. This amount is based on the assumption of 100,000 units of output. See the correct answer for a complete explanation.

D. To solve this problem we can use the same price variance formula (SP - AP) * AQ, or SP * AQ - AP * AQ. The standard price in this case is standard cost for production. We already know what amount of AP * AQ is ($515,000). However, there is a little trick in this question: we must remember how fixed and variable costs behave as the level of production changes. The first half of price variance equation (SP * AQ) needs to be calculated in two steps. First, we determine how much of variable cost should incur according to the decreased level of production: standard amount of variable cost per unit of finished product is $4 and 80,000 units were actually produced; which gives us $320,000 of variable cost incurred. Second, we determine the amount of fixed cost should incurred according to budgeted standard and given level of output. Since fixed cost do not fluctuate with level of production we have to multiply the standard amount of fixed cost per unit of finished goods ($2) by planned amount of units to produce at the beginning of the year (100,000), which gives us $200,000. Adding VC and FC we get $520,000 ($320,000 + $200,000). Now we can calculate the cost variance: $520,000 - $515,000 = $5,000. Since the actual amount is less than planned this means the variance

Part 2 : 11/11/10 07:41:20

(c) HOCK international, page 82

is favorable.

Question 143 - CMA 696 3-24 - Variances - Material and Labor

Ardmore Enterprises uses a standard cost system in its small appliance division. The standard cost of manufacturing one unit of Zeb is as follows:

Materials - 60 pounds at $1.50 per pound $90 Labor - 3 hours at $12 per hour 36 Factory overhead - 3 hours at $8 per hour 24 Total standard cost per unit $150

The budgeted variable factory overhead rate is $3 per labor hour, and the budgeted fixed factory overhead is $27,000 per month. During May, Ardmore produced 1,650 units of Zeb compared with a normal capacity of 1,800 units. The actual cost per unit was as follows:

Materials (purchased and used) - 58 pounds at $1.65 per pound $95.70 Labor - 3.1 hours at $12 per hour 37.20 Factory overhead - $39,930 per 1,650 units 24.20 Total actual cost per unit $157.10

The labor rate variance for May is

A. $4,950 favorable.B. $0C. $1,920 favorable.D. $4,950 unfavorable.

A. The labor rate variance is zero as the standard rate equals the actual rate of $12.00 per hour. The answer given is the material quantity variance. See the correct answer for a complete explanation.

B. The labor rate variance formula is: (Standard Rate - Actual Rate) * Actual Hours. The labor rate variance is zero as the standard rate equals the actual rate of $12.00 per hour.

C. This is the difference between the actual total overhead and the flexible budget overhead. This is not the labor rate variance. See the correct answer for a complete explanation.

D. The labor rate variance is zero as the standard rate equals the actual rate of $12.00 per hour. See the correct answer for a complete explanation.

Question 144 - CMA 1290 3-8 - Overhead

Franklin Glass Works' production budget for the year ended November 30 was based on 200,000 units. Each unit requires two standard hours of labor for completion. Total overhead was budgeted at $900,000 for the year, and the fixed overhead rate was estimated to be $3.00 per unit. Both fixed and variable overhead are assigned to the product on the basis of direct labor hours. The actual data for the year ended November 30 are presented as follows.

Actual production in units 198,000 Actual direct labor hours 440,000 Actual variable overhead $352,000 Actual fixed overhead $575,000

Part 2 : 11/11/10 07:41:20

(c) HOCK international, page 83

Franklin's fixed overhead spending variance for the year is

A. $5,750 favorable.B. $19,000 favorable.C. $25,000 unfavorable.D. $25,000 favorable.

A. The fixed overhead budget/spending variance is the difference between the budgeted fixed overhead costs and the actual fixed overhead amount. See the correct answer for a complete explanation.

B. The fixed overhead budget/spending variance is the difference between the budgeted fixed overhead costs and the actual fixed overhead amount. See the correct answer for a complete explanation.

C. The fixed overhead budget/spending variance is the difference between the budgeted fixed overhead costs and the actual fixed overhead amount. See the correct answer for a complete explanation.

D. The fixed overhead budget/spending variance is the difference between the budgeted fixed overhead costs and the actual fixed overhead amount. The fixed overhead rate was estimated to be $3.00 per unit and 200,000 units were scheduled for production, thus, the total budgeted fixed overhead was $600,000 ($3.00*200,000). Actual fixed overhead was $575,000. The fixed overhead spending variance was $25,000 favorable ($600,000 - $575,000).

Question 145 - CIA 592 IV-19 - Transfer Pricing

Division Z of a company produces a component that it currently sells to outside customers for $20 per unit. At its current level of production, which is 60% of capacity, Division Z's fixed cost of producing this component is $5 per unit and its variable cost is $12 per unit. Division Y of the same company would like to purchase this component from Division Z for $10. Division Z has enough excess capacity to fill Division Y's requirements. The managers of both divisions are compensated based upon reported profits. Which of the following transfer prices will maximize total company profits and be most equitable to the managers of Division Y and Division Z?

A. $18 per unit.B. $12 per unit.C. $22 per unit.D. $20 per unit.

A. The transfer price should be between the variable costs of production ($12) and the market price ($20). $18 is the only option between these amounts.

B. The transfer price should be between the variable costs of production ($12) and the market price ($20). While $12 is at the edge of this range, the selling division will make no contribution from the sale, so they will not gain anything from this contract.

C. The transfer price should be between the variable costs of production ($12) and the market price ($20). $22 is outside of this range.

D. The transfer price should be between the variable costs of production ($12) and the market price ($20). $18 is the only option between these amounts. While $20 is at the edge of this range, the purchasing division will have no savings compared to the outside purchase price so would not gain anything by purchasing internally.

Question 146 - CMA 1296 3-17 - Transfer Pricing

Part 2 : 11/11/10 07:41:20

(c) HOCK international, page 84

In theory, the optimal method for establishing a transfer price is

A. Incremental cost.B. Flexible budget cost.C. Budgeted cost with or without a markup.D. Market price.

A. Cost based transfer pricing is perhaps the best method of transfer pricing if the department that is buying is not required to buy from another internal department. If the buying department is unable to choose its supplier, the manager of the supplying internal department will not have any motivation to control costs, because he or she will know that the costs will simply be charged to the next department.

B. Cost based transfer pricing is perhaps the best method of transfer pricing if the department that is buying is not required to buy from another internal department. If the buying department is unable to choose its supplier, the manager of the supplying internal department will not have any motivation to control costs, because he or she will know that the costs will simply be charged to the next department.

C. Cost based transfer pricing is perhaps the best method of transfer pricing if the department that is buying is not required to buy from another internal department. If the buying department is unable to choose its supplier, the manager of the supplying internal department will not have any motivation to control costs, because he or she will know that the costs will simply be charged to the next department.

D. The transfer price is the price charged by one unit of the company to another unit of the same company for the services or goods produced by the first unit and "sold" to the second unit. The basic issue of transfer prices is how much should one unit of a company charge another unit of the same company for its goods or services. The goal in setting a transfer price is that the method used will stimulate the department managers to do what will provide the greatest benefit to the company as a whole, rather than to act in their own interest. When there is an external market for the product, market price is almost always the best transfer price to use.

Question 147 - CMA 1291 3-3 - Variances - Material and Labor

Arrow Industries employs a standard cost system in which direct materials inventory is carried at standard cost. Arrow has established the following standards for the prime costs of one unit of product.

StandardQuantity

StandardPrice

StandardCost

Direct materials 8 pounds $1.80 per pound $14.40 Direct labor .25 hours 8.00 per hour 2.00 $16.40

During November, Arrow purchased 160,000 pounds of direct materials at a total cost of $304,000. The total factory wages for November were $42,000, 90% of which were for direct labor. Arrow manufactured 19,000 units of product during November using 142,500 pounds of direct materials and 5,000 direct labor hours.

The direct labor price (rate) variance for November is

A. $1,900 unfavorable.B. $2,090 favorable.C. $2,000 unfavorable.D. $2,200 favorable.

A. The labor rate/price variance is favorable as the standard direct labor rate is higher than actual. See the correct answer for a complete explanation.

Part 2 : 11/11/10 07:41:20

(c) HOCK international, page 85

B. The labor rate/price variance is calculated as: (Standard Rate - Actual Rate) * Actual Hours. As we can see from the formula we have to use actual hours (5,000), not hours allowed for production of actual output (4,750). See the correct answer for a complete explanation.

C. The labor rate/price variance is favorable as the standard direct labor rate is higher than actual. See the correct answer for a complete explanation.

D. The labor rate/price variance is: (Standard Rate - Actual Rate) * Actual Hours. The actual direct labor cost was $37,800 ($42,000 * 90%) for the month of November. The actual rate was $7.56 ($37,800 / 5,000). The labor rate/price variance was $2,200 favorable [($8.00 - $7.56) * 5,000].

Question 148 - CMA 1296 3-24 - Variances - Material and Labor

The inventory control supervisor at Wilson Manufacturing Corporation reported that a large quantity of a part purchased for a special order that was never completed remains in stock. The order was not completed because the customer defaulted on the order. The part is not used in any of Wilson's regular products. After consulting with Wilson's engineers, the vice president of production approved the substitution of the purchased part for a regular part in a new product. Wilson's engineers indicated that the purchased part could be substituted providing it was modified. The units manufactured using the substituted part required additional direct labor hours resulting in an unfavorable direct labor efficiency variance in the Production Department. The unfavorable direct labor efficiency variance resulting from the substitution of the purchased part in inventory would best be assigned to the

A. Vice president of production.B. Sales manager.C. Production manager.D. Inventory supervisor.

A. The vice president of production made the decision about the substitution of the regular materials for the special order materials. Thus, the unfavorable direct labor efficiency variance resulting from the substitution of the purchased part in inventory would best be assigned to the vice president of production.

B. The sales manager was not involved in production and decision making about the substitution of the regular materials for the special order materials.

C. Production manager was not involved in the decision making about the substitution of the regular materials for the special order materials.

D. The inventory supervisor was not involved in production and decision making about the substitution of the regular materials for the special order materials.

Question 149 - CMA 684 4-9 - Performance Measurement

Return on investment (ROI) is a term often used to express income earned on capital invested in a business unit. A company's ROI is increased if

A. Sales decrease by the same dollar amount that expenses increase.B. Sales remain the same and expenses are reduced by the same dollar amount that total assets increase.C. Sales increase by the same dollar amount as expenses and total assets.D. Net profit margin on sales increases by the same percentage as total assets.

A.

Part 2 : 11/11/10 07:41:20

(c) HOCK international, page 86

ROI is calculated as net income divided by total assets. One way to solve this problem is to set up some actual numbers for a basic ROI. For example, sales = $500,000, net income = $100,000 and total assets = $400,000. ROI = $100,000 ÷ $400,000, or .25.

If we decrease sales and increase expenses by $25,000 each, sales will decrease to $475,000, expenses will increase to $425,000, and net income will fall to $50,000. ROI will become $50,000 ÷ $400,000, which is lower.

B.

ROI is calculated as net income divided by total assets. One way to solve this problem is to simply set up some actual numbers for a basic ROI. For example, sales = $500,000, net income = $100,000 and total assets = $400,000. ROI = $100,000 ÷ $400,000, or .25.

If we reduce expenses by $50,000 and increase total assets by $50,000, net income will increase by $50,000 to $150,000 because sales remained the same while expenses were reduced. Total assets will increase by $50,000 to $450,000. ROI will change to $150,000 ÷ $450,000, which is .33, an increase.

C.

ROI is calculated as net income divided by total assets. One way to solve this problem is to set up some actual numbers for a basic ROI. For example, sales = $500,000, net income = $100,000 and total assets = $400,000. ROI = $100,000 ÷ $400,000, or .25.

If we increase sales, expenses and total assets by the same amounts, for example by $50,000, the new amounts will be: sales = $550,000, net income = $100,000 (unchanged because both sales and expenses increased by the same amount), and total assets = $450,000. ROI will become $100,000 ÷ $450,000, which is .22 and is a decrease, not an increase.

D.

ROI is calculated as net income divided by total assets. One way to solve this problem is to set up some actual numbers for a basic ROI. For example, sales = $500,000, net income = $100,000 and total assets = $400,000. ROI = $100,000 ÷ $400,000, or .25.

If we increase both the profit margin on sales and the assets of the company by the same percentage, say 10%, ROI will remain unchanged. Net income will increase by 10% to $110,000 and total assets will increase by 10% to $440,000. ROI will be $110,000 ÷ $440,000, which is unchanged at .25.

Question 150 - CMA 1293 3-24 - Variances - Material and Labor

ChemKing uses a standard costing system in the manufacture of its single product. The 35,000 units of raw material in inventory were purchased for $105,000, and two units of raw material are required to produce one unit of final product. In November, the company produced 12,000 units of product. The standard allowed for material was $60,000, and there was an unfavorable quantity variance of $2,500.

The materials price variance for the units used in November was

A. $12,500 unfavorable.B. $12,000 unfavorable.C. $6,000 unfavorable.D. $2,500 unfavorable.

A.

Part 2 : 11/11/10 07:41:20

(c) HOCK international, page 87

The formula for calculating the materials price variance is (SP − AP) × AQ. The standard price is $2.50 per unit of raw materials ($60,000 ÷ 12,000 units ÷ 2 units of materials per unit produced). The actual price was $105,000 ÷ 35,000 units, or $3.00 per unit. The Actual Quantity in the formula is the actual quantity of the raw materials that are used. It is not the actual quantity of product produced, nor is it the standard quantity of materials for the actual quantity produced.

So to calculate the price variance, we need to know how many units of materials were actually used in production, and the question does not tell us that. However, it does tell us that there was an unfavorable quantity variance. So to figure out how many units of materials were actually used, we first have to work out the quantity variance, which the question says is an unfavorable $2,500, to identify the AQ in that formula.

The quantity variance formula is (SQ − AQ) × AP. Since the question tells us that the materials standard is two units of raw materials for each unit produced, we know that the standard quantity of materials for 12,000 units was 24,000 units. The actual quantity we do not know yet. The standard price was $2.50 per unit of raw material ($60,000 standard allowed ÷ 12,000 units produced ÷ 2 units of materials per unit produced). Therefore, our formula is:

(24,000 − AQ) × 2.50 = (2,500)

Solving for AQ, we get AQ = 25,000

Now, we can put the actual quantity of materials used into the materials price variance formula and calculate the materials price variance.

(SP − AP) × AQ

($2.50 − $3.00) × 25,000 = ($12,500) Unfavorable

B. This answer results from using the standard amount of materials for the actual quantity produced as the AQ in the materials price variance formula, instead of the actual quantity of materials used. See the correct answer for a complete explanation.

C. This answer results from using the actual quantity of units produced as the AQ in the materials price variance formula, instead of the actual quantity of materials used. See the correct answer for a complete explanation.

D. $2,500 Unfavorable is the quantity variance, which is given in the question.

Question 151 - CMA 697 3-23 - Variances - Material and Labor

The controller for Durham Skates is reviewing the production cost report for July. An analysis of direct materials costs reflects an unfavorable flexible budget variance of $25. The plant manager believes this is excellent performance on a flexible budget for 5,000 units of direct materials. However, the production supervisor is not pleased with this result because he claims to have saved $1,200 in materials cost on actual production using 4,900 units of direct materials. The standard materials cost is $12 per unit. Actual materials used for the month amounted to $60,025. If the direct materials variance is investigated further, it will reflect a price variance of

A. $2,500 favorable.B. Zero.C. $1,225 unfavorable.D. $1,200 favorable.

A. The material price variance is $1,225 unfavorable. The standard price is lower than actual that gives us an unfavorable price variance. See the correct answer for a complete explanation.

Part 2 : 11/11/10 07:41:20

(c) HOCK international, page 88

B. The material price variance is $1,225 unfavorable. The standard price is lower than actual that gives us an unfavorable price variance. See the correct answer for a complete explanation.

C. The price variance is calculated as follows: (Standard Price - Actual Price) * Actual Quantity. The standard material price is $12.00 per unit, the actual price is $12.25 ($60,025 / 4,900), and the actual quantity is 4,900 units of direct materials. Hence, the material price variance is $1,225 unfavorable ($12.00 - $12,25) * 4,900. The standard price is lower than actual that gives us an unfavorable price variance.

D. The material price variance is $1,225 unfavorable. The standard price is lower than actual that gives us an unfavorable price variance. See the correct answer for a complete explanation.

Question 152 - CMA 1291 3-4 - Variances - Material and Labor

Arrow Industries employs a standard cost system in which direct materials inventory is carried at standard cost. Arrow has established the following standards for the prime costs of one unit of product.

StandardQuantity

StandardPrice

StandardCost

Direct materials 8 pounds $1.80 per pound $14.40 Direct labor .25 hours 8.00 per hour 2.00 $16.40

During November, Arrow purchased 160,000 pounds of direct materials at a total cost of $304,000. The total factory wages for November were $42,000, 90% of which were for direct labor. Arrow manufactured 19,000 units of product during November using 142,500 pounds of direct materials and 5,000 direct labor hours.

The direct labor usage (efficiency) variance for November is

A. $2,000 favorable.B. $1,800 unfavorable.C. $2,000 unfavorable.D. $2,200 favorable.

A. The labor efficiency variance is unfavorable as the hours allowed for the current level of production is less than was actually spent. See the correct answer for a complete explanation.

B. The labor efficiency variance is calculated the same way as the quantity variance was: (Standard Hours for Actual Output - Actual Quantity) * Standard Rate. See the correct answer for a complete explanation.

C. The labor efficiency variance is calculated the same way as the quantity variance was: (Standard Hours for Actual Output - Actual Quantity) * Standard Rate. We know that 19,000 units of product were manufactured during the period. The standard direct labor hours allowed for production of one unit is .25 hours. The standard direct labor hours allowed for production of the period is calculated as: 19,000 * .25 = 4,750 hours. Now we can calculate the labor efficiency variance: (4,750 - 5,000)*$8.00 = ($2,000) unfavorable.

D. The labor efficiency variance is unfavorable as the hours allowed for the current level of production is less than was actually spent. See the correct answer for a complete explanation.

Question 153 - CMA 1294 3-26 - Overhead

Water Control Inc. manufactures water pumps and uses a standard cost system. The standard factory overhead costs

Part 2 : 11/11/10 07:41:20

(c) HOCK international, page 89

per water pump are based on direct labor hours and are as follows:

Variable overhead (4 hours at $8/hour) - $32Fixed overhead (4 hours at $5*/hour) - $20Total overhead cost per unit - $52* Based on a capacity of 100,000 direct labor hours per month.

The following additional information is available for the month of November:

22,000 pumps were produced although 25,000 had been scheduled for production.94,000 direct labor hours were worked at a total cost of $940,000.The standard direct labor rate is $9 per hour.The standard direct labor time per unit is 4 hours.Variable overhead costs were $740,000.Fixed overhead costs were $540,000.

The fixed overhead spending variance for November was

A. $40,000 unfavorable.B. $70,000 unfavorable.C. $240,000 unfavorable.D. $460,000 unfavorable.

A. The fixed overhead budget/spending variance is simply the difference between the budgeted fixed overhead amount and the actual fixed overhead costs. The actual amount of fixed overhead costs was $540,000. The budgeted amount of fixed overhead was $500,000 ($5 of fixed overhead per labor hour multiplied by 100,000 budgeted labor hours). We use budgeted amount of labor hours (100,000) because we consider fixed costs in our calculation and fixed costs do not vary with the level of production. Thus, the amount of fixed cost budgeted is the same for any level of production. The fixed overhead budget/spending variance is $40,000 unfavorable ($500,000 - $540,000). The actual amount is higher than the budgeted amount and that is why this variance is unfavorable.

B. This is the answer if we use the actual amount of labor hours in order to calculate the budgeted fixed overhead costs. In calculation of budgeted amount of fixed overhead we have to use budgeted labor hours (100,000) not actual (94,000).

C. This number is the difference is the difference between the actual variable overhead and the budgeted fixed overhead. See the correct answer for a complete explanation.

D. This answer is incorrect. See the correct answer for a complete explanation.

Question 154 - CMA 1289 4-6 - Overhead

If overhead is applied on the basis of units of output, the variable overhead efficiency variance will be

A. Zero.B. Favorable, if output exceeds the budgeted level.C. A function of the direct labor efficiency variance.D. Unfavorable, if output is less than the budgeted level.

A. The formula for variable overhead efficiency variance is: (Standard Activity Level for Actual Output - Actual Activity Level) * Standard Application Rate. If overhead is applied on the basis of units of output there will be no difference between standard activity level for actual output and actual activity level. Therefore the variance will be equal to zero.

Part 2 : 11/11/10 07:41:20

(c) HOCK international, page 90

B. This answer is incorrect. See the correct answer for a complete explanation.

C. This answer is incorrect. See the correct answer for a complete explanation.

D. This answer is incorrect. See the correct answer for a complete explanation.

Question 155 - CMA 693 3-29 - Performance Measurement

Which one of the following firms is likely to experience dysfunctional motivation on the part of its managers due to its allocation methods?

A. Manhattan Electronics uses the sales revenue of its various divisions to allocate costs connected with the upkeep of its headquarters building. It also uses ROI to evaluate the divisional performances.B. To allocate depreciation of forklifts used by workers at its central warehouse, Shahlimar Electronics uses predetermined amounts calculated on the basis of the long-term average use of the services provided.C. Rainier Industrial does not allow its service departments to pass on their cost overruns to the production departments.D. Tashkent Auto's MIS is operated out of headquarters and serves its various divisions. Tashkent's allocation of the MIS-related costs to its divisions is limited to costs the divisions will incur if they were to outsource their MIS needs.

A. This question asks which of these allocation methods would lead to negative (dysfunctional) behavior on the part of managers. If a division receives more allocated costs for increased levels of sales, this will serve as a minor demotivator for the manager of the divisions. Also, using ROI to measure divisional performance is probably not the best method since the denominator in ROI will include things that are outside the control of the divisional managers.

B. This question asks which of these allocation methods would lead to negative (dysfunctional) behavior on the part of managers. The allocation of costs based on usage is a reasonable method for the allocation of these costs.

C. This question asks which of these allocation methods would lead to negative (dysfunctional) behavior on the part of managers. By not allowing cost overruns in the service departments to be passed to other departments, this will motivate the service departments to control their costs.

D. This question asks which of these allocation methods would lead to negative (dysfunctional) behavior on the part of managers. By allocating costs equal to a market value of the services provided, the management of the divisions will use the services in the necessary manner.

Question 156 - CIA 1191 IV-17 - Performance Measurement

The receipt of raw materials used in the manufacture of products and the shipping of finished goods to customers is under the control of the warehouse supervisor. The warehouse supervisor's time is spent approximately 60% on receiving activities and 40% on shipping activities. Separate staffs are employed for the receiving and shipping operations. The labor-related costs for the warehousing function are as follows:

Warehouse supervisor's salary $40,000 Receiving clerks' wages 75,000 Shipping clerks' wages 55,000 Employee benefit costs (30% of wage and salary costs) 51,000 $221,000

The company employs a responsibility accounting system for performance reporting purposes. The costs are

Part 2 : 11/11/10 07:41:20

(c) HOCK international, page 91

classified on the report as period or product costs. The total labor-related costs that would be listed on the responsibility accounting performance report as product costs under the control of the warehouse supervisor for the warehousing function would be

A. $128,700B. $97,500C. $169,000D. $130,000

A. This answer incorrectly assumes that the supervisor's salary is controllable by the supervisor. See the correct answer for a complete explanation.

B. In a responsibility accounting system, the only costs that a manager should be evaluated on are those that they control. The warehouse supervisor can control only the receiving clerks wages ($75,000 + 30% benefits for a total of $97,500). The supervisor's salary cannot be controlled by the supervisor and the shipping clerk's wages would be controlled by the shipping department.

C. This answer is incorrect. See the correct answer for a complete explanation.

D. This answer is incorrect. See the correct answer for a complete explanation.

Question 157 - CMA 1279 4-11 - Variances - General

The best basis upon which cost standards should be set to measure controllable production inefficiencies is

A. Normal capacity.B. Recent average historical performance.C. Engineering standards based on ideal performance.D. Engineering standards based on attainable performance.

A. Normal level of capacity is an average expected level of production within the time frame of several years given the reasonable expectations of effective and efficient production and customer demand. Since it is based on historical data it can't reflect possible changes in technology, which might take place, and the new standards have to take these changes into account.

B. Standards set on the basis of recent average historical performance don't reflect possible changes in technology, which might take place, and the new standards have to take these changes into account.

C. The ideal, perfect or theoretical level of output assumes that there are no breakdowns, no waste and no time lost to illness, and that the workers are already working at maximum efficiency. If standards are set using this capacity they will be unattainable. As a result, overhead costs will be underapplied.

D. Engineering standards based on attainable performance should be used standard setting. These standards take into consideration possible changes in technology. Capacity used to set standards should be attainable.

Part 2 : 11/11/10 07:41:20

(c) HOCK international, page 92